IASbaba's Flagship Course: Integrated Learning Programme (ILP) - 2024  Read Details

Posts

[DAY 34] 60 DAY RAPID REVISION (RaRe) SERIES for UPSC Prelims 2024 – SCIENCE AND TECHNOLOGY, CURRENT AFFAIRS & CSAT TEST SERIES!

Archives Hello Friends The 60 Days Rapid Revision (RaRe) Series is IASbaba’s Flagship Initiative recommended by Toppers and loved by the aspirants’ community every year. It is the most comprehensive program which will help you complete the syllabus, revise and practice tests on a daily basis. The Programme on a daily basis includes Daily Prelims MCQs from Static (Monday – Saturday) Daily Static Quiz will cover all the topics of static subjects – Polity, History, Geography, Economics, Environment and Science and technology. 20 questions will be posted daily and these questions are framed from the topics mentioned in the schedule. It will ensure timely and streamlined revision of your static subjects. Daily Current Affairs MCQs (Monday – Saturday) Daily 5 Current Affairs questions, based on sources like ‘The Hindu’, ‘Indian Express’ and ‘PIB’, would be published from Monday to Saturday according to the schedule. Daily CSAT Quiz (Monday – Friday) CSAT has been an Achilles heel for many aspirants. Daily 5 CSAT Questions will be published. Note – Daily Test of 20 static questions, 10 current affairs, and 5 CSAT questions. (35 Prelims Questions) in QUIZ FORMAT will be updated on a daily basis. To Know More about 60 Days Rapid Revision (RaRe) Series – CLICK HERE   60 Day Rapid Revision (RaRe) Series Schedule – CLICK HERE  Important Note Comment your Scores in the Comment Section. This will keep you accountable, responsible and sincere in days to come. It will help us come out with the Cut-Off on a Daily Basis. Let us know if you enjoyed today’s test 🙂  You can post your comments in the given format  (1) Your Score (2) Matrix Meter (3) New Learning from the Test Time limit: 0 Test-summary 0 of 35 questions completed Questions: 1 2 3 4 5 6 7 8 9 10 11 12 13 14 15 16 17 18 19 20 21 22 23 24 25 26 27 28 29 30 31 32 33 34 35 Information The following Test is based on the syllabus of 60 Days Plan-2023 for UPSC IAS Prelims 2022. To view Solutions, follow these instructions: Click on – ‘Start Test’ button Solve Questions Click on ‘Test Summary’ button Click on ‘Finish Test’ button Now click on ‘View Questions’ button – here you will see solutions and links. You have already completed the test before. Hence you can not start it again. Test is loading... You must sign in or sign up to start the test. You have to finish following test, to start this test: Results 0 of 35 questions answered correctly Your time: Time has elapsed You have scored 0 points out of 0 points, (0) Average score     Your score     Categories Not categorized 0% Your result has been entered into leaderboard Loading Name: E-Mail: Captcha: maximum of 70 points Pos. Name Entered on Points Result Table is loading No data available 1 2 3 4 5 6 7 8 9 10 11 12 13 14 15 16 17 18 19 20 21 22 23 24 25 26 27 28 29 30 31 32 33 34 35 Answered Review Question 1 of 35 1. Question Consider the following statements regarding Palliative Care: It is an approach that improves the quality of life of patients facing the problem associated with a life-threatening illness. Palliative Care is part of the ‘Mission Flexipool’ under National Health Mission (NHM). Which of the statements given above is/are correct? a) 1 only b) 2 only c) Both 1 and 2 d) Neither 1 nor 2 Correct Solution (c) Palliative care is also known as supportive care which is required in the terminal cases of Cancer, AIDS etc. and can be provided relatively simply and inexpensively. (Hence statement 1 is correct) Effective palliative care requires a broad multidisciplinary approach that includes the family and makes use of available community resources. The Ministry of Health and Family Welfare, Government of India constituted an expert group on Palliative care which submitted its report ‘Proposal of Strategies for Palliative Care in India’ in November, 2012. On the basis of the Report, an EPC note for 12th Five Year Plan was formulated. No separate budget is allocated for the implementation of National Palliative Care Program. However, the Palliative Care is part of the ‘Mission Flexipool’ under National Health Mission (NHM). (Hence statement 2 is correct) Incorrect Solution (c) Palliative care is also known as supportive care which is required in the terminal cases of Cancer, AIDS etc. and can be provided relatively simply and inexpensively. (Hence statement 1 is correct) Effective palliative care requires a broad multidisciplinary approach that includes the family and makes use of available community resources. The Ministry of Health and Family Welfare, Government of India constituted an expert group on Palliative care which submitted its report ‘Proposal of Strategies for Palliative Care in India’ in November, 2012. On the basis of the Report, an EPC note for 12th Five Year Plan was formulated. No separate budget is allocated for the implementation of National Palliative Care Program. However, the Palliative Care is part of the ‘Mission Flexipool’ under National Health Mission (NHM). (Hence statement 2 is correct) Question 2 of 35 2. Question Consider the following statements regarding Meningitis: It is an inflammation of the protective membranes covering the brain and spinal cord. Both Bacterial and Viral infections can cause Meningitis. Which of the statements given above is/are correct? a) 1 only b) 2 only c) Both 1 and 2 d) Neither 1 nor 2 Correct Solution (c) Meningitis is an inflammation (swelling) of the protective membranes covering the brain and spinal cord. These membranes are called meninges. The most common causes of meningitis are viral and bacterial infections. Other causes may include: cancer fungi drug-induced reactions Transmission: Infections that cause meningitis can be spread through sneezing and coughing.  (Hence option (c) is the correct answer) Symptoms: Common symptoms of meningitis are neck stiffness, fever, confusion or altered mental status, headaches, nausea and vomiting. Less frequent symptoms are seizures, coma and neurological deficits (for example hearing or vision loss, cognitive impairment, or weakness of the limbs). Types of meningitis caused by viruses or bacteria can have similar symptoms. Symptoms may be stronger in some types of meningitis than in others and require different treatment. Treatment and Prevention: Meningitis is a medical emergency. It is potentially fatal within 24 hours and requires urgent medical attention. Meningitis can vary in severity, appropriate treatment and care depending on the cause. Meningitis caused by bacteria requires immediate antibiotic treatment. Vaccines offer the best protection against common types of bacterial meningitis. Antibiotics for close contacts of those with meningococcal disease, when given promptly, decreases the risk of transmission. Hence both the statements are correct. Incorrect Solution (c) Meningitis is an inflammation (swelling) of the protective membranes covering the brain and spinal cord. These membranes are called meninges. The most common causes of meningitis are viral and bacterial infections. Other causes may include: cancer fungi drug-induced reactions Transmission: Infections that cause meningitis can be spread through sneezing and coughing.  (Hence option (c) is the correct answer) Symptoms: Common symptoms of meningitis are neck stiffness, fever, confusion or altered mental status, headaches, nausea and vomiting. Less frequent symptoms are seizures, coma and neurological deficits (for example hearing or vision loss, cognitive impairment, or weakness of the limbs). Types of meningitis caused by viruses or bacteria can have similar symptoms. Symptoms may be stronger in some types of meningitis than in others and require different treatment. Treatment and Prevention: Meningitis is a medical emergency. It is potentially fatal within 24 hours and requires urgent medical attention. Meningitis can vary in severity, appropriate treatment and care depending on the cause. Meningitis caused by bacteria requires immediate antibiotic treatment. Vaccines offer the best protection against common types of bacterial meningitis. Antibiotics for close contacts of those with meningococcal disease, when given promptly, decreases the risk of transmission. Hence both the statements are correct. Question 3 of 35 3. Question Consider the following statements: Most of the bacteria in human body are harmless. More than one billion people worldwide suffer from fungus infections each year, which are widespread and prevalent. A virus can replicate itself and infect cells. How many of the above statements are correct? a) Only one b) Only two c) All three d) None Correct Solution (b) Bacteria The human body is inhabited by millions of tiny living organisms, which, all together, are called the human microbiota. Bacteria are microbes found on the skin, in the nose, mouth, and especially in the gut. We acquire these bacteria during birth and the first years of life, and they live with us throughout our lives. The human microbiota is involved in healthy growth, in protecting the body from invaders, in helping digestion, and in regulating moods. Some changes in the microbiota may occur during our growth, depending on the foods we eat, the environment in which we live, the people and animals that interact with us, or medicines that we take, such as antibiotics. The human microbiota helps us to keep us healthy, but sometimes these bacteria can also be harmful. We need to take good care of our microbiota to avoid the development of some diseases, such as obesity and asthma. We should eat healthy foods that contribute to the development of a healthy microbiota. (Hence statement 1 is correct) Fungi The fungal disease sometimes referred to as mycosis, is a condition brought on by fungi. More than one billion people worldwide suffer from fungus infections each year, which are widespread and prevalent. (Hence statement 2 is correct) Other animals can develop a variety of fungal illnesses, some of which can be transferred from animals to people. Fungal infections are quite common and rare as well- they can be categorized by site of infection, geography, and immuno-comprised target as well as in biological classification. Most common: Fungal nail infections Vaginal candidiasis or vaginal yeast infection Ringworm Candida infections of the mouth, throat, and esophagus Fungal diseases based on geographic location Blastomycosis- caused by moist soil fungus in US and Canada Cryptococcus Gattai infection- tropical and sub-tropical areas Para coccidioidomycosis- central and south America Coccidioidomycosis (valley fever)- southwestern US, Mexico, central and south America Histoplasmosis- due to bird or bat droppings.   Virus Infected host cells (eukaryotic or prokaryotic) can be cultured and grown, and then the growth medium can be harvested as a source of the virus. Viruses replicate only within living cells. Some viruses are restricted in the kinds of cells in which they replicate, and a few have not yet been cultivated at all under laboratory conditions. However, most viruses are grown in cultured cells, embryonated hen’s eggs, or laboratory animals. (Hence statement 3 is incorrect) Incorrect Solution (b) Bacteria The human body is inhabited by millions of tiny living organisms, which, all together, are called the human microbiota. Bacteria are microbes found on the skin, in the nose, mouth, and especially in the gut. We acquire these bacteria during birth and the first years of life, and they live with us throughout our lives. The human microbiota is involved in healthy growth, in protecting the body from invaders, in helping digestion, and in regulating moods. Some changes in the microbiota may occur during our growth, depending on the foods we eat, the environment in which we live, the people and animals that interact with us, or medicines that we take, such as antibiotics. The human microbiota helps us to keep us healthy, but sometimes these bacteria can also be harmful. We need to take good care of our microbiota to avoid the development of some diseases, such as obesity and asthma. We should eat healthy foods that contribute to the development of a healthy microbiota. (Hence statement 1 is correct) Fungi The fungal disease sometimes referred to as mycosis, is a condition brought on by fungi. More than one billion people worldwide suffer from fungus infections each year, which are widespread and prevalent. (Hence statement 2 is correct) Other animals can develop a variety of fungal illnesses, some of which can be transferred from animals to people. Fungal infections are quite common and rare as well- they can be categorized by site of infection, geography, and immuno-comprised target as well as in biological classification. Most common: Fungal nail infections Vaginal candidiasis or vaginal yeast infection Ringworm Candida infections of the mouth, throat, and esophagus Fungal diseases based on geographic location Blastomycosis- caused by moist soil fungus in US and Canada Cryptococcus Gattai infection- tropical and sub-tropical areas Para coccidioidomycosis- central and south America Coccidioidomycosis (valley fever)- southwestern US, Mexico, central and south America Histoplasmosis- due to bird or bat droppings.   Virus Infected host cells (eukaryotic or prokaryotic) can be cultured and grown, and then the growth medium can be harvested as a source of the virus. Viruses replicate only within living cells. Some viruses are restricted in the kinds of cells in which they replicate, and a few have not yet been cultivated at all under laboratory conditions. However, most viruses are grown in cultured cells, embryonated hen’s eggs, or laboratory animals. (Hence statement 3 is incorrect) Question 4 of 35 4. Question Consider following statements regarding H5N1 virus: It is a highly pathogenic subtype of avian influenza (AI), a virus that causes severe disease and death in birds. It can cause illness in humans and many other animal species. Choose the correct code: a) 1 only b) 2 only c) Both 1 and 2 d) Neither 1 nor 2 Correct Solution (c) H5N1 is a highly pathogenic subtype of avian influenza (AI) that causes severe disease and death in birds. This subtype has caused a number of human infections through close contact with infected birds, or contaminated environments and is often fatal. It can cause illness in humans and many other animal species. A bird-adapted strain of H5N1, called HPAI A(H5N1) for highly pathogenic avian influenza virus of type A of subtype H5N1, is the highly pathogenic causative agent of H5N1 flu, commonly known as avian influenza (“bird flu”). It is enzootic (maintained in the population) in many bird populations, especially in Southeast Asia. (Hence both statements 1 and 2 are correct) Incorrect Solution (c) H5N1 is a highly pathogenic subtype of avian influenza (AI) that causes severe disease and death in birds. This subtype has caused a number of human infections through close contact with infected birds, or contaminated environments and is often fatal. It can cause illness in humans and many other animal species. A bird-adapted strain of H5N1, called HPAI A(H5N1) for highly pathogenic avian influenza virus of type A of subtype H5N1, is the highly pathogenic causative agent of H5N1 flu, commonly known as avian influenza (“bird flu”). It is enzootic (maintained in the population) in many bird populations, especially in Southeast Asia. (Hence both statements 1 and 2 are correct) Question 5 of 35 5. Question Which among the following cells has the ability to develop into any type of cell? a) Muscle cell b) Endodermal cell c) Ectodermal cell d) Stem cell Correct Solution (d) A stem cell is a cell with the unique ability to develop into specialized cell types in the body. (Hence option (d) is the correct answer) Blood stem cells are produced in the bone marrow and can become any kind of blood cell the body needs. Stem cells are constantly dividing and maturing into different types of blood cells, replacing older and worn-out blood cells in the body. They produce billions of new blood cells every day. If the stem cells cannot make enough new blood cells, many serious health problems can occur. These problems may include infections, anemia, or bleeding. Muscle cell A muscle cell is also known as a myocyte when referring to either a cardiac muscle cell (cardiomyocyte) or a smooth muscle cell, as these are both small cells. A skeletal muscle cell is long and threadlike with many nuclei and is called a muscle fiber. A muscle cell, known technically as a myocyte, is a specialized animal cell which can shorten its length using a series of motor proteins specially arranged within the cell. While several associated proteins help, actin and myosin form thick and thin filaments which slide past each other to contract small units of a muscle cell. These units are called sarcomeres, and many of them run end-to-end within a larger fiber called a myofibril. A single muscle cell contains many nuclei, which are pressed against the cell membrane. A muscle cell is a long cell compared to other forms of cells, and many muscle cells connect together to form the long fibers found in muscle tissue. Ectodermal cell The ectoderm is one of the primary layers of cells that exists in an embryo. The ectoderm cells differentiate into cells that form a number of external structures such as skin, sweat glands, skin sensor receptors, and hair follicles. Endodermal cells The endodermal cells generate only the lining of the digestive tube and its glands; mesodermal mesenchyme cells will surround this tube to provide the muscles for peristalsis. The ectoderm gives rise to the skin and the nervous system. The mesoderm specifies the development of several cell types such as bone, muscle, and connective tissue. Cells in the endoderm layer become the linings of the digestive and respiratory system, and form organs such as the liver and pancreas. Incorrect Solution (d) A stem cell is a cell with the unique ability to develop into specialized cell types in the body. (Hence option (d) is the correct answer) Blood stem cells are produced in the bone marrow and can become any kind of blood cell the body needs. Stem cells are constantly dividing and maturing into different types of blood cells, replacing older and worn-out blood cells in the body. They produce billions of new blood cells every day. If the stem cells cannot make enough new blood cells, many serious health problems can occur. These problems may include infections, anemia, or bleeding. Muscle cell A muscle cell is also known as a myocyte when referring to either a cardiac muscle cell (cardiomyocyte) or a smooth muscle cell, as these are both small cells. A skeletal muscle cell is long and threadlike with many nuclei and is called a muscle fiber. A muscle cell, known technically as a myocyte, is a specialized animal cell which can shorten its length using a series of motor proteins specially arranged within the cell. While several associated proteins help, actin and myosin form thick and thin filaments which slide past each other to contract small units of a muscle cell. These units are called sarcomeres, and many of them run end-to-end within a larger fiber called a myofibril. A single muscle cell contains many nuclei, which are pressed against the cell membrane. A muscle cell is a long cell compared to other forms of cells, and many muscle cells connect together to form the long fibers found in muscle tissue. Ectodermal cell The ectoderm is one of the primary layers of cells that exists in an embryo. The ectoderm cells differentiate into cells that form a number of external structures such as skin, sweat glands, skin sensor receptors, and hair follicles. Endodermal cells The endodermal cells generate only the lining of the digestive tube and its glands; mesodermal mesenchyme cells will surround this tube to provide the muscles for peristalsis. The ectoderm gives rise to the skin and the nervous system. The mesoderm specifies the development of several cell types such as bone, muscle, and connective tissue. Cells in the endoderm layer become the linings of the digestive and respiratory system, and form organs such as the liver and pancreas. Question 6 of 35 6. Question Which of the following best describes the Blastomycosis which was recently seen in the news? a) It is a fungal disease b) It is a bacterial disease c) It spreads through Mosquitoes d) None of these Correct Solution (a) Recently, at least one person died and around 100 were confirmed or suspected to be infected by blastomycosis in the United States. It is an infection caused by a fungus of the genus Blastomyces. It is found in moist soil and decomposing wood and leaves in the mid-western, south-central and south-eastern states of the US. Transmission: The microscopic spores from the fungus can get dispersed in the air and travel freely. Human beings can contract blastomycosis by inhaling the spores. It is not contagious between animals and people through the air. The symptoms of blastomycosis in animals are similar to the symptoms in humans. Symptoms: Fever, Cough, Breathing difficulty and muscle aches etc. Treatment: Itraconazole is a type of antifungal medication that is typically used to treat mild to moderate blastomycosis. Hence option (a) is the correct answer. Incorrect Solution (a) Recently, at least one person died and around 100 were confirmed or suspected to be infected by blastomycosis in the United States. It is an infection caused by a fungus of the genus Blastomyces. It is found in moist soil and decomposing wood and leaves in the mid-western, south-central and south-eastern states of the US. Transmission: The microscopic spores from the fungus can get dispersed in the air and travel freely. Human beings can contract blastomycosis by inhaling the spores. It is not contagious between animals and people through the air. The symptoms of blastomycosis in animals are similar to the symptoms in humans. Symptoms: Fever, Cough, Breathing difficulty and muscle aches etc. Treatment: Itraconazole is a type of antifungal medication that is typically used to treat mild to moderate blastomycosis. Hence option (a) is the correct answer. Question 7 of 35 7. Question Which of the following organizations recently launched “One Health Global Leaders Group on Antimicrobial Resistance (AMR)”? a) World Health Organization, UN Food and Agriculture Organization, and the International Organization on Animal Health. b) World Health Organization, Ministry of Health and Family Welfare, and the International Organization on Animal Health. c) World Health Organization, UN Food and Agriculture Organization, and the Ministry of Family Welfare d) World Health Organization, UN Environmental Programme, and the International Organization on Animal Health Correct Solution (a) Global Leaders Group on Antimicrobial Resistance The Global Leaders Group on Antimicrobial Resistance (AMR Leaders) consists of world leaders and experts from across sectors working together to accelerate political action on antimicrobial resistance (AMR). It was established in November 2020 following the recommendation of the Interagency Coordination Group (IACG) on Antimicrobial Resistance to strengthen global political momentum and leadership on AMR. The inaugural meeting of the Group took place in January 2021. One Health Global Leaders Group on Antimicrobial Resistance (AMR) launched by United Nations Tripartite organizations. World Health Organization (WHO), UN Food and Agriculture Organization (FAO), and the International Organization on Animal Health (OIE). (Hence, statement (a) is correct). The group performs an independent global advisory and advocacy role and works to maintain urgency, public support, political momentum and visibility of the AMR challenge on the global health and development agenda. India is a member of the Initiative.   Incorrect Solution (a) Global Leaders Group on Antimicrobial Resistance The Global Leaders Group on Antimicrobial Resistance (AMR Leaders) consists of world leaders and experts from across sectors working together to accelerate political action on antimicrobial resistance (AMR). It was established in November 2020 following the recommendation of the Interagency Coordination Group (IACG) on Antimicrobial Resistance to strengthen global political momentum and leadership on AMR. The inaugural meeting of the Group took place in January 2021. One Health Global Leaders Group on Antimicrobial Resistance (AMR) launched by United Nations Tripartite organizations. World Health Organization (WHO), UN Food and Agriculture Organization (FAO), and the International Organization on Animal Health (OIE). (Hence, statement (a) is correct). The group performs an independent global advisory and advocacy role and works to maintain urgency, public support, political momentum and visibility of the AMR challenge on the global health and development agenda. India is a member of the Initiative.   Question 8 of 35 8. Question Consider the following statements regarding National Centre for Disease Control (NCDC): It is the nodal agency in the country for disease surveillance, facilitating prevention and control of communicable diseases. It functions under the administrative control of Indian Council of Medical Research (ICMR). Which of the statements given above is/are correct? a) 1 only b) 2 only c) Both 1 and 2 d) Neither 1 nor 2 Correct Solution (a) The National Centre for Disease Control (NCDC) recently alerted states about smaller outbreaks for five diseases—typhoid, malaria, dengue, scrub typhus and hepatitis A. It is an institute under the Indian Directorate General of Health Services, Ministry of Health and Family Welfare. (Hence statement 2 is incorrect) The Director, an officer of the Public Health sub-cadre of Central Health Service, is the administrative and technical head of the Institute. History: The National Centre for Disease Control (NCDC), formerly National Institute of Communicable Diseases (NICD) had its origin as Central Malaria Bureau, established at Kasauli (Himachal Pradesh) in 1909 and following expansion was renamed in 1927 as the Malaria Survey of India. In view of the drastic reduction achieved in the incidence of malaria under National Malaria Eradication Programme (NMEP), Government of India decided to reorganize and expand the activities of the institute to cover other communicable diseases. Thus in 1963 the erstwhile MII was renamed as National Institute of Communicable Diseases (NICD) to shoulder these additional responsibilities. In year 2009, NICD transforms into National Centre for Disease Control (NCDC) with a larger mandate of controlling emerging and re-emerging diseases. Headquarters: New Delhi. Functions: It functions as the nodal agency in the country for disease surveillance facilitating prevention and control of communicable diseases. (Hence statement 1 is correct) In coordination with the State Governments, NCDC has the capacity and capability for disease surveillance, outbreak investigation, and rapid response to contain and combat outbreaks. It also deals with Anti-Microbial Resistance (AMR), an emerging area of concern with far-reaching consequences. It also provides referral diagnostic support, capacity building and technical support to States/UTs in the country. Incorrect Solution (a) The National Centre for Disease Control (NCDC) recently alerted states about smaller outbreaks for five diseases—typhoid, malaria, dengue, scrub typhus and hepatitis A. It is an institute under the Indian Directorate General of Health Services, Ministry of Health and Family Welfare. (Hence statement 2 is incorrect) The Director, an officer of the Public Health sub-cadre of Central Health Service, is the administrative and technical head of the Institute. History: The National Centre for Disease Control (NCDC), formerly National Institute of Communicable Diseases (NICD) had its origin as Central Malaria Bureau, established at Kasauli (Himachal Pradesh) in 1909 and following expansion was renamed in 1927 as the Malaria Survey of India. In view of the drastic reduction achieved in the incidence of malaria under National Malaria Eradication Programme (NMEP), Government of India decided to reorganize and expand the activities of the institute to cover other communicable diseases. Thus in 1963 the erstwhile MII was renamed as National Institute of Communicable Diseases (NICD) to shoulder these additional responsibilities. In year 2009, NICD transforms into National Centre for Disease Control (NCDC) with a larger mandate of controlling emerging and re-emerging diseases. Headquarters: New Delhi. Functions: It functions as the nodal agency in the country for disease surveillance facilitating prevention and control of communicable diseases. (Hence statement 1 is correct) In coordination with the State Governments, NCDC has the capacity and capability for disease surveillance, outbreak investigation, and rapid response to contain and combat outbreaks. It also deals with Anti-Microbial Resistance (AMR), an emerging area of concern with far-reaching consequences. It also provides referral diagnostic support, capacity building and technical support to States/UTs in the country. Question 9 of 35 9. Question Which of the following statements regarding Central Drugs Standard Control Organisation (CDSCO) is incorrect? a) It works under the Department of Pharmaceuticals, Ministry of Chemicals and Fertilizers. b) Drugs Controller General of India (DCGI) is the head of the department of the CDSCO of the Government of India. c) It is the Central Drug Authority for discharging functions assigned under the Drugs and Cosmetics Act of 1940. d) CDSCO along with state regulators, is jointly responsible for the grant of licenses to certain specialized categories of critical Drugs such as vaccines and sera, etc. Correct Solution (a) Central Drugs Standard Control Organisation (CDSCO) has recently started conducting joint inspections of identified Drug Manufacturing Units along with the State Drugs Control Administration. It is the Central Drug Authority for discharging functions assigned to the Central Government under the Drugs and Cosmetics Act of 1940. (Hence statement 3 is correct) It works under the Ministry of Health & Family Welfare, the National Regulatory Authority (NRA) of India. (Hence statement 1 is incorrect) Under the Drugs and Cosmetics Act, CDSCO is responsible for – Approval of Drugs. Conduct Clinical Trials. Laying down the standards for Drugs. Control over the quality of imported Drugs in the country. Coordination of the activities of State Drug Control Organizations. Further CDSCO along with state regulators, is jointly responsible for the grant of licenses to certain specialised categories of critical Drugs such as vaccines and sera, etc. (Hence statement 4 is correct) The Indian government has announced plans to subject all medical devices, including implants and contraception, to CDSCO scrutiny. Drugs Controller General of India (DCGI): DCGI is the head of the department of the CDSCO of the Government of India responsible for approval of licenses of specified categories of drugs such as blood and blood products, IV fluids, vaccines and sera in India. (Hence statement 2 is correct) DCGI also sets standards for the manufacturing, sales, import, and distribution of drugs in India. Incorrect Solution (a) Central Drugs Standard Control Organisation (CDSCO) has recently started conducting joint inspections of identified Drug Manufacturing Units along with the State Drugs Control Administration. It is the Central Drug Authority for discharging functions assigned to the Central Government under the Drugs and Cosmetics Act of 1940. (Hence statement 3 is correct) It works under the Ministry of Health & Family Welfare, the National Regulatory Authority (NRA) of India. (Hence statement 1 is incorrect) Under the Drugs and Cosmetics Act, CDSCO is responsible for – Approval of Drugs. Conduct Clinical Trials. Laying down the standards for Drugs. Control over the quality of imported Drugs in the country. Coordination of the activities of State Drug Control Organizations. Further CDSCO along with state regulators, is jointly responsible for the grant of licenses to certain specialised categories of critical Drugs such as vaccines and sera, etc. (Hence statement 4 is correct) The Indian government has announced plans to subject all medical devices, including implants and contraception, to CDSCO scrutiny. Drugs Controller General of India (DCGI): DCGI is the head of the department of the CDSCO of the Government of India responsible for approval of licenses of specified categories of drugs such as blood and blood products, IV fluids, vaccines and sera in India. (Hence statement 2 is correct) DCGI also sets standards for the manufacturing, sales, import, and distribution of drugs in India. Question 10 of 35 10. Question Consider the following statements regarding Monkey Pox Virus: It is a fungal zoonotic disease that occurs primarily in tropical rainforest areas of Central and West Africa. The first case of monkeypox in humans was recorded in 1970 in the Democratic Republic of Congo. Monkeypox can be eradicated using vaccine completely. How many of the above statements are correct? a) Only one b) Only two c) All three d) None Correct Solution (a) Recently, the World Health Organisation has declared the global monkeypox outbreak a ‘public health emergency of international concern’ (PHEIC). Monkeypox is a viral zoonotic disease that occurs primarily in tropical rainforest areas of Central and West Africa and is occasionally exported to other regions. (Hence statement 1 is incorrect) Monkeypox virus is an enveloped double-stranded DNA virus that belongs to the Orthopoxvirus genus of the Poxviridae family. The first case of monkeypox in humans was recorded in 1970 in the Democratic Republic of Congo during a period of intensified effort to eliminate smallpox. (Hence statement 2 is correct) Scientists have so far discovered two distinct genetic groups of monkeypox virus—Central African and West African. Cause It is caused by monkeypox virus, a member of the Orthopoxvirus genus in the family Poxviridae. Monkeypox belongs to the same family of viruses as smallpox. But Smallpox had a higher fatality rate than monkeypox. Smallpox was declared eradicated worldwide in 1980. Symptoms Monkeypox is characterised by flu-like symptoms and swelling of the lymph nodes, which gradually progresses to a widespread rash all over the face and body. Transmission Monkeypox virus is mostly transmitted to people from wild animals such as rodents and primates, but human-to-human transmission also occurs. It spreads from one person to another via respiratory droplets. Treatment There is currently no specific treatment recommended for monkeypox. (Hence statement 3 is incorrect) Vaccination against smallpox was about 85% effective in preventing monkeypox. Thus, prior childhood smallpox vaccination may result in a milder disease course. Incorrect Solution (a) Recently, the World Health Organisation has declared the global monkeypox outbreak a ‘public health emergency of international concern’ (PHEIC). Monkeypox is a viral zoonotic disease that occurs primarily in tropical rainforest areas of Central and West Africa and is occasionally exported to other regions. (Hence statement 1 is incorrect) Monkeypox virus is an enveloped double-stranded DNA virus that belongs to the Orthopoxvirus genus of the Poxviridae family. The first case of monkeypox in humans was recorded in 1970 in the Democratic Republic of Congo during a period of intensified effort to eliminate smallpox. (Hence statement 2 is correct) Scientists have so far discovered two distinct genetic groups of monkeypox virus—Central African and West African. Cause It is caused by monkeypox virus, a member of the Orthopoxvirus genus in the family Poxviridae. Monkeypox belongs to the same family of viruses as smallpox. But Smallpox had a higher fatality rate than monkeypox. Smallpox was declared eradicated worldwide in 1980. Symptoms Monkeypox is characterised by flu-like symptoms and swelling of the lymph nodes, which gradually progresses to a widespread rash all over the face and body. Transmission Monkeypox virus is mostly transmitted to people from wild animals such as rodents and primates, but human-to-human transmission also occurs. It spreads from one person to another via respiratory droplets. Treatment There is currently no specific treatment recommended for monkeypox. (Hence statement 3 is incorrect) Vaccination against smallpox was about 85% effective in preventing monkeypox. Thus, prior childhood smallpox vaccination may result in a milder disease course. Question 11 of 35 11. Question Which of the following are the applications of CRISPR Technology? Genetic sequence of disease-causing organisms can be altered to make them ineffective. Diseases and disorders including some forms of cancer caused by an undesired genetic mutation can be fixed. Till date genes of plants cannot be edited to make them pest resistant. How many of the above statements are correct? a) Only one b) Only two c) All three d) None Correct Solution (b) The CRISPR (Clustered Regularly Interspaced Short Palindromic Repeats) technology works in a simple way — it locates the specific area in the genetic sequence which has been diagnosed to be the cause of the problem, cuts it out, and replaces it with a new and correct sequence that no longer causes the problem. The technology replicates a natural defense mechanism in some bacteria that uses a similar method to protect itself from virus attacks. Because the entire process is programmable, it has a remarkable efficiency, and has already brought almost miraculous results. There are a whole lot of diseases and disorders, including some forms of cancer, that are caused by an undesired genetic mutation. These can all be fixed with this technology. (Hence statement 2 is correct) There are vast applications elsewhere as well. Genetic sequence of disease-causing organisms can be altered to make them ineffective. (Hence statement 1 is correct) Genes of plants can be edited to make them withstand pests, or improve their tolerance to drought or temperature. (Hence statement 3 is incorrect) Incorrect Solution (b) The CRISPR (Clustered Regularly Interspaced Short Palindromic Repeats) technology works in a simple way — it locates the specific area in the genetic sequence which has been diagnosed to be the cause of the problem, cuts it out, and replaces it with a new and correct sequence that no longer causes the problem. The technology replicates a natural defense mechanism in some bacteria that uses a similar method to protect itself from virus attacks. Because the entire process is programmable, it has a remarkable efficiency, and has already brought almost miraculous results. There are a whole lot of diseases and disorders, including some forms of cancer, that are caused by an undesired genetic mutation. These can all be fixed with this technology. (Hence statement 2 is correct) There are vast applications elsewhere as well. Genetic sequence of disease-causing organisms can be altered to make them ineffective. (Hence statement 1 is correct) Genes of plants can be edited to make them withstand pests, or improve their tolerance to drought or temperature. (Hence statement 3 is incorrect) Question 12 of 35 12. Question Consider the following statements regarding Alzheimer’s disease. It is a progressive heart disorder that typically affects older people. It affects the ability to perform day-to-day activities of human. Alzheimer’s disease can be treated using vaccines. How many of the above statements are correct? a) Only one b) Only two c) All three d) None Correct Solution (a) Alzheimer’s disease is a progressive brain disorder that typically affects people older than 65. (Hence statement 1 is incorrect) When it affects younger individuals, it is considered early onset. The disease destroys brain cells and nerves, and disrupts the message-carrying neurotransmitters. Eventually, a person with Alzheimer’s loses the ability to perform day-to-day activities. (Hence statement 2 is correct) Symptoms include memory loss, difficulty in completing familiar tasks, confusion with time or place, problems in speaking and writing, decreased or poor judgment, and changes in mood and personality. Alzheimer’s disease is also the most common cause of dementia — which is a syndrome and not a disease in itself, and whose symptoms include loss of memory, thinking skills, problems with language, changes in mood and deterioration in behaviour. Treatment: There is no cure for Alzheimer’s, because its exact causes are not known. (Hence statement 3 is incorrect) Most drugs being developed try to slow down or stop the progression of the disease. Incorrect Solution (a) Alzheimer’s disease is a progressive brain disorder that typically affects people older than 65. (Hence statement 1 is incorrect) When it affects younger individuals, it is considered early onset. The disease destroys brain cells and nerves, and disrupts the message-carrying neurotransmitters. Eventually, a person with Alzheimer’s loses the ability to perform day-to-day activities. (Hence statement 2 is correct) Symptoms include memory loss, difficulty in completing familiar tasks, confusion with time or place, problems in speaking and writing, decreased or poor judgment, and changes in mood and personality. Alzheimer’s disease is also the most common cause of dementia — which is a syndrome and not a disease in itself, and whose symptoms include loss of memory, thinking skills, problems with language, changes in mood and deterioration in behaviour. Treatment: There is no cure for Alzheimer’s, because its exact causes are not known. (Hence statement 3 is incorrect) Most drugs being developed try to slow down or stop the progression of the disease. Question 13 of 35 13. Question Consider the following statements: Human papillomavirus (HPV) is a viral infection of the reproductive tract. Majority of cervical cancer is due to HPV. India has recently developed its first human papillomavirus vaccine. How many of the above statements are correct? a) Only one b) Only two c) All three d) None Correct Solution (c) Cervavac, a vaccine developed by the Serum Institute of India (SII), has recently acquired market authorization from the Drugs Controller General of India (DGCI). It is India’s first Quadrivalent Human Papillomavirus Vaccination (qHPV) designed to protect women from cervical cancer. Cervical cancer is a prevalent sexually transmitted infection. It is a type of cancer that occurs in the cells of the cervix, the lower part of the uterus that connects to the vagina. It is mostly caused by long-term infection with particular forms of HPV. It is the second most prevalent cancer form and the second leading cause of cancer death in women of reproductive age (15-44). Hence all the statements are correct. Incorrect Solution (c) Cervavac, a vaccine developed by the Serum Institute of India (SII), has recently acquired market authorization from the Drugs Controller General of India (DGCI). It is India’s first Quadrivalent Human Papillomavirus Vaccination (qHPV) designed to protect women from cervical cancer. Cervical cancer is a prevalent sexually transmitted infection. It is a type of cancer that occurs in the cells of the cervix, the lower part of the uterus that connects to the vagina. It is mostly caused by long-term infection with particular forms of HPV. It is the second most prevalent cancer form and the second leading cause of cancer death in women of reproductive age (15-44). Hence all the statements are correct. Question 14 of 35 14. Question Consider the following statements regarding Artificial insemination and In-Vitro Fertilization: In-Vitro Fertilization technique introduces previously selected semen into the woman’s uterus that has been prepared by stimulating ovulation. Artificial insemination technique consists of retrieving a woman’s eggs to be fertilised in the laboratory. Which of the statements given above is/are correct? a) 1 only b) 2 only c) Both 1 and 2 d) Neither 1 nor 2 Correct Solution (d) Statement 1 Statement 2 Incorrect Incorrect In Vitro Fertilization- This technique consists of retrieving a woman’s eggs to be fertilised in the laboratory and later introducing the obtained embryos inside the uterus. Artificial insemination- This technique introduces previously selected semen into the woman’s uterus that has been prepared by stimulating ovulation. Incorrect Solution (d) Statement 1 Statement 2 Incorrect Incorrect In Vitro Fertilization- This technique consists of retrieving a woman’s eggs to be fertilised in the laboratory and later introducing the obtained embryos inside the uterus. Artificial insemination- This technique introduces previously selected semen into the woman’s uterus that has been prepared by stimulating ovulation. Question 15 of 35 15. Question Solution (b) Statement 1 Statement 2 Statement 3 Incorrect Correct Correct The strength of inactivated vaccines tends to wear off over time, resulting in less long-lasting immunity. Hence multiple doses are required to provide long-lasting immunity. Live-attenuated vaccines cannot be administered to persons with impaired immune response system. In a recombinant vaccine, only a protein from disease causing organism that generates protective immune response is inserted into gene of another cell. a) Only one b) Only two c) All three d) None Correct Test explained Incorrect Test explained Question 16 of 35 16. Question With reference to advancement in technology, consider the following statements: Viruses can be cultured without living cells. Fungi can be cultured only in natural medium. Bacteria can be cultured both naturally and artificial mediums. How many of the above statements are correct? a) Only one b) Only two c) All three d) None Correct Solution (a) Bacteria and Fungi can be cultured in an artificial/ synthetic medium. Whereas viruses require a living host cell for replication. Viruses replicate only within living cells. Some viruses are restricted in the kinds of cells in which they replicate, and a few have not yet been cultivated at all under laboratory conditions. Infected host cells (eukaryotic or prokaryotic) can be cultured and grown, and then the growth medium can be harvested as a source of the virus. (Hence statement 1 and 2 are incorrect and 3 is correct)   Incorrect Solution (a) Bacteria and Fungi can be cultured in an artificial/ synthetic medium. Whereas viruses require a living host cell for replication. Viruses replicate only within living cells. Some viruses are restricted in the kinds of cells in which they replicate, and a few have not yet been cultivated at all under laboratory conditions. Infected host cells (eukaryotic or prokaryotic) can be cultured and grown, and then the growth medium can be harvested as a source of the virus. (Hence statement 1 and 2 are incorrect and 3 is correct)   Question 17 of 35 17. Question Consider the following with respect to E.coli bacteria: coli bacteria are found in the gut of some animals. Most types of E. coli are harmless. They help in keeping the digestive tract healthy. How many of the above statements are correct? a) Only one b) Only two c) All three d) None Correct Solution (c) Statement 1 Statement 2 Statement 3 Correct Correct Correct E. coli (Escherichia coli), is a type of bacteria that normally lives in your intestines. It’s also found in the gut of some animals.  Hence, statement 1 is correct. Most types of E. coli are harmless and even help keep your digestive tract healthy. But some strains can cause diarrhea if you eat contaminated food or drink fouled water. Hence, statement 2 is correct. Most types of E. coli are harmless and even help keep your digestive tract healthy. But some strains can cause diarrhea if you eat contaminated food or drink fouled water. Hence, statement 3 is correct. Incorrect Solution (c) Statement 1 Statement 2 Statement 3 Correct Correct Correct E. coli (Escherichia coli), is a type of bacteria that normally lives in your intestines. It’s also found in the gut of some animals.  Hence, statement 1 is correct. Most types of E. coli are harmless and even help keep your digestive tract healthy. But some strains can cause diarrhea if you eat contaminated food or drink fouled water. Hence, statement 2 is correct. Most types of E. coli are harmless and even help keep your digestive tract healthy. But some strains can cause diarrhea if you eat contaminated food or drink fouled water. Hence, statement 3 is correct. Question 18 of 35 18. Question Consider the following: Antimicrobials are medicines used to prevent and treat infections in humans, animals and plants. Medical procedures, such as surgery, cancer chemotherapy, and organ transplantation, will not be affected by antimicrobial resistance. Which of the statements given above is/are correct? a) 1 only b) 2 only c) Both 1 and 2 d) Neither 1 nor 2 Correct Solution (a) Statement 1 Statement 2 Correct Incorrect Antimicrobials – including antibiotics, antivirals, antifungals and antiparasitics – are medicines used to prevent and treat infections in humans, animals and plants. Hence, statement 1 is true. Without effective tools for the prevention and adequate treatment of drug-resistant infections and improved access to existing and new quality-assured antimicrobials, the number of people for whom treatment is failing or who die of infections will increase. Medical procedures, such as surgery, including caesarean sections or hip replacements, cancer chemotherapy, and organ transplantation, will become riskier. Hence, statement 2 is not true. Incorrect Solution (a) Statement 1 Statement 2 Correct Incorrect Antimicrobials – including antibiotics, antivirals, antifungals and antiparasitics – are medicines used to prevent and treat infections in humans, animals and plants. Hence, statement 1 is true. Without effective tools for the prevention and adequate treatment of drug-resistant infections and improved access to existing and new quality-assured antimicrobials, the number of people for whom treatment is failing or who die of infections will increase. Medical procedures, such as surgery, including caesarean sections or hip replacements, cancer chemotherapy, and organ transplantation, will become riskier. Hence, statement 2 is not true. Question 19 of 35 19. Question Consider the following with respect to Spinal Muscular Atrophy (SMA): It is a neurological condition caused by a defect in the SMN1 gene. There are broadly three types of SMA depending on the age at which weakness starts and severity of weakness. SMA Type 1 being most severe. How many of the statements given above is/are correct? a) Only one b) Only two c) All three d) None Correct Solution (c) Statement 1,2,3 Correct Spinal Muscular Atrophy is a neurological condition caused by a defect in the SMN1 gene. Normally, every person is born with a gene called SMN1 which produces a protein called SMN protein, in many cells in our body. This protein is essential for normal functioning of nerve cells in the spinal cord called the ‘anterior horn cells’. The anterior horn cells control the skeletal muscles essential for all our movements (of limbs, neck, back, breathing and swallowing etc,). However, absence of SMN1 gene causes reduction in the amount of SMN protein produced in anterior horn cells. The reduced quantity of SMN protein causes gradual death of anterior horn cells, and thus progressive weakness of muscles of limbs, trunk and breathing and swallowing muscles. Depending on the age of child and severity of SMA, the speed with which the muscles become weaker varies. There are broadly three types of SMA depending on the age at which weakness starts and severity of weakness, with type 1 being most severe. Incorrect Solution (c) Statement 1,2,3 Correct Spinal Muscular Atrophy is a neurological condition caused by a defect in the SMN1 gene. Normally, every person is born with a gene called SMN1 which produces a protein called SMN protein, in many cells in our body. This protein is essential for normal functioning of nerve cells in the spinal cord called the ‘anterior horn cells’. The anterior horn cells control the skeletal muscles essential for all our movements (of limbs, neck, back, breathing and swallowing etc,). However, absence of SMN1 gene causes reduction in the amount of SMN protein produced in anterior horn cells. The reduced quantity of SMN protein causes gradual death of anterior horn cells, and thus progressive weakness of muscles of limbs, trunk and breathing and swallowing muscles. Depending on the age of child and severity of SMA, the speed with which the muscles become weaker varies. There are broadly three types of SMA depending on the age at which weakness starts and severity of weakness, with type 1 being most severe. Question 20 of 35 20. Question Consider the following statements: Adenoviruses have single-stranded DNA genomes whereas retroviruses have double-stranded DNA genomes. Common cold is mostly caused by an adenovirus whereas AIDS is caused by a retrovirus. Which of the statements given above is/are correct? a) 1 only b) 2 only c) Both 1 and 2 d) Neither 1 nor 2 Correct Solution (b) Statement 1 Statement 2 Incorrect Correct Adenoviruses are medium-sized, nonenveloped viruses with an icosahedral nucleocapsid containing a double-stranded DNA genome. Retroviruses (Rv) are double-stranded RNA viruses that integrate in to host the chromosome, and thus provide ‘permanent’ transfer of genes to a cell. Adenoviruses are common viruses that typically cause mild cold- or flu-like illness. Adenoviruses can cause illness in people of all ages any time of year.   HIV is classified as a retrovirus because it contains reverse transcriptase. Retroviruses are a family of viruses that are grouped together based on how they are structured and how they replicate within a host. Besides human immunodeficiency virus (HIV), the virus that causes AIDS, there two other retroviruses that can cause human illness. One is called human T-lymphotropic virus type 1 (HTLV-1) and the other is called human T-lymphotropic virus type 2 (HTLV-II). Both of these viruses are transmitted between people through sexual contact, infected blood or tissue exposure, or during pregnancy or childbirth from an infected person to their child. Incorrect Solution (b) Statement 1 Statement 2 Incorrect Correct Adenoviruses are medium-sized, nonenveloped viruses with an icosahedral nucleocapsid containing a double-stranded DNA genome. Retroviruses (Rv) are double-stranded RNA viruses that integrate in to host the chromosome, and thus provide ‘permanent’ transfer of genes to a cell. Adenoviruses are common viruses that typically cause mild cold- or flu-like illness. Adenoviruses can cause illness in people of all ages any time of year.   HIV is classified as a retrovirus because it contains reverse transcriptase. Retroviruses are a family of viruses that are grouped together based on how they are structured and how they replicate within a host. Besides human immunodeficiency virus (HIV), the virus that causes AIDS, there two other retroviruses that can cause human illness. One is called human T-lymphotropic virus type 1 (HTLV-1) and the other is called human T-lymphotropic virus type 2 (HTLV-II). Both of these viruses are transmitted between people through sexual contact, infected blood or tissue exposure, or during pregnancy or childbirth from an infected person to their child. Question 21 of 35 21. Question Consider the following statements regarding Valmiki Tiger Reserve (VTR): It forms the easternmost limit of the Himalayan Terai forests in India. It is the only tiger reserve in Jharkhand. It is surrounded by the Royal Chitwan National Park of Nepal in the north. Its vegetation is dominated by the moist mixed deciduous forest. How many of the above statements are correct? a) Only one b) Only two c) Only three d) All four Correct Solution (c) Valmiki Tiger Reserve (VTR) forms the easternmost limit of the Himalayan Terai forests in India. Hence statement 1 is correct. It is situated in the Gangetic plains bio-geographic zone of the country. The forest of this region has a combination of bhabar and terai tracts. Rivers Gandak, Pandai, Manor, Harha, Masan, and Bhapsa flow through various parts of the reserve. It is the only tiger reserve in the West Champaran district of Bihar. Palamu Tiger Reserve is the only tiger reserve in Jharkhand. Hence statement 2 is incorrect. It is surrounded by the Royal Chitwan National Park of Nepal in the north. Hence statement 3 is correct. Its vegetation is dominated by the moist mixed deciduous forest. Hence statement 4 is correct. It also includes open-land vegetation, sub-mountainous semi-evergreen formations, freshwater swamps, riparian fringes, alluvial grasslands, high hill savannahs, and wetlands. Incorrect Solution (c) Valmiki Tiger Reserve (VTR) forms the easternmost limit of the Himalayan Terai forests in India. Hence statement 1 is correct. It is situated in the Gangetic plains bio-geographic zone of the country. The forest of this region has a combination of bhabar and terai tracts. Rivers Gandak, Pandai, Manor, Harha, Masan, and Bhapsa flow through various parts of the reserve. It is the only tiger reserve in the West Champaran district of Bihar. Palamu Tiger Reserve is the only tiger reserve in Jharkhand. Hence statement 2 is incorrect. It is surrounded by the Royal Chitwan National Park of Nepal in the north. Hence statement 3 is correct. Its vegetation is dominated by the moist mixed deciduous forest. Hence statement 4 is correct. It also includes open-land vegetation, sub-mountainous semi-evergreen formations, freshwater swamps, riparian fringes, alluvial grasslands, high hill savannahs, and wetlands. Question 22 of 35 22. Question Consider the following statements about the Strait of Gibraltar: It is a narrow waterway separating Europe from Asia. It is bordered by Morocco in the north. It connects the Mediterranean Sea to the Atlantic Ocean. How many of the above statements are correct? a) Only one b) Only two c) All three d) None Correct Solution (a) The Strait of Gibraltar is a narrow waterway separating Europe from Africa. Hence statement 1 is incorrect. It is one of the busiest waterways in the world. Approximately 300 ships cross the Strait every day, about one ship every 5 minutes. It is bordered by Spain and the British Overseas Territory of Gibraltar in the north. It is bordered by Morocco and the Spanish exclave of Ceuta in the south. Hence statement 2 is incorrect. Its depth ranges from 300 to 900 meters, and it forms a significant gap between the high plateau of Spain and the Atlas Mountains of Northern Africa. It connects the Mediterranean Sea to the Atlantic Ocean. Hence statement 3 is correct. An important port located on the strait is the Moroccan port of Tanger-Med, near Tangier. Incorrect Solution (a) The Strait of Gibraltar is a narrow waterway separating Europe from Africa. Hence statement 1 is incorrect. It is one of the busiest waterways in the world. Approximately 300 ships cross the Strait every day, about one ship every 5 minutes. It is bordered by Spain and the British Overseas Territory of Gibraltar in the north. It is bordered by Morocco and the Spanish exclave of Ceuta in the south. Hence statement 2 is incorrect. Its depth ranges from 300 to 900 meters, and it forms a significant gap between the high plateau of Spain and the Atlas Mountains of Northern Africa. It connects the Mediterranean Sea to the Atlantic Ocean. Hence statement 3 is correct. An important port located on the strait is the Moroccan port of Tanger-Med, near Tangier. Question 23 of 35 23. Question Consider the following statements about Reindeers: They are known to see ultraviolet light. They are spotted in Europe, Asia, and North America. They are classified as critically endangered on the IUCN List. How many of the above statements are correct? a) Only one b) Only two c) All three d) None Correct Solution (b) Reindeers are known to see ultraviolet light. It helps them to spot predators whose white fur is more visible in UV light. Hence statement 1 is correct. They can use their nose to warm the air they breathe in before it enters their lungs. They are spotted in Europe, Asia, and North America. Hence statement 2 is correct. Both male and female reindeer grow antlers. However, male reindeer antlers tend to be twice as big. They are herbivores that are found in polar and arctic climates. They are classified as vulnerable on the IUCN List. Hence statement 3 is incorrect. Note: Researchers have discovered that the more time reindeer spend ruminating, the less time they spend in non-rapid eye movement (non-REM) sleep. It might help reindeer get enough sleep during the summer months when food is abundant and reindeer feed almost 24/7.   Incorrect Solution (b) Reindeers are known to see ultraviolet light. It helps them to spot predators whose white fur is more visible in UV light. Hence statement 1 is correct. They can use their nose to warm the air they breathe in before it enters their lungs. They are spotted in Europe, Asia, and North America. Hence statement 2 is correct. Both male and female reindeer grow antlers. However, male reindeer antlers tend to be twice as big. They are herbivores that are found in polar and arctic climates. They are classified as vulnerable on the IUCN List. Hence statement 3 is incorrect. Note: Researchers have discovered that the more time reindeer spend ruminating, the less time they spend in non-rapid eye movement (non-REM) sleep. It might help reindeer get enough sleep during the summer months when food is abundant and reindeer feed almost 24/7.   Question 24 of 35 24. Question Consider the following statements: Rights of Persons with Disabilities Act of 2016 includes mental illness, cerebral palsy, and muscular dystrophy as types of disabilities. India has ratified the United Nations Convention on the Rights of Persons with Disabilities. The subject of ‘relief of the disabled and unemployable’ is specified in the union list of the seventh schedule of the Indian constitution. How many of the above statements are correct? a) Only one b) Only two c) All three d) None Correct Solution (b) Rights of Persons with Disabilities Act of 2016 includes mental illness, cerebral palsy, autism, chronic neurological conditions, speech and language disability, thalassemia, hemophilia, sickle cell disease, multiple disabilities including deaf blindness, acid attack victims, Parkinson’s disease, and muscular dystrophy as types of disabilities. Hence statement 1 is correct. It increases the quantum of reservations for people suffering from disabilities from 3% to 4% in government jobs and from 3% to 5% in higher education institutes. Every child with a benchmark disability between the age group of 6 and 18 years shall have the right to free education. “Person with a disability” means a person with long-term physical, mental, intellectual, or sensory impairments which, in interaction with barriers, hinder his full and effective participation in society equally with others. India has ratified the United Nations Convention on Rights of Persons with Disability in 2007. Hence statement 2 is correct. It was adopted in 2006, and defines persons with disabilities as those “who have long-term physical, mental, intellectual or sensory impairments which in interaction with various barriers may hinder their full and effective participation in society on an equal basis with others”. The subject of ‘relief of the disabled and unemployable’ is specified in the state list of the seventh schedule of the Indian constitution. Hence statement 3 is incorrect. Article 41 of the Directive Principles of State Policy (DPSP) states that the State shall make effective provisions for securing the right to work, education, and public assistance in cases of unemployment, old age, sickness, and disablement, within the limits of its economic capacity and development.   Incorrect Solution (b) Rights of Persons with Disabilities Act of 2016 includes mental illness, cerebral palsy, autism, chronic neurological conditions, speech and language disability, thalassemia, hemophilia, sickle cell disease, multiple disabilities including deaf blindness, acid attack victims, Parkinson’s disease, and muscular dystrophy as types of disabilities. Hence statement 1 is correct. It increases the quantum of reservations for people suffering from disabilities from 3% to 4% in government jobs and from 3% to 5% in higher education institutes. Every child with a benchmark disability between the age group of 6 and 18 years shall have the right to free education. “Person with a disability” means a person with long-term physical, mental, intellectual, or sensory impairments which, in interaction with barriers, hinder his full and effective participation in society equally with others. India has ratified the United Nations Convention on Rights of Persons with Disability in 2007. Hence statement 2 is correct. It was adopted in 2006, and defines persons with disabilities as those “who have long-term physical, mental, intellectual or sensory impairments which in interaction with various barriers may hinder their full and effective participation in society on an equal basis with others”. The subject of ‘relief of the disabled and unemployable’ is specified in the state list of the seventh schedule of the Indian constitution. Hence statement 3 is incorrect. Article 41 of the Directive Principles of State Policy (DPSP) states that the State shall make effective provisions for securing the right to work, education, and public assistance in cases of unemployment, old age, sickness, and disablement, within the limits of its economic capacity and development.   Question 25 of 35 25. Question Consider the following statements regarding Huntington’s Disease (HD): It is a hereditary neurodegenerative disorder characterized by the progressive breakdown of nerve cells in the brain. It is caused by a mutation in the huntingtin gene (HTT), leading to the production of a defective form of the huntingtin protein. Its symptoms include involuntary movements, cognitive decline, and emotional disturbances. How many of the above statements are correct? a) Only one b) Only two c) All three d) None Correct Solution (c) Huntington’s Disease (HD) is a hereditary neurodegenerative disorder characterized by the progressive breakdown of nerve cells in the brain. Hence statement 1 is correct. It is caused by a mutation in the huntingtin gene (HTT), leading to the production of a defective form of the huntingtin protein. Hence statement 2 is correct. Its symptoms include involuntary movements, cognitive decline, and emotional disturbances. Hence statement 3 is correct. It is currently incurable and treatment cannot reverse its progression or slow it down. However, healthcare providers can offer medications to help with certain symptoms. Incorrect Solution (c) Huntington’s Disease (HD) is a hereditary neurodegenerative disorder characterized by the progressive breakdown of nerve cells in the brain. Hence statement 1 is correct. It is caused by a mutation in the huntingtin gene (HTT), leading to the production of a defective form of the huntingtin protein. Hence statement 2 is correct. Its symptoms include involuntary movements, cognitive decline, and emotional disturbances. Hence statement 3 is correct. It is currently incurable and treatment cannot reverse its progression or slow it down. However, healthcare providers can offer medications to help with certain symptoms. Question 26 of 35 26. Question Consider the following statements: Bikaji Cama established the Paris Indian Society with co-founders Munchershah Burjorji Godrej and S.R. Rana. Sucheta Kriplani founded the All-India Mahila Congress and was the first woman Chief Minister of Uttar Pradesh. Kanaklata Barua led a group of freedom fighters called Mukti Bahini during the Swadeshi Movement. How many of the above statements are correct? a) Only one b) Only two c) All three d) None Correct Solution (b) Bikaji Cama (1861-1936) established the Paris Indian Society with co-founders Munchershah Burjorji Godrej and S.R. Rana. Hence statement 1 is correct. She actively participated in the Indian freedom movement and advocated for Swaraj (self-rule). Sucheta Kriplani (1908-1974) founded the All India Mahila Congress in 1940 to advocate women’s rights and empowerment. She was the first woman Chief Minister of Uttar Pradesh. Hence statement 2 is correct. She worked closely with Mahatma Gandhi during the tumultuous time of the Partition riots, accompanying him to Noakhali in 1946. Kanaklata Barua (1924-1942) led a group of freedom fighters called Mukti Bahini during the Quit India Movement. She died during a procession to unfurl the Tricolour at Gohpur police station on September 20, 1942. Hence statement 3 is incorrect. Incorrect Solution (b) Bikaji Cama (1861-1936) established the Paris Indian Society with co-founders Munchershah Burjorji Godrej and S.R. Rana. Hence statement 1 is correct. She actively participated in the Indian freedom movement and advocated for Swaraj (self-rule). Sucheta Kriplani (1908-1974) founded the All India Mahila Congress in 1940 to advocate women’s rights and empowerment. She was the first woman Chief Minister of Uttar Pradesh. Hence statement 2 is correct. She worked closely with Mahatma Gandhi during the tumultuous time of the Partition riots, accompanying him to Noakhali in 1946. Kanaklata Barua (1924-1942) led a group of freedom fighters called Mukti Bahini during the Quit India Movement. She died during a procession to unfurl the Tricolour at Gohpur police station on September 20, 1942. Hence statement 3 is incorrect. Question 27 of 35 27. Question Consider the following statements about Polar Stratospheric Clouds (PSC): These are formed in very cold conditions over polar regions and within the troposphere. These clouds provide a surface for chemical reactions where benign forms of chlorine are converted into reactive, ozone-destroying forms. Choose the correct code: a) 1 only b) 2 only c) Both 1 and 2 d) Neither 1 nor 2 Correct Solution (b) Polar Stratospheric Clouds (PSC) are formed in very cold conditions over polar regions and within the stratosphere, around 12-19 miles (19-31km) high, far above our normal clouds. Hence statement 1 is incorrect. These clouds are made of smaller ice particles than those that form more common clouds. These small particles help to scatter light in a different way, which gives them their unique appearance. The frequency of these clouds is often a harbinger of ozone depletion, as they play a crucial role in stratospheric chemistry. These clouds provide a surface for chemical reactions where benign forms of chlorine are converted into reactive, ozone-destroying forms. Hence statement 2 is correct. They also remove nitrogen compounds that would otherwise moderate the destructive impact of chlorine on the ozone layer. Incorrect Solution (b) Polar Stratospheric Clouds (PSC) are formed in very cold conditions over polar regions and within the stratosphere, around 12-19 miles (19-31km) high, far above our normal clouds. Hence statement 1 is incorrect. These clouds are made of smaller ice particles than those that form more common clouds. These small particles help to scatter light in a different way, which gives them their unique appearance. The frequency of these clouds is often a harbinger of ozone depletion, as they play a crucial role in stratospheric chemistry. These clouds provide a surface for chemical reactions where benign forms of chlorine are converted into reactive, ozone-destroying forms. Hence statement 2 is correct. They also remove nitrogen compounds that would otherwise moderate the destructive impact of chlorine on the ozone layer. Question 28 of 35 28. Question Consider the following statements about the Bhoomi Rashi Portal: It is an e-governance initiative of the Ministry of Road Transport and Highways. It intends to expedite the process of land acquisition for National Highways. Choose the correct code: a) 1 only b) 2 only c) Both 1 and 2 d) Neither 1 nor 2 Correct Solution (c) The Bhoomi Rashi Portal is an e-governance initiative of the Ministry of Road Transport and Highways. Hence statement 1 is correct. It acts as a single-point platform for the online processing of land acquisition notifications to accelerate highway infrastructure development projects in India. It intends to expedite the process of land acquisition for National Highways. Hence statement 2 is correct. This portal would enhance the efficiency of the land acquisition process, ensure transparency and accountability, and result in the e-transfer of benefits directly to the accounts of the beneficiaries.   Incorrect Solution (c) The Bhoomi Rashi Portal is an e-governance initiative of the Ministry of Road Transport and Highways. Hence statement 1 is correct. It acts as a single-point platform for the online processing of land acquisition notifications to accelerate highway infrastructure development projects in India. It intends to expedite the process of land acquisition for National Highways. Hence statement 2 is correct. This portal would enhance the efficiency of the land acquisition process, ensure transparency and accountability, and result in the e-transfer of benefits directly to the accounts of the beneficiaries.   Question 29 of 35 29. Question Consider the following statements about Divya Kala Mela: It is a unique event showcasing the products and craftsmanship of Divyang entrepreneurs and artisans from across the country. It is organised by the Ministry of Culture through the National Divyangjan Finance and Development Corporation (NDFDC). Choose the correct code: a) 1 only b) 2 only c) Both 1 and 2 d) Neither 1 nor 2 Correct Solution (a) Divya Kala Mela is a unique event showcasing the products and craftsmanship of Divyang entrepreneurs and artisans from across the country. Hence statement 1 is correct. It aims to strengthen the economic empowerment of Divyangjan and provide a bigger platform for marketing and showcasing the products and skills of Divyangjan. It is organised by the Department of Empowerment of Persons with Disabilities (Divyangjan), Ministry of Social Justice and Empowerment (MoSJ&E) through the National Divyangjan Finance and Development Corporation (NDFDC). Hence statement 2 is incorrect. Incorrect Solution (a) Divya Kala Mela is a unique event showcasing the products and craftsmanship of Divyang entrepreneurs and artisans from across the country. Hence statement 1 is correct. It aims to strengthen the economic empowerment of Divyangjan and provide a bigger platform for marketing and showcasing the products and skills of Divyangjan. It is organised by the Department of Empowerment of Persons with Disabilities (Divyangjan), Ministry of Social Justice and Empowerment (MoSJ&E) through the National Divyangjan Finance and Development Corporation (NDFDC). Hence statement 2 is incorrect. Question 30 of 35 30. Question Consider the following statements about Kanheri Caves:  The name Kanheri occurs in the Nasik inscription of the Satavahana ruler Vasisthiputra Pulumavi. The earliest reference of Kanheri is ascribed to Megasthenes who visited India during the reign of Chandragupta Maurya. They are located within the forests of the Sanjay Gandhi National Park. How many of the above statements are correct? a) Only one b) Only two c) All three d) None Correct Solution (b) Kanheri Caves are a group of caves and rock-cut monuments located on the western outskirts of Mumbai. The name Kanheri is derived from ‘Kanhagiri’ in Prakrit. The name Kanheri occurs in the Nasik inscription of the Satavahana ruler Vasisthiputra Pulumavi. Hence statement 1 is correct. The Kanheri caves comprise more than 110 different rock-cut monolithic excavations and are one of the largest single excavations in the country. The earliest reference to Kanheri is ascribed to Fa-Hein who visited India during 399-411 CE. Hence statement 2 is incorrect. Kanheri Caves flourished under the patronage of Satavahana, Traikutakas, Vakatakas, and Silaharas. They are located within the forests of the Sanjay Gandhi National Park. Hence statement 3 is correct. Note: Grey Francolin (Ortygornis pondicerianus) was reported inside the Sanjay Gandhi National Park (SGNP) of Mumbai. Grey Francolin is a ground-dwelling bird known for its distinctive calls and cryptic plumage. They are carnivores (insectivores) and herbivores (granivores). Incorrect Solution (b) Kanheri Caves are a group of caves and rock-cut monuments located on the western outskirts of Mumbai. The name Kanheri is derived from ‘Kanhagiri’ in Prakrit. The name Kanheri occurs in the Nasik inscription of the Satavahana ruler Vasisthiputra Pulumavi. Hence statement 1 is correct. The Kanheri caves comprise more than 110 different rock-cut monolithic excavations and are one of the largest single excavations in the country. The earliest reference to Kanheri is ascribed to Fa-Hein who visited India during 399-411 CE. Hence statement 2 is incorrect. Kanheri Caves flourished under the patronage of Satavahana, Traikutakas, Vakatakas, and Silaharas. They are located within the forests of the Sanjay Gandhi National Park. Hence statement 3 is correct. Note: Grey Francolin (Ortygornis pondicerianus) was reported inside the Sanjay Gandhi National Park (SGNP) of Mumbai. Grey Francolin is a ground-dwelling bird known for its distinctive calls and cryptic plumage. They are carnivores (insectivores) and herbivores (granivores). Question 31 of 35 31. Question (1/9), (1/3), 1, ?, 9. What number should fill the Question mark? a) (2/3) b) 3 c) 3/9 d) 6 Correct Solution (b) This is a multiplication series; each number is 3 times the previous number. Incorrect Solution (b) This is a multiplication series; each number is 3 times the previous number. Question 32 of 35 32. Question The difference of two numbers is 1365. On dividing the larger number by the smaller, we get 6 as quotient and the 15 as remainder. What is the smaller number? a) 270 b) 295 c) 240 d) 360 Correct Solution (a) Let the smaller number be x Then larger number = (x + 1365) ∴ x + 1365 = 6x + 15 ⇒ 5x = 1350 ⇒ x = 270 ∴ Smaller number = 270 Incorrect Solution (a) Let the smaller number be x Then larger number = (x + 1365) ∴ x + 1365 = 6x + 15 ⇒ 5x = 1350 ⇒ x = 270 ∴ Smaller number = 270 Question 33 of 35 33. Question In a race of 200m, B can give a start of 10m to A and C can give a start of 20m to B. The start that C can give to A in the same race is? a) 27m b) 30m c) 25m d) 29m Correct Solution (d) B : A = 200 : 190 C : B = 200 : 180 = C/A = (C/B)*(B/A) = (200/180)*(200/190) = 200/171 ∴ C can give to A, a start of (200 – 171) m = 29 m Incorrect Solution (d) B : A = 200 : 190 C : B = 200 : 180 = C/A = (C/B)*(B/A) = (200/180)*(200/190) = 200/171 ∴ C can give to A, a start of (200 – 171) m = 29 m Question 34 of 35 34. Question Find the HCF of (3125-1) and (335-1) a) 312-1 b) 35-1 c) 34-1 d) None of these Correct Solution (b) The solution of this question is based on the rule, The HCF of (am – 1) and (an – 1) is given by (aHCF of m, n – 1) Thus for this question the answer is (35 – 1) Since, 5 is the HCF of 35 and 125. Incorrect Solution (b) The solution of this question is based on the rule, The HCF of (am – 1) and (an – 1) is given by (aHCF of m, n – 1) Thus for this question the answer is (35 – 1) Since, 5 is the HCF of 35 and 125. Question 35 of 35 35. Question Prof. Suman takes a number of quizzes for a course. All the quizzes are out of 100. A student can get an A grade in the course if the average of her scores is more than or equal to 90. Grade B is awarded to a student if the average of her scores is between 87 and 89 (both included). If the average is below 87, the student gets a C grade. Ramesh is preparing for his last quiz, and he realizes that he must score a minimum of 97 to get an A grade. After the quiz, he realizes that he will score 70, and he will just manage a B. How many quizzes did Prof. Suman take? a) 6 b) 7 c) 8 d) 9 Correct Solution (d) Let the number of quizzes before the last quiz be x. Let the average score of Ramesh be n. According to the question, ((nx+97)/x) = 9090x – nx = 97…….(1) And, ((nx+70)/x) = 8787x – nx = 70……..(2) Subtract both equations 3x = 27x = 9 Hence, the number of quizzes is 9. Incorrect Solution (d) Let the number of quizzes before the last quiz be x. Let the average score of Ramesh be n. According to the question, ((nx+97)/x) = 9090x – nx = 97…….(1) And, ((nx+70)/x) = 8787x – nx = 70……..(2) Subtract both equations 3x = 27x = 9 Hence, the number of quizzes is 9. window.wpProQuizInitList = window.wpProQuizInitList || []; window.wpProQuizInitList.push({ id: '#wpProQuiz_3631', init: { quizId: 3631, mode: 1, globalPoints: 70, timelimit: 1800, resultsGrade: [0], bo: 704, qpp: 0, catPoints: [70], formPos: 0, lbn: "Test-summary", json: {"32097":{"type":"single","id":32097,"catId":0,"points":2,"correct":[0,0,1,0]},"32098":{"type":"single","id":32098,"catId":0,"points":2,"correct":[0,0,1,0]},"32099":{"type":"single","id":32099,"catId":0,"points":2,"correct":[0,1,0,0]},"32102":{"type":"single","id":32102,"catId":0,"points":2,"correct":[0,0,1,0]},"32104":{"type":"single","id":32104,"catId":0,"points":2,"correct":[0,0,0,1]},"32105":{"type":"single","id":32105,"catId":0,"points":2,"correct":[1,0,0,0]},"32106":{"type":"single","id":32106,"catId":0,"points":2,"correct":[1,0,0,0]},"32109":{"type":"single","id":32109,"catId":0,"points":2,"correct":[1,0,0,0]},"32111":{"type":"single","id":32111,"catId":0,"points":2,"correct":[1,0,0,0]},"32113":{"type":"single","id":32113,"catId":0,"points":2,"correct":[1,0,0,0]},"32114":{"type":"single","id":32114,"catId":0,"points":2,"correct":[0,1,0,0]},"32115":{"type":"single","id":32115,"catId":0,"points":2,"correct":[1,0,0,0]},"32116":{"type":"single","id":32116,"catId":0,"points":2,"correct":[0,0,1,0]},"32118":{"type":"single","id":32118,"catId":0,"points":2,"correct":[0,0,0,1]},"32121":{"type":"single","id":32121,"catId":0,"points":2,"correct":[0,1,0,0]},"32123":{"type":"single","id":32123,"catId":0,"points":2,"correct":[1,0,0,0]},"32124":{"type":"single","id":32124,"catId":0,"points":2,"correct":[0,0,1,0]},"32127":{"type":"single","id":32127,"catId":0,"points":2,"correct":[1,0,0,0]},"32128":{"type":"single","id":32128,"catId":0,"points":2,"correct":[0,0,1,0]},"32130":{"type":"single","id":32130,"catId":0,"points":2,"correct":[0,1,0,0]},"32132":{"type":"single","id":32132,"catId":0,"points":2,"correct":[0,0,1,0]},"32134":{"type":"single","id":32134,"catId":0,"points":2,"correct":[1,0,0,0]},"32137":{"type":"single","id":32137,"catId":0,"points":2,"correct":[0,1,0,0]},"32138":{"type":"single","id":32138,"catId":0,"points":2,"correct":[0,1,0,0]},"32139":{"type":"single","id":32139,"catId":0,"points":2,"correct":[0,0,1,0]},"32141":{"type":"single","id":32141,"catId":0,"points":2,"correct":[0,1,0,0]},"32142":{"type":"single","id":32142,"catId":0,"points":2,"correct":[0,1,0,0]},"32145":{"type":"single","id":32145,"catId":0,"points":2,"correct":[0,0,1,0]},"32148":{"type":"single","id":32148,"catId":0,"points":2,"correct":[1,0,0,0]},"32151":{"type":"single","id":32151,"catId":0,"points":2,"correct":[0,1,0,0]},"32154":{"type":"single","id":32154,"catId":0,"points":2,"correct":[0,1,0,0]},"32156":{"type":"single","id":32156,"catId":0,"points":2,"correct":[1,0,0,0]},"32157":{"type":"single","id":32157,"catId":0,"points":2,"correct":[0,0,0,1]},"32160":{"type":"single","id":32160,"catId":0,"points":2,"correct":[0,1,0,0]},"32162":{"type":"single","id":32162,"catId":0,"points":2,"correct":[0,0,0,1]}} } }); All the Best IASbaba

Daily Prelims CA Quiz

UPSC Quiz – 2024 : IASbaba’s Daily Current Affairs Quiz 12th April 2024

For Previous Daily Quiz (ARCHIVES) – CLICK HERE The Current Affairs questions are based on sources like ‘The Hindu’, ‘Indian Express’ and ‘PIB’, which are very important sources for UPSC Prelims Exam. The questions are focused on both the concepts and facts. The topics covered here are generally different from what is being covered under ‘Daily Current Affairs/Daily News Analysis (DNA) and Daily Static Quiz’ to avoid duplication. The questions would be published from Monday to Saturday before 2 PM. One should not spend more than 10 minutes on this initiative. Gear up and Make the Best Use of this initiative. Do remember that, “the difference between Ordinary and EXTRA-Ordinary is PRACTICE!!” Important Note: Don’t forget to post your marks in the comment section. Also, let us know if you enjoyed today’s test 🙂After completing the 5 questions, click on ‘View Questions’ to check your score, time taken, and solutions.To take the Test Click Here

DAILY CURRENT AFFAIRS IAS | UPSC Prelims and Mains Exam – 11th April 2024

Archives (PRELIMS & MAINS Focus)   Hepatitis Syllabus Prelims – Science Context: India is one of the countries with the highest burden of viral hepatitis — with 2.9 crore people living with Hepatitis B infection and 0.55 crore living with Hepatitis C infection, according to the Global Hepatitis Report 2024 released by the World Health Organisation (WHO) Background:- There were over 50,000 new Hepatitis B cases and 1.4 lakh new Hepatitis C cases reported in 2022. And these infections killed 1.23 lakh people in India in 2022 as per the report. About Hepatitis: Hepatitis means inflammation of the liver. The liver is a vital organ that processes nutrients, filters the blood, and fights infections. When the liver is inflamed or damaged, its function can be affected. Heavy alcohol use, toxins, some medications, and certain medical conditions can cause hepatitis. However, hepatitis is often caused by a virus. There are five main strains of the hepatitis virus, referred to as types A, B, C, D and E. While they all cause liver disease, they differ in important ways including modes of transmission, severity of the illness, geographical distribution and prevention methods. Hepatitis B (HBV) is known to cause acute infection with nausea, vomitting and yellowing of the eye and skin for several weeks. Liver failure happens in severe cases. But the challenge is the chronic, life-long liver disease that it causes, especially when children get it. The chronic infection can lead to scarring of the liver called cirrhosis and increase the risk of liver cancer. Many people with the hepatitis C (HCV) virus don’t have symptoms or know they are infected. Those who do develop symptoms two to 12 weeks after exposure report yellow skin or eyes, loss of appetite, nausea, stomach ache, fever, dark urine, light-coloured stool, joint pain and exhaustion. Both infections are transmitted from mother to child during delivery, during transfusion of blood that hasn’t been screened properly, during contact with the blood of an infected person or while sharing needles by drug users.Both viruses can also be transmitted sexually. Since hepatitis B can be prevented through vaccination, the report highlights the need to ensure coverage. Hepatitis C is curable with medicines. Source: Indian Express Previous Year Question Q.1) Which of the following statements is not correct? (UPSC CSE 2015) Hepatitis B virus is transmitted much like HIV. Hepatitis B, unlike Hepatitis C, does not have a vaccine. Globally, the number of people infected with Hepatitis B and C viruses are several times more than those infected with HIV. Some of those infected with Hepatitis B and C viruses do not show the symptoms for many years. Lavender Cultivation Syllabus Prelims & Mains – Agriculture Context:When lavender cultivation was first introduced to Bhaderwah subdivision in Jammu and Kashmir’s Doda district in 2015, most farmers were sceptical and only a few adopted it. Today, these early adopters have significantly expanded cultivation, employ 30-40 people each on a full-time basis, have their own distillation units for making lavender oil, and market their products in different parts of the country. Background: The CSIR (Council Of Scientific and Industrial Research) – Aroma Mission is a flagship project of CSIR under which Lavender cultivation is being promoted in the temperate regions of J&K.The aim of the project is to increase the income of small and marginal farmers and develop agriculture-based Startups.Besides lavender, the Aroma Mission aims to inject scientific research and development into the cultivation of aromatic plants such as rose, lemon grass, rosemary and wild marigold. About Lavender Cultivation: Lavender, which can be grown in areas that experience snowfall and have a hilly terrain, blossoms and is ready for harvesting two-and-a-half years after it is planted. The same plant can bear flowers for 18-20 years, and does not require the amount of insecticides, pesticides, and other chemicals used on conventional crops like maize and paddy. The uses of lavender includes manufacturing perfumes, soaps, agarbatti, and room fresheners.Lavender oil has a calming effect and is a stress buster. CSIR developed an elite variety of lavender suitable for cultivation in temperate regions of J&K and provided free saplings and end-to-end agro-technologies to farmers and also installed distillation units for essential oil extraction in several regions of J&K. The success of Lavender cultivation in J&K earned it the sobriquet, ‘Purple Revolution’. The variety of Lavender is highly suitable for cultivation in the rainfed temperate regions of India. The crop was officially introduced in Bhaderwah by the Council Of Scientific and Industrial Research-Indian Institute of Integrative Medicine (CSIR-IIIM), Jammu, in 2015. According to officials, more than 700 acres of farmland has been brought under lavender cultivation in the area since 2017, and another 100 acres is set to be added. CSIR plans to distribute lavender plants to farmers in parts of Kathua, Rajouri, Poonch, and the Kashmir Valley, besides Uttrakhand, Himachal Pradesh and the Northeast. Source: Indian Express CREDIT-DEPOSIT RATIO Syllabus Prelims – Economy Context: Data from the Reserve Bank of India (RBI) reveals that the credit-deposit ratio has reached its highest level in at least 20 years. Background: This surge in the credit-deposit ratio indicates that more of the banks’ deposit base is being utilized for loans, including home loans and other consumption-related loans. About CREDIT-DEPOSIT RATIO The credit-deposit ratio is a financial metric that measures the proportion of a bank’s deposit base that is used for lending. It is calculated by dividing the total loans extended by the bank by its total deposits. A higher ratio indicates that a larger portion of the bank’s deposits is being lent out as credit, while a lower ratio suggests more deposits are being held in reserve or invested in other assets. For instance, if a bank has a credit-deposit ratio of 80%, it means that 80% of its deposits are being used to provide loans. This ratio is crucial for assessing a bank’s liquidity and lending capacity. Source: Livemint GULF COOPERATION COUNCIL (GCC) Syllabus Prelims – Current Event Context: Recently, for the first time since its inception, the Gulf Cooperation Council (GCC) announced its ‘Vision for Regional Security’ in Riyadh. Background: The GCC plays a significant role in regional affairs, promoting cooperation across various domains. About GULF COOPERATION COUNCIL (GCC) The Gulf Cooperation Council (GCC), also known as the Cooperation Council for the Arab States of the Gulf, is a regional, intergovernmental, political, and economic union. GCC comprising six countries in the Arabian Peninsula: Bahrain, Kuwait, Oman, Qatar, Saudi Arabia, United Arab Emirates. The GCC was established in 1981 with a formal charter. The charter was signed in Abu Dhabi City, United Arab Emirates, on 25 May 1981 (corresponding to 21 Rajab 1401 on the Islamic calendar). The GCC aims to foster economic, scientific, and business cooperation among its member states. It serves as a platform for regional collaboration and coordination. The main headquarters of the GCC is located in Riyadh, the capital of Saudi Arabia. The Peninsula Shield Force serves as the military arm of the GCC, formed in 1984. Source: Arab News TSAT-1A SATELLITE Syllabus Prelims – Science Context: Tata Advanced Systems Limited (TASL) has achieved a remarkable milestone by successfully deploying the TSAT-1A satellite into space. Background: This groundbreaking achievement resulted from a strategic collaboration between TASL and Satellogic Inc., a leader in sub-meter resolution Earth Observation (EO) data collection. About TSAT-1A SATELLITE: TSAT-1A is the first military spy satellite initiative in India’s private sector. It is an Optical sub-metre-resolution Earth observation satellite. It is Built by Tata Advanced Systems Limited (TASL) in collaboration with Satellogic Inc. SpaceX’s Falcon 9 rocket lauched it from Kennedy Space Center, Florida. Capabilities: Military-grade imagery with sub-meter resolution. Multispectral and hyperspectral imaging for detailed analysis of land, water, and natural resources. Applications: Used by Indian defence forces for intelligence gathering. Information sharing with friendly nations to enhance preparedness and strategic decision-making. Source: Hindustan Times Internet Shutdown in India Syllabus Mains – GS 2 Context: India has topped the global list of countries imposing internet bans, with about 60% of all blackouts recorded in the world, between 2016 and 2022 having been in India. Background: Freedom in cyberspace hinges on a freely accessible, functional, and affordable internet. The extent of the freedom can be measured based on availability of mobile and broadband services, internet speed, and access to websites and social media platforms. Provisions related to Internet Shutdown: Section 5(2) of the Indian Telegraph Act, 1885, read with Temporary Suspension of Telecom Services (Public Emergency and Public Safety) Rules, 2017 allows the union or state home secretary to order the suspension of any telegraph service (including internet) in case of public emergency or public safety. Such an order must be reviewed by a committee within five days and cannot last for more than 15 days. In an urgent situation, an officer of the joint secretary level or above, authorized by the union or state home secretary, can issue the order. Section 144 of the Code of Criminal Procedure empowers a district magistrate, a sub-divisional magistrate or any other executive magistrate specially empowered by the state government to issue orders to prevent or stop any nuisance or disturbance of public tranquility. Such orders can include the suspension of internet services in a particular area for a specified period. Significance/Arguments supporting Internet Shutdown: Internet shutdowns can help to prevent the spread of hate speech and fake news that could incite violence and riots.  For example, the government announced the internet shutdown in Delhi NCR following the farmers’ protest on Republic Day to combat misinformation and maintain law and order. Internet shutdowns can help curb the organization and mobilization of protests that could disrupt public order and security.  For example, the government imposed internet shutdowns in Kashmir and other parts of the country after the abrogation of Article 370 to prevent any anti-national activities and separatist movements. Internet shutdowns can help protect national security and sovereignty from external threats and cyberattacks.  For example, the government suspended internet services in some border areas during the standoff with China to prevent any espionage or sabotage. Internet shutdowns can help control the distribution and consumption of content that could be harmful or offensive to certain groups or individuals.  For example, the government blocks internet access in some regions to prevent the circulation of objectionable images or videos. Issues/Concerns related to Internet Shutdown: Internet shutdowns violate Fundamental Rights under Article 19(1)(a) and Article 19(1)(g). The freedom of speech and expression and freedom to practice any profession over the medium of the internet enjoy constitutional protection under Article 19(1)(a) and Article 19(1)(g) – the Supreme Court in Anuradha Bhasin vs Union of India case (2020). Internet Shutdown violates the Right to Information which has been declared as a Fundamental Right under Article 19 by the Supreme Court in the Raj Narain vs State of UP (1975) case. Internet Shutdowns violate the Right to Internet which was declared a Fundamental Right under Article 21 by the Kerala High Court in the Faheema Shirin v. State of Kerala case. Internet Shutdowns are often implemented without clear legal frameworks or oversight mechanisms, leading to arbitrary and disproportionate restrictions on internet access.  The absence of accountability mechanisms exacerbates the risk of abuse of power by authorities, who may impose shutdowns without adequate justification or recourse for affected individuals. Beyond the immediate social and political implications, internet shutdowns have significant economic repercussions. Disruptions to online commerce, communication, and financial transactions disrupt business operations, hinder economic growth, and deter investment, ultimately undermining long-term development objectives. According to Top10VPN, India lost Rs 2,091 crore ($255.2 million) in the first half of 2023 because of internet shutdowns. Internet Shutdowns disrupt the fabric of society by impeding communication networks, hindering access to vital services, and isolating individuals from their communities. Social cohesion may suffer as a result, as people are unable to connect, organize, or mobilize effectively, leading to feelings of isolation and alienation. Source: Hindu Practice MCQs Daily Practice MCQs Q1.) With reference to TSAT-1A satellite, consider the following statements: TSAT-1A is the first military spy satellite initiative in India’s private sector. It is an Earth observation satellite. It is Built by Tata Advanced Systems Limited. SpaceX’s Falcon 9 rocket lauched it from Kennedy Space Center, Florida. How many of the statements given above are correct? Only one Only two Only three All four Q2.)Consider the following countries: Iran Iraq Kuwait Oman Qatar Saudi Arabia How many of the above-mentioned countries are the members of the Gulf Cooperation Council (GCC)? Only two Only three Only four Only five Q3.) With reference to the credit-deposit ratio, consider the following statements: The credit-deposit ratio is a financial metric that measures the proportion of a bank’s deposit base that is used for lending. A higher ratio indicates that a larger portion of the bank’s deposits is being lent out as credit. Which of the statements given above is/ are correct? 1 only 2 only Both 1 and 2 Neither 1 nor 2 Comment the answers to the above questions in the comment section below!! ANSWERS FOR ’  11th April  2024 – Daily Practice MCQs’ will be updated along with tomorrow’s Daily Current Affairs.st ANSWERS FOR  4th April – Daily Practice MCQs Answers- Daily Practice MCQs Q.1) –  c Q.2) – c Q.3) – b

Daily Prelims CA Quiz

UPSC Quiz – 2024 : IASbaba’s Daily Current Affairs Quiz 11th April 2024

For Previous Daily Quiz (ARCHIVES) – CLICK HERE The Current Affairs questions are based on sources like ‘The Hindu’, ‘Indian Express’ and ‘PIB’, which are very important sources for UPSC Prelims Exam. The questions are focused on both the concepts and facts. The topics covered here are generally different from what is being covered under ‘Daily Current Affairs/Daily News Analysis (DNA) and Daily Static Quiz’ to avoid duplication. The questions would be published from Monday to Saturday before 2 PM. One should not spend more than 10 minutes on this initiative. Gear up and Make the Best Use of this initiative. Do remember that, “the difference between Ordinary and EXTRA-Ordinary is PRACTICE!!” Important Note: Don’t forget to post your marks in the comment section. Also, let us know if you enjoyed today’s test 🙂After completing the 5 questions, click on ‘View Questions’ to check your score, time taken, and solutions. To take the Test Click Here

[DAY 33] 60 DAY RAPID REVISION (RaRe) SERIES for UPSC Prelims 2024 – ECONOMY, CURRENT AFFAIRS & CSAT TEST SERIES!

Archives Hello Friends The 60 Days Rapid Revision (RaRe) Series is IASbaba’s Flagship Initiative recommended by Toppers and loved by the aspirants’ community every year. It is the most comprehensive program which will help you complete the syllabus, revise and practice tests on a daily basis. The Programme on a daily basis includes Daily Prelims MCQs from Static (Monday – Saturday) Daily Static Quiz will cover all the topics of static subjects – Polity, History, Geography, Economics, Environment and Science and technology. 20 questions will be posted daily and these questions are framed from the topics mentioned in the schedule. It will ensure timely and streamlined revision of your static subjects. Daily Current Affairs MCQs (Monday – Saturday) Daily 5 Current Affairs questions, based on sources like ‘The Hindu’, ‘Indian Express’ and ‘PIB’, would be published from Monday to Saturday according to the schedule. Daily CSAT Quiz (Monday – Friday) CSAT has been an Achilles heel for many aspirants. Daily 5 CSAT Questions will be published. Note – Daily Test of 20 static questions, 10 current affairs, and 5 CSAT questions. (35 Prelims Questions) in QUIZ FORMAT will be updated on a daily basis. To Know More about 60 Days Rapid Revision (RaRe) Series – CLICK HERE   60 Day Rapid Revision (RaRe) Series Schedule – CLICK HERE  Important Note Comment your Scores in the Comment Section. This will keep you accountable, responsible and sincere in days to come. It will help us come out with the Cut-Off on a Daily Basis. Let us know if you enjoyed today’s test 🙂  You can post your comments in the given format  (1) Your Score (2) Matrix Meter (3) New Learning from the Test Time limit: 0 Test-summary 0 of 35 questions completed Questions: 1 2 3 4 5 6 7 8 9 10 11 12 13 14 15 16 17 18 19 20 21 22 23 24 25 26 27 28 29 30 31 32 33 34 35 Information The following Test is based on the syllabus of 60 Days Plan-2023 for UPSC IAS Prelims 2022. To view Solutions, follow these instructions: Click on – ‘Start Test’ button Solve Questions Click on ‘Test Summary’ button Click on ‘Finish Test’ button Now click on ‘View Questions’ button – here you will see solutions and links. You have already completed the test before. Hence you can not start it again. Test is loading... You must sign in or sign up to start the test. You have to finish following test, to start this test: Results 0 of 35 questions answered correctly Your time: Time has elapsed You have scored 0 points out of 0 points, (0) Average score     Your score     Categories Not categorized 0% Your result has been entered into leaderboard Loading Name: E-Mail: Captcha: maximum of 70 points Pos. Name Entered on Points Result Table is loading No data available 1 2 3 4 5 6 7 8 9 10 11 12 13 14 15 16 17 18 19 20 21 22 23 24 25 26 27 28 29 30 31 32 33 34 35 Answered Review Question 1 of 35 1. Question Consider the following statements regarding Treasury Bills (T-Bills): They provide long-term capital to the government. They satisfy the CRR and SLR requirements of the banking institutions that invest in them. Which of the statements given above is/are correct? a) 1 only b) 2 only c) Both 1 and 2 d) Neither 1 nor 2 Correct Solution (b) Statement 1 Statement 2 Incorrect Correct Started in 1986, they are instruments of money market used by the Central Government to fulfil its short- term liquidity requirement unto the period of 364 days.   The TBS other than providing short-term cushion to the government, it also functions as short-term investment avenues for the banks and financial institutions, besides functioning as requirements of the CRR and SLR of the banking institutions. Incorrect Solution (b) Statement 1 Statement 2 Incorrect Correct Started in 1986, they are instruments of money market used by the Central Government to fulfil its short- term liquidity requirement unto the period of 364 days.   The TBS other than providing short-term cushion to the government, it also functions as short-term investment avenues for the banks and financial institutions, besides functioning as requirements of the CRR and SLR of the banking institutions. Question 2 of 35 2. Question With reference to ‘Chit Funds’, consider the following statements: Registration and regulation of Chit funds are carried out by State Governments. Chit funds are included in the definition of Non- Banking Financial Companies by RBI. Which of the statements given above is/are correct? a) 1 only b) 2 only c) Both 1 and 2 d) Neither 1 nor 2 Correct Solution (c) Statement 1 Statement 2 Correct Correct Central Government has not framed any Rules of operation for them. Thus, Registration and Regulation of Chit funds are carried out by State Governments under the Rules framed by them.   Functionally, Chit funds are included in the definition of Non- Banking Financial Companies by RBI under the sub-head miscellaneous non-banking company (MNBC). But RBI has not laid out any separate regulatory framework for them. Incorrect Solution (c) Statement 1 Statement 2 Correct Correct Central Government has not framed any Rules of operation for them. Thus, Registration and Regulation of Chit funds are carried out by State Governments under the Rules framed by them.   Functionally, Chit funds are included in the definition of Non- Banking Financial Companies by RBI under the sub-head miscellaneous non-banking company (MNBC). But RBI has not laid out any separate regulatory framework for them. Question 3 of 35 3. Question With reference to Venture Capital Fund (VCF), consider the following statements: Venture Capital Funds ensure that the money of the investors is used to fund projects which have a potential to grow. They are regulated by the guidelines issued by the Reserve Bank of India (RBI). Which of the statements given above is/are correct? a) 1 only b) 2 only c) Both 1 and 2 d) Neither 1 nor 2 Correct Solution (a) Statement 1 Statement 2 Correct Incorrect Venture Capital Funds ensure that the money of the investors is used to fund projects which have a potential to grow and the money provided in the process is known as Venture Capital. They are institutions that are dedicated to funding new ventures and are regulated by the guidelines issued by the Securities and Exchange Board of India (SEBI).   Incorrect Solution (a) Statement 1 Statement 2 Correct Incorrect Venture Capital Funds ensure that the money of the investors is used to fund projects which have a potential to grow and the money provided in the process is known as Venture Capital. They are institutions that are dedicated to funding new ventures and are regulated by the guidelines issued by the Securities and Exchange Board of India (SEBI).   Question 4 of 35 4. Question With reference to Alternative Investment Funds, consider the following statements: It refers to any privately pooled investment fund from domestic sources only. Currently they do not come under the jurisdiction of any regulatory agency in India. Choose the correct code: a) 1 only b) 2 only c) Both 1 and 2 d) Neither 1 nor 2 Correct Solution (d) Statement 1 Statement 2 Incorrect Incorrect It refers to any privately pooled investment fund, (whether from Indian or foreign sources), in the form of a trust or a company or a body corporate or a Limited Liability Partnership (LLP) which are not presently covered by any Regulation of SEBI governing fund management (like, Regulations governing Mutual Fund or Collective Investment Scheme) nor coming under the direct regulation of any other sectoral regulators in India-IRDA, PFRDA, RBI. In India, Alternative Investment Funds (AIFs) are defined in Regulation 2(1)(b) of Securities and Exchange Board of India (Alternative Investment Funds) Regulations, 2012. Incorrect Solution (d) Statement 1 Statement 2 Incorrect Incorrect It refers to any privately pooled investment fund, (whether from Indian or foreign sources), in the form of a trust or a company or a body corporate or a Limited Liability Partnership (LLP) which are not presently covered by any Regulation of SEBI governing fund management (like, Regulations governing Mutual Fund or Collective Investment Scheme) nor coming under the direct regulation of any other sectoral regulators in India-IRDA, PFRDA, RBI. In India, Alternative Investment Funds (AIFs) are defined in Regulation 2(1)(b) of Securities and Exchange Board of India (Alternative Investment Funds) Regulations, 2012. Question 5 of 35 5. Question Which of the following financial instruments can be classified as securities? Bonds Shares Fixed Deposits Choose the correct code: a) Only one b) Only two c) All three d) None Correct Solution (b) Statement 1 Statement 2 Statement 3 Correct Correct Incorrect Financial instruments which are tradable or that can be bought and sold are generally referred to as securities. For e.g., shares, bonds etc. A government bond is a debt instrument issued by the Central and State Governments of India. Issuance of such bonds occur when the issuing body (Central or State governments) faces a liquidity crisis and requires funds for the purpose of infrastructure development. The key aspect is that it should be transferable. For instance, In India, the word “securities” is defined in clause (h) of section 2 of the Securities Contracts (Regulation) Act, 1956 (SCRA). The term securities include the following in India: shares, scrips, stocks, bonds, debentures, debenture stock or other marketable securities of a like nature in or of any incorporated company or other body corporates derivatives Fixed deposits are low-risk investments as they aren’t dependent on market fluctuations. Therefore, these are not included in securities. Incorrect Solution (b) Statement 1 Statement 2 Statement 3 Correct Correct Incorrect Financial instruments which are tradable or that can be bought and sold are generally referred to as securities. For e.g., shares, bonds etc. A government bond is a debt instrument issued by the Central and State Governments of India. Issuance of such bonds occur when the issuing body (Central or State governments) faces a liquidity crisis and requires funds for the purpose of infrastructure development. The key aspect is that it should be transferable. For instance, In India, the word “securities” is defined in clause (h) of section 2 of the Securities Contracts (Regulation) Act, 1956 (SCRA). The term securities include the following in India: shares, scrips, stocks, bonds, debentures, debenture stock or other marketable securities of a like nature in or of any incorporated company or other body corporates derivatives Fixed deposits are low-risk investments as they aren’t dependent on market fluctuations. Therefore, these are not included in securities. Question 6 of 35 6. Question With reference to ‘Capital Gains Tax’, which of the following statements is/are correct? It is levied on long term gains only. Capital gains tax in India need not be paid in case the individual inherits the property and there is no sale. Which of the statements given above is/are correct? a) 1 only b) 2 only c) Both 1 and 2 d) Neither 1 nor 2 Correct Solution (b) Statement 1 Statement 2 Incorrect Correct Capital gain can be defined as any profit that is received through the sale of a capital asset. The profit that is received falls under the income category. Therefore, a tax needs to be paid on the income that is received. The tax that is paid is called capital gains tax and it can either be long term or short term. Under the Income Tax Act, capital gains tax in India need not be paid in case the individual inherits the property and there is no sale. However, if the person who has inherited the property decides to sell it, tax will have to be paid on the income that has been generated from the sale. Incorrect Solution (b) Statement 1 Statement 2 Incorrect Correct Capital gain can be defined as any profit that is received through the sale of a capital asset. The profit that is received falls under the income category. Therefore, a tax needs to be paid on the income that is received. The tax that is paid is called capital gains tax and it can either be long term or short term. Under the Income Tax Act, capital gains tax in India need not be paid in case the individual inherits the property and there is no sale. However, if the person who has inherited the property decides to sell it, tax will have to be paid on the income that has been generated from the sale. Question 7 of 35 7. Question With reference to debentures, consider the following statements: Debenture is a movable property. The debenture holders have the right to vote in meetings of the company. Select the correct answer using the code given below: a) 1 only b) 2 only c) Both 1 and 2 d) Neither 1 nor 2 Correct Solution (a) Statement 1 Statement 2 Correct Incorrect Denture is a movable property. It is in the form of a certificate of indebtedness of the company and issued by the company itself. It generally creates a charge on the undertaking or undertakings of the company. As the debenture holders are not the owner of the company so they are not entitled with the administration and management of the company. The debenture holders cannot claim the privilege to vote in any meeting of the company. Incorrect Solution (a) Statement 1 Statement 2 Correct Incorrect Denture is a movable property. It is in the form of a certificate of indebtedness of the company and issued by the company itself. It generally creates a charge on the undertaking or undertakings of the company. As the debenture holders are not the owner of the company so they are not entitled with the administration and management of the company. The debenture holders cannot claim the privilege to vote in any meeting of the company. Question 8 of 35 8. Question With reference to capital markets, consider the following statements: Capital market deals in financial instruments and commodities that are long-term securities. Secondary Market exclusively deals with the issue of new securities. Which of the statements given above is/are correct? a) 1 only b) 2 only c) Both 1 and 2 d) Neither 1 nor 2 Correct Solution (a) Statement 1 Statement 2 Correct Incorrect Capital market deals in financial instruments and commodities that are long-term securities. They have a maturity of at least more than one year.   The most important type of capital market is the primary market. It is what we call the new issue market. It exclusively deals with the issue of new securities, i.e., securities that are issued to investors for the very first time. Incorrect Solution (a) Statement 1 Statement 2 Correct Incorrect Capital market deals in financial instruments and commodities that are long-term securities. They have a maturity of at least more than one year.   The most important type of capital market is the primary market. It is what we call the new issue market. It exclusively deals with the issue of new securities, i.e., securities that are issued to investors for the very first time. Question 9 of 35 9. Question Qualified Foreign Investors (QFIs) are allowed to invest in which of the following segments of the capital market? Corporate bonds Commercial Papers Mutual funds Select the correct answer using the code given below a) 1 and 2 only b) 3 only c) 2 and 3 only d) 1, 2 and 3 Correct Solution (d) Statement 1 Statement 2 Statement 3 Correct Correct Correct The Qualified Foreign Investor (QFI) is sub-category of Foreign Portfolio Investor and refers to any foreign individuals, groups or associations, or resident, however, restricted to those from a country that is a member of Financial Action Task Force (FATF). QFIs are allowed to make investments in the Corporate bonds listed/to be listed on recognized stock exchanges G-Securities, T-Bills and Commercial Papers by opening a demat account in of the SEBI approved Qualified Depository Participant (QDP). QFIs are allowed to make investments in the Equity and Debt schemes of Indian mutual funds. Incorrect Solution (d) Statement 1 Statement 2 Statement 3 Correct Correct Correct The Qualified Foreign Investor (QFI) is sub-category of Foreign Portfolio Investor and refers to any foreign individuals, groups or associations, or resident, however, restricted to those from a country that is a member of Financial Action Task Force (FATF). QFIs are allowed to make investments in the Corporate bonds listed/to be listed on recognized stock exchanges G-Securities, T-Bills and Commercial Papers by opening a demat account in of the SEBI approved Qualified Depository Participant (QDP). QFIs are allowed to make investments in the Equity and Debt schemes of Indian mutual funds. Question 10 of 35 10. Question Which of the followings entities is/are allowed to participate in “Call Money Market”? Land Development Banks Regional Rural Banks Payment Banks Select the correct answer from the codes given below: a) 3 only b) 1 and 2 only c) 1, 2 and 3 d) None Correct Solution (a) Statement 1 Statement 2 Statement 3 Incorrect Incorrect Correct Participants in the call money market are banks and related entities specified by the RBI. Scheduled commercial banks (excluding RRBs), co-operative banks (other than Land Development Banks) and Primary  Dealers (PDs), are permitted to participate in call/notice money market both as borrowers and lenders. Financial markets broad basing is at the centre stage for the Reserve Bank of India with the inclusion of Regional Rural Banks into the money market. As per the new regulations, Payment Banks are also allowed to participate in CMM as both lenders and borrowers. Incorrect Solution (a) Statement 1 Statement 2 Statement 3 Incorrect Incorrect Correct Participants in the call money market are banks and related entities specified by the RBI. Scheduled commercial banks (excluding RRBs), co-operative banks (other than Land Development Banks) and Primary  Dealers (PDs), are permitted to participate in call/notice money market both as borrowers and lenders. Financial markets broad basing is at the centre stage for the Reserve Bank of India with the inclusion of Regional Rural Banks into the money market. As per the new regulations, Payment Banks are also allowed to participate in CMM as both lenders and borrowers. Question 11 of 35 11. Question Consider the following statements about the Foreign Portfolio Investment (FPI): It consists of securities and other financial assets held by the investors in another country. It provides the investor with direct ownership of a company’s assets. The American Depositary Receipt’s and the Global Depository Receipts form part of Foreign Portfolio Investment . It is a part of a country’s Capital Account. How many of the above statements are correct? a) Only one b) Only two c) Only three d) All four Correct Solution (c) Statement 1 Statement 2 Statement 3 Statement 4 Correct Incorrect Correct Correct The Foreign Portfolio Investment (FPI) consists of securities and other financial assets held by the investors in another country. It does not provide the investor with direct ownership of a company’s assets and is relatively liquid, depending on the volatility of the market. The FPI holdings can include stocks, ADRs (the American Depositary Receipts), GDRs (the Global Depositary Receipts), bonds, mutual funds and exchange traded funds. On a macro-level, the FPI is a part of a country’s Capital Account and is shown on its Balance of Payments (BOP). The BOP calculates the amount of money flowing from one country to other countries over a financial year. Note: Foreign Portfolio Investment (FPI): The Foreign Portfolio Investment (FPI) consists of securities and other financial assets held by the investors in another country. It does not provide the investor with direct ownership of a company’s assets and is relatively liquid, depending on the volatility of the market. Along with the Foreign Direct Investment (FDI), the FPI is one of the common ways to invest in an overseas economy. The FDI and the FPI are both important sources of funding for most economies. On a macro-level, the FPI is a part of a country’s Capital Account and is shown on its Balance of Payments (BOP). The BOP calculates the amount of money flowing from one country to other countries over a financial year. The FPI holdings can include stocks, ADRs (the American Depositary Receipts), GDRs (the Global Depositary Receipts), bonds, mutual funds and exchange traded funds. Incorrect Solution (c) Statement 1 Statement 2 Statement 3 Statement 4 Correct Incorrect Correct Correct The Foreign Portfolio Investment (FPI) consists of securities and other financial assets held by the investors in another country. It does not provide the investor with direct ownership of a company’s assets and is relatively liquid, depending on the volatility of the market. The FPI holdings can include stocks, ADRs (the American Depositary Receipts), GDRs (the Global Depositary Receipts), bonds, mutual funds and exchange traded funds. On a macro-level, the FPI is a part of a country’s Capital Account and is shown on its Balance of Payments (BOP). The BOP calculates the amount of money flowing from one country to other countries over a financial year. Note: Foreign Portfolio Investment (FPI): The Foreign Portfolio Investment (FPI) consists of securities and other financial assets held by the investors in another country. It does not provide the investor with direct ownership of a company’s assets and is relatively liquid, depending on the volatility of the market. Along with the Foreign Direct Investment (FDI), the FPI is one of the common ways to invest in an overseas economy. The FDI and the FPI are both important sources of funding for most economies. On a macro-level, the FPI is a part of a country’s Capital Account and is shown on its Balance of Payments (BOP). The BOP calculates the amount of money flowing from one country to other countries over a financial year. The FPI holdings can include stocks, ADRs (the American Depositary Receipts), GDRs (the Global Depositary Receipts), bonds, mutual funds and exchange traded funds. Question 12 of 35 12. Question Foreign exchange reserves are important for the Economies around the world for which of the following reasons? To keep the value of their currencies at a fixed rate. To maintain liquidity in case of an economic crisis. To provide confidence to the investors. To fund sectors like infrastructure. To meet external obligations. Select the correct answer using the code given below: a) 1, 2, 3 and 5 only b) 1, 2 and 4 only c) 1, 2, 3 and 4 only d) 1, 2, 3, 4 and 5 Correct Solution (d) There are seven ways the central banks use the foreign exchange reserves. First, the countries use their foreign exchange reserves to keep the value of their currencies at a fixed rate. A good example is China, which pegs the value of its currency, the yuan, to the dollar. When China stockpiles dollars, it raises the dollar value compared to that of the yuan. That makes Chinese exports cheaper than the American-made goods, increasing the sales. Second, those with a floating exchange rate system use reserves to keep the value of their currency lower than the dollar. They do this for the same reasons as those with fixed-rate systems. Even though Japan’s currency, the yen, is a floating system, the Central Bank of Japan buys U.S. Treasurys to keep its value lower than the dollar. Like China, this keeps Japan’s exports relatively cheaper, boosting trade and economic growth. Such currency trading takes place in the foreign exchange market. A third and critical function is to maintain liquidity in case of an economic crisis. For example, a flood or volcano might temporarily suspend the local exporters’ ability to produce goods. That cuts off their supply of foreign currency to pay for imports. In that case, the central bank can exchange its foreign currency for their local currency, allowing them to pay for and receive the imports. Similarly, the foreign investors will get spooked if a country has a war, military coup, or other blow to confidence. They withdraw their deposits from the country’s banks, creating a severe shortage in foreign currency. This pushes down the value of the local currency, since fewer people want it. That makes imports more expensive, creating inflation. The central bank supplies foreign currency to keep the markets steady. It also buys the local currency to support its value and prevent inflation. This reassures the foreign investors, who return to the economy. A fourth reason is to provide confidence. The central bank assures the foreign investors that it is ready to take action to protect their investments. It will also prevent a sudden flight to safety and loss of capital for the country. In that way, a strong position in the foreign currency reserves can prevent economic crises caused when an event triggers a flight to safety. Fifth, reserves are always needed to make sure a country will meet its external obligations. These include international payment obligations, including sovereign and commercial debts. They also include the financing of imports and the ability to absorb any unexpected capital movements. Sixth, some countries use their reserves to fund sectors, such as infrastructure. China, for instance, has used part of its forex reserves for recapitalizing some of its state-owned banks. Seventh, most central banks want to boost returns without compromising safety. They know the best way to do that is to diversify their portfolios. They will often hold gold and other safe, interest-bearing investments.   Incorrect Solution (d) There are seven ways the central banks use the foreign exchange reserves. First, the countries use their foreign exchange reserves to keep the value of their currencies at a fixed rate. A good example is China, which pegs the value of its currency, the yuan, to the dollar. When China stockpiles dollars, it raises the dollar value compared to that of the yuan. That makes Chinese exports cheaper than the American-made goods, increasing the sales. Second, those with a floating exchange rate system use reserves to keep the value of their currency lower than the dollar. They do this for the same reasons as those with fixed-rate systems. Even though Japan’s currency, the yen, is a floating system, the Central Bank of Japan buys U.S. Treasurys to keep its value lower than the dollar. Like China, this keeps Japan’s exports relatively cheaper, boosting trade and economic growth. Such currency trading takes place in the foreign exchange market. A third and critical function is to maintain liquidity in case of an economic crisis. For example, a flood or volcano might temporarily suspend the local exporters’ ability to produce goods. That cuts off their supply of foreign currency to pay for imports. In that case, the central bank can exchange its foreign currency for their local currency, allowing them to pay for and receive the imports. Similarly, the foreign investors will get spooked if a country has a war, military coup, or other blow to confidence. They withdraw their deposits from the country’s banks, creating a severe shortage in foreign currency. This pushes down the value of the local currency, since fewer people want it. That makes imports more expensive, creating inflation. The central bank supplies foreign currency to keep the markets steady. It also buys the local currency to support its value and prevent inflation. This reassures the foreign investors, who return to the economy. A fourth reason is to provide confidence. The central bank assures the foreign investors that it is ready to take action to protect their investments. It will also prevent a sudden flight to safety and loss of capital for the country. In that way, a strong position in the foreign currency reserves can prevent economic crises caused when an event triggers a flight to safety. Fifth, reserves are always needed to make sure a country will meet its external obligations. These include international payment obligations, including sovereign and commercial debts. They also include the financing of imports and the ability to absorb any unexpected capital movements. Sixth, some countries use their reserves to fund sectors, such as infrastructure. China, for instance, has used part of its forex reserves for recapitalizing some of its state-owned banks. Seventh, most central banks want to boost returns without compromising safety. They know the best way to do that is to diversify their portfolios. They will often hold gold and other safe, interest-bearing investments.   Question 13 of 35 13. Question Which of the following correctly describes an Escrow Account? a) A bank account through which NRIs can operate their finances in India. b) An overseas bank account used by MNCs for routing the funds through tax havens for the purpose of tax evasion. c) A third party bank account for safeguarding the seller against its buyer from the payment risk. d) A bank account which earns regular interest in return for a fixed monthly deposits. Correct Solution (c) An escrow account in simple terms is a third-party account. It is a separate bank account to hold money which belongs to others and where the money parked will be released only under the fulfilment of certain conditions of a contract. An escrow account is an arrangement for safeguarding the seller against its buyer from the payment risk for the goods or services sold by the former to the latter. This is done by removing the control over cash flows from the hands of the buyer to an independent agent. The independent agent, i.e, the holder of the escrow account would ensure that the appropriation of cash flows is as per the agreed terms and conditions between the transacting parties. Escrow account has become the standard in various transactions and business deals. In India escrow account is widely used in public private partnership projects in infrastructure. RBI has also permitted Banks (Authorized Dealer Category I) to open escrow accounts on behalf of Non-Resident corporates for acquisition / transfer of shares / convertible shares of an Indian company. Incorrect Solution (c) An escrow account in simple terms is a third-party account. It is a separate bank account to hold money which belongs to others and where the money parked will be released only under the fulfilment of certain conditions of a contract. An escrow account is an arrangement for safeguarding the seller against its buyer from the payment risk for the goods or services sold by the former to the latter. This is done by removing the control over cash flows from the hands of the buyer to an independent agent. The independent agent, i.e, the holder of the escrow account would ensure that the appropriation of cash flows is as per the agreed terms and conditions between the transacting parties. Escrow account has become the standard in various transactions and business deals. In India escrow account is widely used in public private partnership projects in infrastructure. RBI has also permitted Banks (Authorized Dealer Category I) to open escrow accounts on behalf of Non-Resident corporates for acquisition / transfer of shares / convertible shares of an Indian company. Question 14 of 35 14. Question With reference to Primary Deficit, consider the following statements: The primary deficit is the sum of Fiscal deficit and interest payments. It includes the burden of the past debt. Which of the statements given above is/are correct? a) 1 only b) 2 only c) Both 1 and 2 d) Neither 1 nor 2 Correct Solution (d) Statement 1 Statement 2 Incorrect Incorrect Gross Primary deficit is defined as gross fiscal deficit minus net interest payments. Net primary deficit, is gross primary deficit minus net domestic lending. It excludes the burden of the past debt and shows the net increase in the government’s indebtedness due to the current year’s fiscal operations. A reduction in primary deficit is reflective of government’s efforts at bridging the fiscal gap during a financial year. Incorrect Solution (d) Statement 1 Statement 2 Incorrect Incorrect Gross Primary deficit is defined as gross fiscal deficit minus net interest payments. Net primary deficit, is gross primary deficit minus net domestic lending. It excludes the burden of the past debt and shows the net increase in the government’s indebtedness due to the current year’s fiscal operations. A reduction in primary deficit is reflective of government’s efforts at bridging the fiscal gap during a financial year. Question 15 of 35 15. Question Which of the following is/are the factors determining the exchange rate of a country? Interest rates Money supply Financial stability Choose the correct code: a) Only one b) Only two c) All three d) None Correct Solution (c) Three Factors Affecting the Exchange Rates- Interest rates: The interest rate paid by a country’s central bank is a big factor. The higher interest rate makes that currency more valuable. The investors will exchange their currency for the higher-paying one. They then save it in that country’s bank to receive the higher interest rate. Money supply: The money supply that is created by the country’s central bank. If the government prints too much currency, then there is too much of it chasing too few goods. The currency holders will bid up the prices of goods and services. That creates inflation. If way too much money is printed, it causes hyperinflation. Some cash holders will invest overseas where there is no inflation, but they will find that there is not as much demand for their currency, since there is so much of it. That is why inflation can push the value of a currency down. Financial stability: Country’s economic growth and financial stability impact its currency exchange rates. If the country has a strong, growing economy, then the investors will buy its goods and services. They will need more of its currency to do so. If the financial stability looks bad, they will be less willing to invest in that country. They want to be sure they will get paid back if they hold the government bonds in that currency. Because of these factors, the demand for a country’s currency depends on what is happening in that country. Incorrect Solution (c) Three Factors Affecting the Exchange Rates- Interest rates: The interest rate paid by a country’s central bank is a big factor. The higher interest rate makes that currency more valuable. The investors will exchange their currency for the higher-paying one. They then save it in that country’s bank to receive the higher interest rate. Money supply: The money supply that is created by the country’s central bank. If the government prints too much currency, then there is too much of it chasing too few goods. The currency holders will bid up the prices of goods and services. That creates inflation. If way too much money is printed, it causes hyperinflation. Some cash holders will invest overseas where there is no inflation, but they will find that there is not as much demand for their currency, since there is so much of it. That is why inflation can push the value of a currency down. Financial stability: Country’s economic growth and financial stability impact its currency exchange rates. If the country has a strong, growing economy, then the investors will buy its goods and services. They will need more of its currency to do so. If the financial stability looks bad, they will be less willing to invest in that country. They want to be sure they will get paid back if they hold the government bonds in that currency. Because of these factors, the demand for a country’s currency depends on what is happening in that country. Question 16 of 35 16. Question Which of the following are the features of a Contestable Market? There are no barriers to entry or exit barriers. There are sunk costs involved in such markets. Both the incumbent firms and new entrants have access to the same level of technology. How many of the above statements are correct? a) Only one b) Only two c) All three d) None Correct Solution (b) Statement 1 Statement 2 Statement 3 Correct Incorrect Correct There are no barriers to entry or exit barriers is one of the features of a Contestable Market. There are no sunk costs is one of the features of a Contestable Market. Both the incumbent firms and new entrants have access to the same level of technology is one of the features of a Contestable Market.   Note: Contestable Market Theory: ‘The Contestable Market Theory‘ is an economic concept stating that the companies with a few rivals behave in a competitive manner, when the market they operate in has weak barriers to entry. ‘Contestable‘ in economics means that a company can be challenged or contested by the rival companies looking to enter the industry or the market. In other words, a Contestable Market is a market whereby the companies can enter and leave freely with low sunk costs. Sunk costs are major irrecoverable costs to enter an industry, such as the purchase of a manufacturing plant or equipment. The Contestable Market Theory assumes that even in a monopoly or oligopoly (It is a market form wherein a market or an industry is dominated by a small group of large sellers), the dominant companies will act competitively when there is a lack of barriers for the competitors. Dominant players in an industry will do everything to reduce the contestability of their industry by preventing new entrants from driving them out of business. The features of a Contestable Market include: There are no barriers to entry or exit barriers. There are no sunk costs, i.e.; the costs that have already been incurred and cannot be recovered. Both the incumbent firms and new entrants have access to the same level of technology. The continuous risk of new entrants emerging and stealing market share leads the incumbents to focus more on maximizing sales rather than profits. They realize that if they are too profitable, an entrant could easily come and undercut their business. Incorrect Solution (b) Statement 1 Statement 2 Statement 3 Correct Incorrect Correct There are no barriers to entry or exit barriers is one of the features of a Contestable Market. There are no sunk costs is one of the features of a Contestable Market. Both the incumbent firms and new entrants have access to the same level of technology is one of the features of a Contestable Market.   Note: Contestable Market Theory: ‘The Contestable Market Theory‘ is an economic concept stating that the companies with a few rivals behave in a competitive manner, when the market they operate in has weak barriers to entry. ‘Contestable‘ in economics means that a company can be challenged or contested by the rival companies looking to enter the industry or the market. In other words, a Contestable Market is a market whereby the companies can enter and leave freely with low sunk costs. Sunk costs are major irrecoverable costs to enter an industry, such as the purchase of a manufacturing plant or equipment. The Contestable Market Theory assumes that even in a monopoly or oligopoly (It is a market form wherein a market or an industry is dominated by a small group of large sellers), the dominant companies will act competitively when there is a lack of barriers for the competitors. Dominant players in an industry will do everything to reduce the contestability of their industry by preventing new entrants from driving them out of business. The features of a Contestable Market include: There are no barriers to entry or exit barriers. There are no sunk costs, i.e.; the costs that have already been incurred and cannot be recovered. Both the incumbent firms and new entrants have access to the same level of technology. The continuous risk of new entrants emerging and stealing market share leads the incumbents to focus more on maximizing sales rather than profits. They realize that if they are too profitable, an entrant could easily come and undercut their business. Question 17 of 35 17. Question Consider the following statements: The Securities and Exchange Board of India (SEBI) was set up in 1988 and was given statutory recognition in 1992. Capital market refers to the market for short to medium-term funds for investment purposes. Bombay Stock Exchange is the largest stock market in the country. How many of the above statements are correct? a) Only one b) Only two c) All three d) None Correct Solution (a) Statement 1 Statement 2 Statement 3 Correct Incorrect Incorrect The Securities and Exchange Board of India (SEBI) was set up in 1988 and was given statutory recognition in 1992. Capital market refers to the market for long-term funds for investment purposes. The capital market is the source of funds for the corporates and the governments, and provides opportunities to the savers to park their long-term savings. The capital market comprises two segments: the primary and the secondary markets. The setting up to the NSE (The National Stock Exchange of India Ltd) is a landmark in the Indian capital markets. At present, the NSE is the largest stock market in the country. Trading on the NSE can be done throughout the country through the network of satellite terminals. The NSE has introduced inter-regional clearing facilities. Incorrect Solution (a) Statement 1 Statement 2 Statement 3 Correct Incorrect Incorrect The Securities and Exchange Board of India (SEBI) was set up in 1988 and was given statutory recognition in 1992. Capital market refers to the market for long-term funds for investment purposes. The capital market is the source of funds for the corporates and the governments, and provides opportunities to the savers to park their long-term savings. The capital market comprises two segments: the primary and the secondary markets. The setting up to the NSE (The National Stock Exchange of India Ltd) is a landmark in the Indian capital markets. At present, the NSE is the largest stock market in the country. Trading on the NSE can be done throughout the country through the network of satellite terminals. The NSE has introduced inter-regional clearing facilities. Question 18 of 35 18. Question With reference to the Interim Budget, consider the following statements: The Interim Budget is a budget which is presented in an election year and is applicable for the entire financial year. Usually, the Interim Budget does not contain the Finance Bill, but includes the Vote-on-Account. Which of the statements given above is/are correct? a) 1 only b) 2 only c) Both 1 and 2 d) Neither 1 nor 2 Correct Solution (b) Statement 1 Statement 2 Incorrect Correct The Interim Budget is essentially a budget which is announced in an election year by the incumbent government. The Interim Budget would be applicable until the new government takes over after the General Elections. Hence, unlike the Union Budget, the Interim Budget would be applicable for a part of the financial year. The Interim Budget does not contain the Finance Bill, but includes the Vote-on-Account. The government needs approval from the Parliament to withdraw money from the Consolidated Fund of India to meet its day-to-day expenses. Hence, the incumbent government presents the Vote-on-Account in order to seek approval from the Parliament for withdrawing money to meet its expenditure needs until the new government takes over. Incorrect Solution (b) Statement 1 Statement 2 Incorrect Correct The Interim Budget is essentially a budget which is announced in an election year by the incumbent government. The Interim Budget would be applicable until the new government takes over after the General Elections. Hence, unlike the Union Budget, the Interim Budget would be applicable for a part of the financial year. The Interim Budget does not contain the Finance Bill, but includes the Vote-on-Account. The government needs approval from the Parliament to withdraw money from the Consolidated Fund of India to meet its day-to-day expenses. Hence, the incumbent government presents the Vote-on-Account in order to seek approval from the Parliament for withdrawing money to meet its expenditure needs until the new government takes over. Question 19 of 35 19. Question Which among the following is the most important reason(s) for the government to prefer the Exchange Traded Fund (ETF) route for undertaking disinvestment in the PSUs? Ability to undertake disinvestment in multiple PSUs at a single time. Enables the government to reduce its ownership in the PSUs to below 51% and yet retain management control. Enables the government to sell its entire stake to a single private sector entity. How many of the statements given above is/are correct? a) Only one b) Only two c) All three d) None Correct Solution (a) Exchange Traded Fund (ETF): The Exchange Traded Fund (ETF) is a fund that is created by pooling together the assets and then dividing this cumulated asset into individual units that are traded on the stock exchange. The value of the ETF comes from the value of the underlying assets (shares of stock, bonds, foreign currency, etc.). These ETFs, listed in the stock exchanges, are similar to shares and can be traded like ordinary shares. In nature, the ETFs are Index Funds, because they comprise shares of different companies. The share-based ETFs, issued by the government, include CPSE ETF and Bharat-22 ETF. These share-based ETFs enable the government to undertake the disinvestment in multiple PSUs at a single time and hence, reduces the time for undertaking disinvestment. This is the main reason for the higher preference of the government towards the ETF route for undertaking disinvestment. Incorrect Solution (a) Exchange Traded Fund (ETF): The Exchange Traded Fund (ETF) is a fund that is created by pooling together the assets and then dividing this cumulated asset into individual units that are traded on the stock exchange. The value of the ETF comes from the value of the underlying assets (shares of stock, bonds, foreign currency, etc.). These ETFs, listed in the stock exchanges, are similar to shares and can be traded like ordinary shares. In nature, the ETFs are Index Funds, because they comprise shares of different companies. The share-based ETFs, issued by the government, include CPSE ETF and Bharat-22 ETF. These share-based ETFs enable the government to undertake the disinvestment in multiple PSUs at a single time and hence, reduces the time for undertaking disinvestment. This is the main reason for the higher preference of the government towards the ETF route for undertaking disinvestment. Question 20 of 35 20. Question The Panda Bonds, sometimes seen in the news, refer to which of the following? a) Yuan-denominated bonds issued in Chinese mainland by an overseas entity. b) Yuan-denominated bonds issued in Chinese mainland by the Chinese companies. c) Rupee denominated bonds issued in Chinese mainland. d) Yuan denominated bonds issued in India. Correct Solution (a) Panda Bonds: The Panda Bonds are the Yuan-denominated bonds issued in the Chinese mainland market by an overseas entity. They are usually issued by the foreign companies and the Chinese companies operating overseas to raise capital. The International Finance Corporation and the Asian Development Bank have issued the Panda Bonds in 2005. Besides, countries like the Philippines and most recently, Pakistan have issued the Panda Bonds. Incorrect Solution (a) Panda Bonds: The Panda Bonds are the Yuan-denominated bonds issued in the Chinese mainland market by an overseas entity. They are usually issued by the foreign companies and the Chinese companies operating overseas to raise capital. The International Finance Corporation and the Asian Development Bank have issued the Panda Bonds in 2005. Besides, countries like the Philippines and most recently, Pakistan have issued the Panda Bonds. Question 21 of 35 21. Question Consider the following statements about ‘Iyothee Thass Pandithar’: He was a prominent anti-caste activist and a practitioner of Siddha medicine. He established the Advaidananda Sabha and launched a magazine called Dravida Pandian. He organized the Todas and other tribes of the Nilgiri Hills into a formidable force for the freedom movement. He established the Sakya Buddhist Society in Bengal. How many of the above statements are correct? a) Only one b) Only two c) Only three d) All four Correct Solution (c) Iyothee Thass Pandithar was a prominent anti-caste activist and a practitioner of Siddha medicine. Hence statement 1 is correct. He was born on 20 May 1845 in Madras presidency. He established the Advaidananda Sabha and launched a magazine called Dravida Pandian. Hence statement 2 is correct. He established the “Dravida Mahajana Sabha ” in 1891 along with Rettamalai Srinivasan. He organized the Todas and other tribes of the Nilgiri Hills into a formidable force for the freedom movement. Hence statement 3 is correct. He established the Sakya Buddhist Society in Madras with branches all over South India. Hence statement 4 is incorrect. The Sakya Buddhist Society was also known as the Indian Buddhist Association and was established in the year 1898. To manage and coordinate the functioning of the society, he began a weekly magazine, Tamizhan in 1907. Incorrect Solution (c) Iyothee Thass Pandithar was a prominent anti-caste activist and a practitioner of Siddha medicine. Hence statement 1 is correct. He was born on 20 May 1845 in Madras presidency. He established the Advaidananda Sabha and launched a magazine called Dravida Pandian. Hence statement 2 is correct. He established the “Dravida Mahajana Sabha ” in 1891 along with Rettamalai Srinivasan. He organized the Todas and other tribes of the Nilgiri Hills into a formidable force for the freedom movement. Hence statement 3 is correct. He established the Sakya Buddhist Society in Madras with branches all over South India. Hence statement 4 is incorrect. The Sakya Buddhist Society was also known as the Indian Buddhist Association and was established in the year 1898. To manage and coordinate the functioning of the society, he began a weekly magazine, Tamizhan in 1907. Question 22 of 35 22. Question Consider the following statements about Mumps: It is a contagious disease caused by a paramyxovirus. It is an acute disease that infects children and young adults. There is no specific treatment for mumps. How many of the above statements are correct? a) Only one b) Only two c) All three d) None Correct Solution (c) Mumps is a contagious disease caused by a paramyxovirus. Hence statement 1 is correct. It can cause the parotid glands, the ones on each side of the face to swell and they may also be tender or painful. It is an acute disease that infects children and young adults. Hence statement 2 is correct. Humans are the only known host for the mumps virus, which is spread via direct contact or by airborne droplets from the upper respiratory tract of infected individuals. There is no specific treatment for mumps. Hence statement 3 is correct. The various symptoms may be relieved with medicines. The best way to protect kids is to make sure they’re immunized against mumps. Incorrect Solution (c) Mumps is a contagious disease caused by a paramyxovirus. Hence statement 1 is correct. It can cause the parotid glands, the ones on each side of the face to swell and they may also be tender or painful. It is an acute disease that infects children and young adults. Hence statement 2 is correct. Humans are the only known host for the mumps virus, which is spread via direct contact or by airborne droplets from the upper respiratory tract of infected individuals. There is no specific treatment for mumps. Hence statement 3 is correct. The various symptoms may be relieved with medicines. The best way to protect kids is to make sure they’re immunized against mumps. Question 23 of 35 23. Question Consider the following statements about different types of ‘Otoliths’ that aid fish in balance and hearing: Sagitta is involved in the detection of sound and the process of hearing, or converting sound waves into electrical signals. Asteriscus is involved in the detection of sound and the process of hearing. Lapillus is involved in the detection of gravitational force and sound. How many of the above statements are correct? a) Only one b) Only two c) All three d) None Correct Solution (c) The otolith is a stony lump in the fish ear. These are much like tree rings which reveal fish’s age. Different forms or isotopes of oxygen in the otolith indicate the temperature the fish experienced when it was alive. Carbon isotopes reveal how quickly food is converted into energy. Fish carry their fitness trackers in their ears. They are commonly known as “ear stones,” which are hard, calcium carbonate structures located directly behind the brains of bony fishes. The different types of otoliths that aid fish in balance and hearing are: Sagitta is involved in the detection of sound and the process of hearing, or converting sound waves into electrical signals. Hence statement 1 is correct. Asteriscus is involved in the detection of sound and the process of hearing. Hence statement 2 is correct. Lapillus is involved in the detection of gravitational force and sound. Hence statement 3 is correct.   Incorrect Solution (c) The otolith is a stony lump in the fish ear. These are much like tree rings which reveal fish’s age. Different forms or isotopes of oxygen in the otolith indicate the temperature the fish experienced when it was alive. Carbon isotopes reveal how quickly food is converted into energy. Fish carry their fitness trackers in their ears. They are commonly known as “ear stones,” which are hard, calcium carbonate structures located directly behind the brains of bony fishes. The different types of otoliths that aid fish in balance and hearing are: Sagitta is involved in the detection of sound and the process of hearing, or converting sound waves into electrical signals. Hence statement 1 is correct. Asteriscus is involved in the detection of sound and the process of hearing. Hence statement 2 is correct. Lapillus is involved in the detection of gravitational force and sound. Hence statement 3 is correct.   Question 24 of 35 24. Question Consider the following statements about the Codex Alimentarius Commission: It is an international food safety and quality standard-setting body. It was created by the World Health Organisation and the Food and Agriculture Organization of the United Nations. It meets in regular sessions once in three years alternating between Geneva and Rome. How many of the above statements are correct? a) Only one b) Only two c) All three d) None Correct Solution (b) The Codex Alimentarius Commission is an international food safety and quality standard-setting body. Hence statement 1 is correct. Its objective is protecting consumer’s health and ensuring fair practices in the food trade. It was created by the World Health Organisation and the Food and Agriculture Organization of the United Nations. Hence statement 2 is correct. It consists of 189 member countries. Its membership is open to all Member Nations and Associate Members of FAO and WHO which are interested in international food standards. It meets in regular sessions once a year alternating between Geneva and Rome. Hence statement 3 is incorrect. Incorrect Solution (b) The Codex Alimentarius Commission is an international food safety and quality standard-setting body. Hence statement 1 is correct. Its objective is protecting consumer’s health and ensuring fair practices in the food trade. It was created by the World Health Organisation and the Food and Agriculture Organization of the United Nations. Hence statement 2 is correct. It consists of 189 member countries. Its membership is open to all Member Nations and Associate Members of FAO and WHO which are interested in international food standards. It meets in regular sessions once a year alternating between Geneva and Rome. Hence statement 3 is incorrect. Question 25 of 35 25. Question Consider the following statements about the Indian Tent Turtle: It is listed as critically endangered on the IUCN Red List. It is protected under Schedule I of the Wildlife Protection Act of 1972. It is listed under the Appendix I of the CITES. How many of the above statements are correct? a) Only one b) Only two c) All three d) None Correct Solution (a) The Indian tent turtle (Pangshura tentoria) is a species of turtle in the family Geoemydidae. It is an active swimmer and a mainly herbivorous species. Its habitats include still water pools on the riverside and slow-running water near the river banks. It is native to India, Nepal, and Bangladesh. The Indian Tent Turtle is listed as least concern on the IUCN Red List. Hence statement 1 is incorrect. It is protected under Schedule I of the Wildlife Protection Act of 1972. Hence statement 2 is correct. It is listed under the Appendix II of the CITES. Hence statement 3 is incorrect. Incorrect Solution (a) The Indian tent turtle (Pangshura tentoria) is a species of turtle in the family Geoemydidae. It is an active swimmer and a mainly herbivorous species. Its habitats include still water pools on the riverside and slow-running water near the river banks. It is native to India, Nepal, and Bangladesh. The Indian Tent Turtle is listed as least concern on the IUCN Red List. Hence statement 1 is incorrect. It is protected under Schedule I of the Wildlife Protection Act of 1972. Hence statement 2 is correct. It is listed under the Appendix II of the CITES. Hence statement 3 is incorrect. Question 26 of 35 26. Question Consider the following statements about De Winton’s Golden Mole: It is an elusive blind mole endemic to Australia that swims through sand and lives in inaccessible burrows. Its natural habitats are subtropical dry shrubland, Mediterranean-type shrubby vegetation, and sandy shores. It is currently the only golden mole species listed as critically endangered on the IUCN List. How many of the above statements are correct? a) Only one b) Only two c) All three d) None Correct Solution (b) De Winton’s Golden Mole is an elusive blind mole endemic to South Africa that swims through sand and lives in inaccessible burrows. Hence statement 1 is incorrect. It has only ever been recorded in one location – Port Nolloth, on South Africa’s northwest coast – where it was last sighted in 1937. Its natural habitats are subtropical dry shrubland, Mediterranean-type shrubby vegetation, and sandy shores. Hence statement 2 is correct. The upper parts have short, dense fur that is slate grey with a yellowish tinge. Individual hairs have grey bases, whitish shafts, and fawn tips. The face, cheeks, and lips have a more intense yellowish tinge. The underparts are rather paler than the upper parts, with individual hairs having white tips. It is currently the only golden mole species listed as critically endangered on the IUCN List. Hence statement 3 is correct. It is among the 25 “most wanted lost” species that are the focus of Global Wildlife Conservation’s “Search for Lost Species” initiative. Incorrect Solution (b) De Winton’s Golden Mole is an elusive blind mole endemic to South Africa that swims through sand and lives in inaccessible burrows. Hence statement 1 is incorrect. It has only ever been recorded in one location – Port Nolloth, on South Africa’s northwest coast – where it was last sighted in 1937. Its natural habitats are subtropical dry shrubland, Mediterranean-type shrubby vegetation, and sandy shores. Hence statement 2 is correct. The upper parts have short, dense fur that is slate grey with a yellowish tinge. Individual hairs have grey bases, whitish shafts, and fawn tips. The face, cheeks, and lips have a more intense yellowish tinge. The underparts are rather paler than the upper parts, with individual hairs having white tips. It is currently the only golden mole species listed as critically endangered on the IUCN List. Hence statement 3 is correct. It is among the 25 “most wanted lost” species that are the focus of Global Wildlife Conservation’s “Search for Lost Species” initiative. Question 27 of 35 27. Question Consider the following statements about Operation Storm Makers II: It was conducted by Interpol in which India was a participant. It aims to target fraud schemes fuelled by victims of human trafficking. Choose the correct code: a) 1 only b) 2 only c) Both 1 and 2 d) Neither 1 nor 2 Correct Solution (c) Operation Storm Makers II was conducted by Interpol in which India was a participant. Hence statement 1 is correct. Human trafficking-fuelled frauds are expanding beyond Southeast Asia. Victims are lured by fake job ads and forced to commit online fraud such as – Fake cryptocurrency investments, Work-from-home scams, and Lottery and online gambling scams. It aims to target fraud schemes fuelled by victims of human trafficking. Hence statement 2 is correct.   Incorrect Solution (c) Operation Storm Makers II was conducted by Interpol in which India was a participant. Hence statement 1 is correct. Human trafficking-fuelled frauds are expanding beyond Southeast Asia. Victims are lured by fake job ads and forced to commit online fraud such as – Fake cryptocurrency investments, Work-from-home scams, and Lottery and online gambling scams. It aims to target fraud schemes fuelled by victims of human trafficking. Hence statement 2 is correct.   Question 28 of 35 28. Question Consider the following statements: Abelmoschus Odishae is a new frog species discovered in a moist deciduous forest in Odisha. Indian Council of Agricultural Research (ICAR) is an autonomous organisation under the Department of Agricultural Research and Education (DARE). Choose the correct code: a) 1 only b) 2 only c) Both 1 and 2 d) Neither 1 nor 2 Correct Solution (b) Abelmoschus Odishae is a new plant species of ‘wild okra’ discovered in a moist deciduous forest in Odisha. Hence statement 1 is incorrect. It can be used for hybridisation to create a better variety with high disease resistance. It can also play a vital role in widening the genetic base of okra. Indian Council of Agricultural Research (ICAR) is an autonomous organisation under the Department of Agricultural Research and Education (DARE), Ministry of Agriculture, and Farmers Welfare. Hence statement 2 is correct. It was formerly known as the Imperial Council of Agricultural Research, it was established on 16 July 1929, as a registered society under the Societies Registration Act, 1860. It is responsible for coordinating agricultural education and research in India.   Incorrect Solution (b) Abelmoschus Odishae is a new plant species of ‘wild okra’ discovered in a moist deciduous forest in Odisha. Hence statement 1 is incorrect. It can be used for hybridisation to create a better variety with high disease resistance. It can also play a vital role in widening the genetic base of okra. Indian Council of Agricultural Research (ICAR) is an autonomous organisation under the Department of Agricultural Research and Education (DARE), Ministry of Agriculture, and Farmers Welfare. Hence statement 2 is correct. It was formerly known as the Imperial Council of Agricultural Research, it was established on 16 July 1929, as a registered society under the Societies Registration Act, 1860. It is responsible for coordinating agricultural education and research in India.   Question 29 of 35 29. Question Consider the following statements about the Red Sea: It is the world’s northernmost tropical sea which lies between Asia and Africa. Reef systems are better developed along the sea because of their greater depths and efficient water circulation patterns. It is bordered by Egypt to the north and west and by Sudan, Eritrea, and Djibouti to the west. Choose the correct code: a) Only one b) Only two c) All three d) None Correct Solution (c) Red Sea is the world’s northernmost tropical sea which lies between Asia and Africa. Hence statement 1 is correct. It is connected to the Arabian Sea and the Indian Ocean to the south through the Gulf of Aden and the narrow strait of Bab el-Mandeb. Reef systems are better developed along the sea because of their greater depths and efficient water circulation patterns. Hence statement 2 is correct. Its well-known islands include Tiran Island, which is located near the mouth of the Gulf of Aqaba, and Shadwan Island, which is located at the entrance of the Gulf of Suez. Yemen and Saudi Arabia border the Red Sea to the east. It is bordered by Egypt to the north and west and by Sudan, Eritrea, and Djibouti to the west. Hence statement 3 is correct. Incorrect Solution (c) Red Sea is the world’s northernmost tropical sea which lies between Asia and Africa. Hence statement 1 is correct. It is connected to the Arabian Sea and the Indian Ocean to the south through the Gulf of Aden and the narrow strait of Bab el-Mandeb. Reef systems are better developed along the sea because of their greater depths and efficient water circulation patterns. Hence statement 2 is correct. Its well-known islands include Tiran Island, which is located near the mouth of the Gulf of Aqaba, and Shadwan Island, which is located at the entrance of the Gulf of Suez. Yemen and Saudi Arabia border the Red Sea to the east. It is bordered by Egypt to the north and west and by Sudan, Eritrea, and Djibouti to the west. Hence statement 3 is correct. Question 30 of 35 30. Question Consider the following statements about Mohenjo Daro: It lies in Pakistan’s Sindh province, on the bank of the Indus River. It was divided into two parts – the citadel and the lower town. Its ruins were designated a UNESCO World Heritage Site. How many of the above statements are correct? a) Only one b) Only two c) All three d) None Correct Solution (c) Mohenjo Daro lies in Pakistan’s Sindh province, on the bank of the Indus River. Hence statement 1 is correct. The people of Mohenjo-Daro lived in houses that had similar features throughout the town. They consisted of a central courtyard surrounded by rooms. Almost every household had individual toilets and bathrooms. It was divided into two parts – the citadel and the lower town. Hence statement 2 is correct. The western mound, or Citadel was built on a raised platform of mud brick and consisted of all the important administrative structures like The Great Bath, granaries, and the College of Priests. It was fortified by a thick mud-brick retaining wall. The eastern or low-lying part of the town was also fortified and was meant for the settlement of commoners. Numerous streets and small alleys ran across this part of the city. Within this section, many sub-parts were built according to the guild systems. As people settled here, a large number of artifacts and information regarding the burial practices were found in this area. Its ruins were designated a UNESCO World Heritage Site in 1980. Hence statement 3 is correct. It was laid out in a rectilinear grid and built out of baked bricks, the city featured a complex water management system, complete with a sophisticated drainage and covered sewer system, and baths in nearly every house. Incorrect Solution (c) Mohenjo Daro lies in Pakistan’s Sindh province, on the bank of the Indus River. Hence statement 1 is correct. The people of Mohenjo-Daro lived in houses that had similar features throughout the town. They consisted of a central courtyard surrounded by rooms. Almost every household had individual toilets and bathrooms. It was divided into two parts – the citadel and the lower town. Hence statement 2 is correct. The western mound, or Citadel was built on a raised platform of mud brick and consisted of all the important administrative structures like The Great Bath, granaries, and the College of Priests. It was fortified by a thick mud-brick retaining wall. The eastern or low-lying part of the town was also fortified and was meant for the settlement of commoners. Numerous streets and small alleys ran across this part of the city. Within this section, many sub-parts were built according to the guild systems. As people settled here, a large number of artifacts and information regarding the burial practices were found in this area. Its ruins were designated a UNESCO World Heritage Site in 1980. Hence statement 3 is correct. It was laid out in a rectilinear grid and built out of baked bricks, the city featured a complex water management system, complete with a sophisticated drainage and covered sewer system, and baths in nearly every house. Question 31 of 35 31. Question The sum of 3 numbers is 85. If third number be one fifth of the first and the first number be twice the second, find the second number   a) 20 b) 25 c) 10 d) 15 Correct Solution (b) We know from the details of the question that the ratio of the numbers = 1: 1/2: 1/5 Taking LCM we get the ratio = 10: 5: 2 The total of the ratios = 17 Second number = 5/17 × 85 = 25   Incorrect Solution (b) We know from the details of the question that the ratio of the numbers = 1: 1/2: 1/5 Taking LCM we get the ratio = 10: 5: 2 The total of the ratios = 17 Second number = 5/17 × 85 = 25   Question 32 of 35 32. Question A contract is to be completed in 60 days if 105 persons work, each working at 8 hours a day. After 30 days, 2/5 of the work is completed. How many additional persons should be deployed so that the work will be completed in the scheduled time, each person now working 9 hours a day?   a) 35 b) 40 c) 28 d) 43 Correct Solution (a) Persons worked (M1) = 105 Number of hours each person worked per day (H1) = 8 Number of days they worked (D1) = 30 Work completed (W1)= 2/5 Remaining days (D2)= 60 – 30 = 30 Remaining Work to be completed (W2) = (1 – 2/5 ) = 3/5 Let the total number of persons who do the remaining work (M2) = x Number of hours each person needs to be work per day (H2) = 9 = M1D1H1/W1 = M2D2H2/W2 = (105×30×8)/ (2/5) = (x×30×9)/ (3/5) => x = 140 So, additional person = 140 – 105 = 35 Incorrect Solution (a) Persons worked (M1) = 105 Number of hours each person worked per day (H1) = 8 Number of days they worked (D1) = 30 Work completed (W1)= 2/5 Remaining days (D2)= 60 – 30 = 30 Remaining Work to be completed (W2) = (1 – 2/5 ) = 3/5 Let the total number of persons who do the remaining work (M2) = x Number of hours each person needs to be work per day (H2) = 9 = M1D1H1/W1 = M2D2H2/W2 = (105×30×8)/ (2/5) = (x×30×9)/ (3/5) => x = 140 So, additional person = 140 – 105 = 35 Question 33 of 35 33. Question It is not only the untreated sewage water and industrial effluents, but also the solid wastes and construction material discharged by individuals, companies and municipal bodies that have caused the suffocation of the Yamuna. Also, floodplains have been encroached upon by settlements. Hence, ensuring supply of quality drinking water is not only expensive, it also needs improvement in governance. It needs technical knowledge on measurement and regulation of water quality. It is not the fault of the DJB or the Delhi government alone that they have not been able to ensure 100% supply of quality water to the citizens of Delhi, considering the constraints they face, especially those concerning the water resources management and laws in the country. Which one of the following is the most logical and rational inference that can be made from the above passage? a) The water quality of Yamuna water can be improved only by clearing the encroachment on the flood plains. b) Many factors are at play that need to be addressed for improving the quality of water of a river. c) Improper management of resources and lack of proper laws have no effect on the water quality of a river. d) Climate change and unplanned development is taking a toll on the quality of water of a river. Correct Solution (b) Many factors like control of effluents and dumping of construction material, removing the encroachments from the flood plains, proper monitoring of water quality etc. are needed to improve the quality of the water of a river. The impact of climate change has not been discussed in the passage. Keeping these things in the mind, the most appropriate among the given answer options is option B. Incorrect Solution (b) Many factors like control of effluents and dumping of construction material, removing the encroachments from the flood plains, proper monitoring of water quality etc. are needed to improve the quality of the water of a river. The impact of climate change has not been discussed in the passage. Keeping these things in the mind, the most appropriate among the given answer options is option B. Question 34 of 35 34. Question The structure of India‘s population—that has been famously described as a “pyramid” is changing to that of an “unhealthy middle-aged man with a big head and fat belly”. The heavy belly comes from a dramatic increase in the size of the lower middle class and the big head comes from rising income inequality. The richest 1% of India now hold 58% of wealth, and the richest 10% over 80%. This structural shift will have profound consequences for politics and economics, with an aspiring class that is very large and growing, and with abject poverty declining to a much smaller percentage of the population. The terms “big head” and “fat belly”, as used in the passage, refer to a) Changing lifestyle of Indian people and the ensuing physical and mental health issues b) Uneven patterns of income distribution across different strata of population. c) Change in population profile of Indian population. d) Unhealthy dietary habits leading to an increase in average waist size of Indians Correct Solution (b) The passage begins by mentioning the structure of Indian population vis-a-vis its income, which has a pyramidal pattern. The terms “big head” and “fat belly” refer to the change in this pyramidal pattern. “Heavy belly” refers to a dramatic increase in the size of the lower middle class, and “big head” refers to rising income inequality. Option C might seem a little confusing. The passage talks about income distribution, not the number of people as a function of their ages. Hence, option C is incorrect. Incorrect Solution (b) The passage begins by mentioning the structure of Indian population vis-a-vis its income, which has a pyramidal pattern. The terms “big head” and “fat belly” refer to the change in this pyramidal pattern. “Heavy belly” refers to a dramatic increase in the size of the lower middle class, and “big head” refers to rising income inequality. Option C might seem a little confusing. The passage talks about income distribution, not the number of people as a function of their ages. Hence, option C is incorrect. Question 35 of 35 35. Question As a poet, Kabir transcended many of the divisions that existed in India. He can be celebrated as Dalit hero or as a Brahmin. Kabir and the Kabir Panth are accepted as a part of Hinduism. A large corpus of his poems is included in the Guru Granth Sahib. His presence in Indian Islamic thought, Qawwali singing and architecture has also been well documented. Nineteenth century missionaries noted the similarity of his thoughts to Christianity. His indebtedness to Buddhist Siddhas has been a subject of scholarship in the last century. Jain poets emulated his style, so much so that the 17th century Anandghan was dubbed as the ‘Jain Kabir’. The passage seems to argue that a) Kabir‘s thoughts are reflected in almost all major religions in India. b) Kabir supported different religions at different phases of his life. c) Kabir denounced untouchability and favoured equality among different castes. d) Kabir‘s presence in Indian Islamic thought is more prominent as compared to other religions. Correct Solution (a) Option A is correct. Throughout the passage it can be seen that the author is explaining how Kabir‘s thoughts have been incorporated by various religions in India. For example, he is celebrated as a Dalit hero or a Brahmin. His poems which reflected his thoughts are included in Guru Granth Sahib (Sikhs). Similar connection with other religions like Christianity, Jainism and Islam is also established in the passage. Option B is incorrect. The passage only explains how Kabir‘s thoughts have been included in various religions around India. It nowhere mentions whether Kabir supported any religion or not. Hence, this statement is not correct. Option C is incorrect. Though the passage mentions in the beginning, “He can be celebrated as Dalit hero or as a Brahmin”, it cannot be conclusively established from this paragrapn that Kabir advocated equality between different castes. Also there is no mention of untouchability in the passage. Hence, this statement is not correct. Option D is incorrect. The passage mentions the presence of Kabir’s thoughts in Islam through the statement, “His presence in Indian Islamic thought, Qawwali singing and architecture has also been well documented”. But, the passage does not argue that his presence in Islamic thought is more prominent than that in other religions.   Incorrect Solution (a) Option A is correct. Throughout the passage it can be seen that the author is explaining how Kabir‘s thoughts have been incorporated by various religions in India. For example, he is celebrated as a Dalit hero or a Brahmin. His poems which reflected his thoughts are included in Guru Granth Sahib (Sikhs). Similar connection with other religions like Christianity, Jainism and Islam is also established in the passage. Option B is incorrect. The passage only explains how Kabir‘s thoughts have been included in various religions around India. It nowhere mentions whether Kabir supported any religion or not. Hence, this statement is not correct. Option C is incorrect. Though the passage mentions in the beginning, “He can be celebrated as Dalit hero or as a Brahmin”, it cannot be conclusively established from this paragrapn that Kabir advocated equality between different castes. Also there is no mention of untouchability in the passage. Hence, this statement is not correct. Option D is incorrect. The passage mentions the presence of Kabir’s thoughts in Islam through the statement, “His presence in Indian Islamic thought, Qawwali singing and architecture has also been well documented”. But, the passage does not argue that his presence in Islamic thought is more prominent than that in other religions.   window.wpProQuizInitList = window.wpProQuizInitList || []; window.wpProQuizInitList.push({ id: '#wpProQuiz_3627', init: { quizId: 3627, mode: 1, globalPoints: 70, timelimit: 1800, resultsGrade: [0], bo: 704, qpp: 0, catPoints: [70], formPos: 0, lbn: "Test-summary", json: {"32004":{"type":"single","id":32004,"catId":0,"points":2,"correct":[0,1,0,0]},"32006":{"type":"single","id":32006,"catId":0,"points":2,"correct":[0,0,1,0]},"32008":{"type":"single","id":32008,"catId":0,"points":2,"correct":[1,0,0,0]},"32009":{"type":"single","id":32009,"catId":0,"points":2,"correct":[0,0,0,1]},"32011":{"type":"single","id":32011,"catId":0,"points":2,"correct":[0,1,0,0]},"32014":{"type":"single","id":32014,"catId":0,"points":2,"correct":[0,1,0,0]},"32016":{"type":"single","id":32016,"catId":0,"points":2,"correct":[1,0,0,0]},"32019":{"type":"single","id":32019,"catId":0,"points":2,"correct":[1,0,0,0]},"32022":{"type":"single","id":32022,"catId":0,"points":2,"correct":[0,0,0,1]},"32024":{"type":"single","id":32024,"catId":0,"points":2,"correct":[1,0,0,0]},"32026":{"type":"single","id":32026,"catId":0,"points":2,"correct":[0,0,1,0]},"32028":{"type":"single","id":32028,"catId":0,"points":2,"correct":[0,0,0,1]},"32029":{"type":"single","id":32029,"catId":0,"points":2,"correct":[0,0,1,0]},"32031":{"type":"single","id":32031,"catId":0,"points":2,"correct":[0,0,0,1]},"32033":{"type":"single","id":32033,"catId":0,"points":2,"correct":[0,0,1,0]},"32034":{"type":"single","id":32034,"catId":0,"points":2,"correct":[0,1,0,0]},"32036":{"type":"single","id":32036,"catId":0,"points":2,"correct":[1,0,0,0]},"32038":{"type":"single","id":32038,"catId":0,"points":2,"correct":[0,1,0,0]},"32041":{"type":"single","id":32041,"catId":0,"points":2,"correct":[1,0,0,0]},"32043":{"type":"single","id":32043,"catId":0,"points":2,"correct":[1,0,0,0]},"32046":{"type":"single","id":32046,"catId":0,"points":2,"correct":[0,0,1,0]},"32047":{"type":"single","id":32047,"catId":0,"points":2,"correct":[0,0,1,0]},"32048":{"type":"single","id":32048,"catId":0,"points":2,"correct":[0,0,1,0]},"32050":{"type":"single","id":32050,"catId":0,"points":2,"correct":[0,1,0,0]},"32053":{"type":"single","id":32053,"catId":0,"points":2,"correct":[1,0,0,0]},"32054":{"type":"single","id":32054,"catId":0,"points":2,"correct":[0,1,0,0]},"32055":{"type":"single","id":32055,"catId":0,"points":2,"correct":[0,0,1,0]},"32057":{"type":"single","id":32057,"catId":0,"points":2,"correct":[0,1,0,0]},"32059":{"type":"single","id":32059,"catId":0,"points":2,"correct":[0,0,1,0]},"32061":{"type":"single","id":32061,"catId":0,"points":2,"correct":[0,0,1,0]},"32062":{"type":"single","id":32062,"catId":0,"points":2,"correct":[0,1,0,0]},"32064":{"type":"single","id":32064,"catId":0,"points":2,"correct":[1,0,0,0]},"32067":{"type":"single","id":32067,"catId":0,"points":2,"correct":[0,1,0,0]},"32069":{"type":"single","id":32069,"catId":0,"points":2,"correct":[0,1,0,0]},"32071":{"type":"single","id":32071,"catId":0,"points":2,"correct":[1,0,0,0]}} } }); All the Best IASbaba

Daily Prelims CA Quiz

UPSC Quiz – 2024 : IASbaba’s Daily Current Affairs Quiz 10th April 2024

For Previous Daily Quiz (ARCHIVES) – CLICK HERE The Current Affairs questions are based on sources like ‘The Hindu’, ‘Indian Express’ and ‘PIB’, which are very important sources for UPSC Prelims Exam. The questions are focused on both the concepts and facts. The topics covered here are generally different from what is being covered under ‘Daily Current Affairs/Daily News Analysis (DNA) and Daily Static Quiz’ to avoid duplication. The questions would be published from Monday to Saturday before 2 PM. One should not spend more than 10 minutes on this initiative. Gear up and Make the Best Use of this initiative. Do remember that, “the difference between Ordinary and EXTRA-Ordinary is PRACTICE!!” Important Note: Don’t forget to post your marks in the comment section. Also, let us know if you enjoyed today’s test 🙂After completing the 5 questions, click on ‘View Questions’ to check your score, time taken, and solutions. To take the Test Click Here

[DAY 32] 60 DAY RAPID REVISION (RaRe) SERIES for UPSC Prelims 2024 – GEOGRAPHY, CURRENT AFFAIRS & CSAT TEST SERIES!

Archives Hello Friends The 60 Days Rapid Revision (RaRe) Series is IASbaba’s Flagship Initiative recommended by Toppers and loved by the aspirants’ community every year. It is the most comprehensive program which will help you complete the syllabus, revise and practice tests on a daily basis. The Programme on a daily basis includes Daily Prelims MCQs from Static (Monday – Saturday) Daily Static Quiz will cover all the topics of static subjects – Polity, History, Geography, Economics, Environment and Science and technology. 20 questions will be posted daily and these questions are framed from the topics mentioned in the schedule. It will ensure timely and streamlined revision of your static subjects. Daily Current Affairs MCQs (Monday – Saturday) Daily 5 Current Affairs questions, based on sources like ‘The Hindu’, ‘Indian Express’ and ‘PIB’, would be published from Monday to Saturday according to the schedule. Daily CSAT Quiz (Monday – Friday) CSAT has been an Achilles heel for many aspirants. Daily 5 CSAT Questions will be published. Note – Daily Test of 20 static questions, 10 current affairs, and 5 CSAT questions. (35 Prelims Questions) in QUIZ FORMAT will be updated on a daily basis. To Know More about 60 Days Rapid Revision (RaRe) Series – CLICK HERE   60 Day Rapid Revision (RaRe) Series Schedule – CLICK HERE  Important Note Comment your Scores in the Comment Section. This will keep you accountable, responsible and sincere in days to come. It will help us come out with the Cut-Off on a Daily Basis. Let us know if you enjoyed today’s test 🙂  You can post your comments in the given format  (1) Your Score (2) Matrix Meter (3) New Learning from the Test Time limit: 0 Test-summary 0 of 35 questions completed Questions: 1 2 3 4 5 6 7 8 9 10 11 12 13 14 15 16 17 18 19 20 21 22 23 24 25 26 27 28 29 30 31 32 33 34 35 Information The following Test is based on the syllabus of 60 Days Plan-2023 for UPSC IAS Prelims 2022. To view Solutions, follow these instructions: Click on – ‘Start Test’ button Solve Questions Click on ‘Test Summary’ button Click on ‘Finish Test’ button Now click on ‘View Questions’ button – here you will see solutions and links. You have already completed the test before. Hence you can not start it again. Test is loading... You must sign in or sign up to start the test. You have to finish following test, to start this test: Results 0 of 35 questions answered correctly Your time: Time has elapsed You have scored 0 points out of 0 points, (0) Average score     Your score     Categories Not categorized 0% Your result has been entered into leaderboard Loading Name: E-Mail: Captcha: maximum of 70 points Pos. Name Entered on Points Result Table is loading No data available 1 2 3 4 5 6 7 8 9 10 11 12 13 14 15 16 17 18 19 20 21 22 23 24 25 26 27 28 29 30 31 32 33 34 35 Answered Review Question 1 of 35 1. Question Consider the following statements with respect to the Five-Year Planning (FYP) phase: Implementation of Family Planning Programs was amongst the major targets of the seventh FYP. The ninth FYP for the first time had a separate plan document on states. The concept of Inclusive growth was introduced for the first time in the eighth FYP. Select the correct answer using the code given below: a) Only one b) Only two c) All three d) None Correct Solution (d) Statement 1 Statement 2 Statement 3 Incorrect Incorrect Incorrect The objective of fourth FYP (1969-74) includes “growth with stability and progressive achievement of self- reliance”. Implementation of Family Planning Programs was amongst the major targets of the Plan. The tenth FYP (2002 – 2007) sought to double national per capita income and create a hundred million jobs in 10 years. The 10th FYP emphasized regional imbalance and for the first time had a separate plan document on states. The eleventh plan (2007 – 2012) introduced the concept of inclusive growth. It focused on human resources, especially health and skill development. Incorrect Solution (d) Statement 1 Statement 2 Statement 3 Incorrect Incorrect Incorrect The objective of fourth FYP (1969-74) includes “growth with stability and progressive achievement of self- reliance”. Implementation of Family Planning Programs was amongst the major targets of the Plan. The tenth FYP (2002 – 2007) sought to double national per capita income and create a hundred million jobs in 10 years. The 10th FYP emphasized regional imbalance and for the first time had a separate plan document on states. The eleventh plan (2007 – 2012) introduced the concept of inclusive growth. It focused on human resources, especially health and skill development. Question 2 of 35 2. Question Which of the following expenditures are considered as Non-Developmental expenditure? Expenditure on Administration Maintenance of law and order Cost of tax collection Select the correct answer using the code given below. a) 1 and 2 only b) 2 and 3 only c) 1 and 3 only d) 1, 2 and 3 Correct Solution (d) Statement 1 Statement 2 Statement 3 Correct Correct Correct Non-developmental expenditure includes expenditures made for administrative service, defence service, debt servicing, subsidies, etc. Non-developmental expenditure includes expenditure made on maintenance of law and order. Cost of tax collection, cost of an audit, printing of notes, internal law and order are also included under Non-development expenditure. Incorrect Solution (d) Statement 1 Statement 2 Statement 3 Correct Correct Correct Non-developmental expenditure includes expenditures made for administrative service, defence service, debt servicing, subsidies, etc. Non-developmental expenditure includes expenditure made on maintenance of law and order. Cost of tax collection, cost of an audit, printing of notes, internal law and order are also included under Non-development expenditure. Question 3 of 35 3. Question Consider the following pairs: Deficit                                                 Description Effective Revenue Deficit:: Fiscal deficit – Interest payments Primary Deficit::            Revenue Deficit – Grants for the creation of capital assets Monetized Deficit::            Government deficit financed by borrowing from the RBI How many of the above pairs is/are correctly matched? a) None of the pairs b) One pair only c) Two pairs only d) Three pairs only Correct Solution (b) Statement 1 Statement 2 Statement 3 Incorrect Incorrect Correct Revenue deficit is the difference between revenue receipts and revenue expenditure. Effective Revenue deficit is a new term introduced in the Union Budget 2011-12.   Effective Revenue Deficit = Revenue Deficit – Grants for the creation of capital assets. Primary Deficit is the difference between the current year’s fiscal deficit (Total income– Total expenditure of the government) and the interest paid on the borrowings of the previous year.   Primary Deficit = Fiscal deficit – Interest payments Monetized deficit is also known as the ‘net reserve bank credit to the government’. It is that part of the government deficit which is financed solely by borrowing from the RBI Incorrect Solution (b) Statement 1 Statement 2 Statement 3 Incorrect Incorrect Correct Revenue deficit is the difference between revenue receipts and revenue expenditure. Effective Revenue deficit is a new term introduced in the Union Budget 2011-12.   Effective Revenue Deficit = Revenue Deficit – Grants for the creation of capital assets. Primary Deficit is the difference between the current year’s fiscal deficit (Total income– Total expenditure of the government) and the interest paid on the borrowings of the previous year.   Primary Deficit = Fiscal deficit – Interest payments Monetized deficit is also known as the ‘net reserve bank credit to the government’. It is that part of the government deficit which is financed solely by borrowing from the RBI Question 4 of 35 4. Question With reference to the First Five-Year Plan in India, consider the following statement: It focused on rapid industrialization using heavy and basic industries. The actual growth rate exceeded the target growth rate set by the five-year plan. Choose the correct code: a) 1 only b) 2 only c) Both 1 and 2 d) Neither 1 nor 2 Correct Solution (b) Statement 1 Statement 2 Incorrect Correct An influx of refugees, severe food shortage & mounting inflation confronted the country at the onset of the First Five-year Plan. The Plan Focused on agriculture, price stability, power, and transport. The First Five-Year Plan (1951-56) kept the Growth rate target of 2.1%. Whereas, the actual Growth rate was 3.6%.   Incorrect Solution (b) Statement 1 Statement 2 Incorrect Correct An influx of refugees, severe food shortage & mounting inflation confronted the country at the onset of the First Five-year Plan. The Plan Focused on agriculture, price stability, power, and transport. The First Five-Year Plan (1951-56) kept the Growth rate target of 2.1%. Whereas, the actual Growth rate was 3.6%.   Question 5 of 35 5. Question Which of the following can lead to the condition of ‘Fiscal Slippage’ in an economy? Farm loan waivers Decline in tax collection Increase in disinvestment proceeds How many of the above statements are correct? a) Only one b) Only two c) All three d) None Correct Solution (b) Statement 1 Statement 2 Statement 3 Correct Correct Incorrect Fiscal slippage in simple terms is any deviation in expenditure from the expected. When the Government’s expenditure surpasses the expected or estimated levels, the nation might face the threat of fiscal slippage. Farm loan waivers increase the expenditure that will lead to the condition of fiscal slippage. Tax rate cuts lead to a decline in tax revenue that will lead to the condition of fiscal slippage.   Increase in disinvestment proceed rather prevents (not lead to) the situation of fiscal slippage Incorrect Solution (b) Statement 1 Statement 2 Statement 3 Correct Correct Incorrect Fiscal slippage in simple terms is any deviation in expenditure from the expected. When the Government’s expenditure surpasses the expected or estimated levels, the nation might face the threat of fiscal slippage. Farm loan waivers increase the expenditure that will lead to the condition of fiscal slippage. Tax rate cuts lead to a decline in tax revenue that will lead to the condition of fiscal slippage.   Increase in disinvestment proceed rather prevents (not lead to) the situation of fiscal slippage Question 6 of 35 6. Question With reference to Revenue Deficit, consider the following statements: It includes such transactions that affect the current income and expenditure of the government. It signifies that government is using up the savings to finance a part of its consumption expenditure. Which of the statements given above is/are correct? a) 1 only b) 2 only c) Both 1 and 2 d) Neither 1 nor 2 Correct Solution (c) Statement 1 Statement 2 Correct Correct The revenue deficit includes only such transactions that affect the current income and expenditure of the government. When the government incurs a revenue deficit, it implies that the government is dissaving and is using up the savings of the other sectors of the economy to finance a part of its consumption expenditure.   This situation means that the government will have to borrow not only to finance its investment but also its consumption requirements. This will lead to a build-up of stock of debt and interest liabilities and force the government, eventually, to cut expenditure. Since a major part of revenue expenditure is committed expenditure, it cannot be reduced. Often the government reduces productive capital expenditure or welfare expenditure. This would mean lower growth and adverse welfare implications. Incorrect Solution (c) Statement 1 Statement 2 Correct Correct The revenue deficit includes only such transactions that affect the current income and expenditure of the government. When the government incurs a revenue deficit, it implies that the government is dissaving and is using up the savings of the other sectors of the economy to finance a part of its consumption expenditure.   This situation means that the government will have to borrow not only to finance its investment but also its consumption requirements. This will lead to a build-up of stock of debt and interest liabilities and force the government, eventually, to cut expenditure. Since a major part of revenue expenditure is committed expenditure, it cannot be reduced. Often the government reduces productive capital expenditure or welfare expenditure. This would mean lower growth and adverse welfare implications. Question 7 of 35 7. Question Consider the following statements regarding ‘Short Term Capital Gains Tax’ in India: It refers to a tax on the gain that arises from the sale of an asset after holding it less than sixty months. Securities Transaction Tax (STT) is an indirect tax levied on sale and purchase of equities. Select the correct answer using the code given below: a) 1 only b) 2 only c) Both 1 and 2 d) Neither 1 nor 2 Correct Solution (d) Statement 1 Statement 2 Incorrect Incorrect Short Term Capital Gains Tax (STCG) is the tax levied on profits generated from the sale of an asset which is held for a short-term period which differs for various items – for example it is 36 months or less in the case of immovable property such as land and building and 12 months or less for equity, bonds, government securities etc. The Securities Transaction Tax (STT) is a type of direct tax which is levied at the time of purchase and sale of securities listed on stock exchanges in India. Securities are tradable investment instruments such as shares bonds, debentures, equity-oriented mutual funds (MFs) and so on and are issued either by companies or by the Indian government. This tax was introduced in the Union Budget of 2004. Incorrect Solution (d) Statement 1 Statement 2 Incorrect Incorrect Short Term Capital Gains Tax (STCG) is the tax levied on profits generated from the sale of an asset which is held for a short-term period which differs for various items – for example it is 36 months or less in the case of immovable property such as land and building and 12 months or less for equity, bonds, government securities etc. The Securities Transaction Tax (STT) is a type of direct tax which is levied at the time of purchase and sale of securities listed on stock exchanges in India. Securities are tradable investment instruments such as shares bonds, debentures, equity-oriented mutual funds (MFs) and so on and are issued either by companies or by the Indian government. This tax was introduced in the Union Budget of 2004. Question 8 of 35 8. Question Which of the following statements is correct with respect to ‘General Anti-Avoidance Rule (GAAR)’? a) It is designed to prevent tax avoidance in cross border transactions only. b) Its provisions are included in the Income Tax Act, 1961. c) Its provisions are effective from January 2019. d) As of now, there are no appellate tribunals to appeal against the decisions of tax authorities regarding GAAR provisions. Correct Solution (b) GAAR is an anti-tax avoidance law under Chapter X-A of the Income Tax Act, 1961 of India. It is framed by the Department of Revenue under the Ministry of Finance. GAAR applies to any arrangement that is considered an Impermissible Avoidance Arrangement (IAA). Furthermore, under its provisions, certain transactions are deemed to lack commercial substance. Once the Revenue authorities decide to treat an arrangement as an IAA, the onus to prove otherwise is on taxpayers. Consequently, they are required to substantiate the commercial reasons for such arrangements and that availing tax benefit was not the main purpose for these transactions. Under the Income Tax act, a taxpayer has the right to appeal to the Income Tax Appellate Tribunal (ITAT) against an order passed by the Revenue authorities along with the direction of the Approving Panel. Incorrect Solution (b) GAAR is an anti-tax avoidance law under Chapter X-A of the Income Tax Act, 1961 of India. It is framed by the Department of Revenue under the Ministry of Finance. GAAR applies to any arrangement that is considered an Impermissible Avoidance Arrangement (IAA). Furthermore, under its provisions, certain transactions are deemed to lack commercial substance. Once the Revenue authorities decide to treat an arrangement as an IAA, the onus to prove otherwise is on taxpayers. Consequently, they are required to substantiate the commercial reasons for such arrangements and that availing tax benefit was not the main purpose for these transactions. Under the Income Tax act, a taxpayer has the right to appeal to the Income Tax Appellate Tribunal (ITAT) against an order passed by the Revenue authorities along with the direction of the Approving Panel. Question 9 of 35 9. Question In the context of Indian economy, what is ‘Economic Capital Framework’? a) It is a part of the annual budget documents, intended to be placed before the parliament, outlining the various strategies for capital generation. b) It is a framework developed by the RBI to determine the appropriate level of risk provisions and distribution of profits between the RBI and the Union Government. c) It is the annual submission of RBI under the Basel Committee outlining the former's commitments. d) It is the framework developed by the NITI Aayog that guides the sharing of revenues between the Centre and the states. Correct Solution (b) The economic capital framework provides a methodology for determining the appropriate level of risk provisions and profit distribution to be made under Section 47 of the RBI Act, 1934. As per this provision, the central bank is required to pay balance of its profits to the central government after making provision for bad and doubtful debts, depreciation in assets, and contributions to staff. Incorrect Solution (b) The economic capital framework provides a methodology for determining the appropriate level of risk provisions and profit distribution to be made under Section 47 of the RBI Act, 1934. As per this provision, the central bank is required to pay balance of its profits to the central government after making provision for bad and doubtful debts, depreciation in assets, and contributions to staff. Question 10 of 35 10. Question Which of the following fiscal policy statements are required to be laid before the Parliament under the Fiscal Responsibility and Budget Management Act, 2003 (FRBMA)? Medium-term Fiscal Policy Fiscal Policy Strategy Outcome Budget Medium-term Expenditure Framework Select the correct answer using the code given below. a) 1 and 3 only b) 1, 2 and 4 only c) 2 and 4 only d) 1, 2, 3 and 4 Correct Solution (b) Statement 1 Statement 2 Statement 3 Statement 4 Correct Correct Incorrect Correct The FRBM Act includes the Medium Term Fiscal Policy. It sets out the three-year rolling targets for five specific fiscal indicators in relation to GDP at market prices, namely (i) Revenue Deficit, (ii) Fiscal Deficit, (iii) Effective Revenue Deficit (iv) Tax to GDP ratio and (v) Total outstanding Central Government Debt at the end of the year. The Fiscal Policy Strategy Statement, presented to Parliament as per the FRBM Act. The Strategy Statement outlines the strategic priorities of Government in the fiscal area for the ensuing financial year relating to taxation, expenditure, lending and investments, administered pricing, borrowings and guarantees. The Statement explains how the current policies are in conformity with sound fiscal management principles and gives the rationale for any major deviation in key fiscal measures. The introduction of Outcome Budget is an executive action by the government. From the year 2017-18 onwards, it has been decided that the output and outcomes of the schemes of 68 Ministries and Departments will be available along with the financial outlays as a part of the Budget documents, so that clearly defined objectives and goals for each scheme can be seen by all. The medium-term expenditure framework (MTEF) statement sets a three-year rolling target for expenditure indicators, along with specifications of underpinning assumptions and risks. This statement is presented in Parliament under Section 3 of the FRBM Act, 2003.   The statement provides an estimate of expenditure for various sectors, including education, health, rural development, energy, subsidies and pension, and so on. Incorrect Solution (b) Statement 1 Statement 2 Statement 3 Statement 4 Correct Correct Incorrect Correct The FRBM Act includes the Medium Term Fiscal Policy. It sets out the three-year rolling targets for five specific fiscal indicators in relation to GDP at market prices, namely (i) Revenue Deficit, (ii) Fiscal Deficit, (iii) Effective Revenue Deficit (iv) Tax to GDP ratio and (v) Total outstanding Central Government Debt at the end of the year. The Fiscal Policy Strategy Statement, presented to Parliament as per the FRBM Act. The Strategy Statement outlines the strategic priorities of Government in the fiscal area for the ensuing financial year relating to taxation, expenditure, lending and investments, administered pricing, borrowings and guarantees. The Statement explains how the current policies are in conformity with sound fiscal management principles and gives the rationale for any major deviation in key fiscal measures. The introduction of Outcome Budget is an executive action by the government. From the year 2017-18 onwards, it has been decided that the output and outcomes of the schemes of 68 Ministries and Departments will be available along with the financial outlays as a part of the Budget documents, so that clearly defined objectives and goals for each scheme can be seen by all. The medium-term expenditure framework (MTEF) statement sets a three-year rolling target for expenditure indicators, along with specifications of underpinning assumptions and risks. This statement is presented in Parliament under Section 3 of the FRBM Act, 2003.   The statement provides an estimate of expenditure for various sectors, including education, health, rural development, energy, subsidies and pension, and so on. Question 11 of 35 11. Question Which of the following actions implemented by the government may lead to reduction in its deficit? Increase in the rate of personal Income Tax. Sale of shares of Public Sector Undertakings. Introduction of new welfare schemes by the government. How many of the above statements are correct? a) Only one b) Only two c) All three d) None Correct Solution (b) Statement 1 Statement 2 Statement 3 Correct Correct Incorrect Government deficit can be reduced by an increase in taxes or reduction in expenditure. In India, the government has been trying to increase tax revenue with greater reliance on direct taxes. There has also been an attempt to raise receipts through the sale of shares in PSUs. The other way is to change the scope of the government by withdrawing from some of the areas where it operated before. Cutting back government programmes in vital areas like agriculture, education, health, poverty alleviation, etc. would adversely affect the economy. Incorrect Solution (b) Statement 1 Statement 2 Statement 3 Correct Correct Incorrect Government deficit can be reduced by an increase in taxes or reduction in expenditure. In India, the government has been trying to increase tax revenue with greater reliance on direct taxes. There has also been an attempt to raise receipts through the sale of shares in PSUs. The other way is to change the scope of the government by withdrawing from some of the areas where it operated before. Cutting back government programmes in vital areas like agriculture, education, health, poverty alleviation, etc. would adversely affect the economy. Question 12 of 35 12. Question The government of a nation performs its income distribution function through which of the following mechanisms? Levying taxes Transfer Payment Controlling money supply How many of the above statements are correct? a) Only one b) Only two c) All three d) None Correct Solution (b) Statement 1 Statement 2 Statement 3 Correct Correct Incorrect Through its tax and expenditure policy, the government attempts to bring about a distribution of income that is considered, fair by society. The government affects the personal disposable income of households by making transfer payments and collecting taxes and, therefore, can alter the income distribution. This is the distribution function. Transfer Payment is a payment made or income received in which no goods or services are being paid for, such as a benefit payment or subsidy. Unlike the exchange transaction which mutually benefits all the parties involved in it, the transfer payment consists of a donor and a recipient, with the donor giving up something of value without receiving anything in return. In any period, the level of expenditures may not be sufficient for full utilisation of labour and other resources of the economy. Since wages and prices are generally rigid downwards (they do not fall below a level), employment cannot be restored automatically. Hence, policy measures are needed to raise aggregate demand. Incorrect Solution (b) Statement 1 Statement 2 Statement 3 Correct Correct Incorrect Through its tax and expenditure policy, the government attempts to bring about a distribution of income that is considered, fair by society. The government affects the personal disposable income of households by making transfer payments and collecting taxes and, therefore, can alter the income distribution. This is the distribution function. Transfer Payment is a payment made or income received in which no goods or services are being paid for, such as a benefit payment or subsidy. Unlike the exchange transaction which mutually benefits all the parties involved in it, the transfer payment consists of a donor and a recipient, with the donor giving up something of value without receiving anything in return. In any period, the level of expenditures may not be sufficient for full utilisation of labour and other resources of the economy. Since wages and prices are generally rigid downwards (they do not fall below a level), employment cannot be restored automatically. Hence, policy measures are needed to raise aggregate demand. Question 13 of 35 13. Question Which of the following is/are included in Public Debt of India? Treasury bills Outstanding external debts Borrowing from small savings How many of the above statements are correct? a) Only one b) Only two c) All three d) None Correct Solution (b) Statement 1 Statement 2 Statement 3 Correct Correct Incorrect In India, public debt refers to a part of the total borrowings by the Union Government which includes such items as market loans, special bearer bonds, treasury bills and special loans and securities issued by the Reserve Bank. It also includes the outstanding external debt. At end-March 2021, India’s external debt was placed at US$ 570.0 billion Public debt does not include the following items of borrowings: ·       small savings, ·       provident funds, ·       other accounts, reserve funds and deposits. Incorrect Solution (b) Statement 1 Statement 2 Statement 3 Correct Correct Incorrect In India, public debt refers to a part of the total borrowings by the Union Government which includes such items as market loans, special bearer bonds, treasury bills and special loans and securities issued by the Reserve Bank. It also includes the outstanding external debt. At end-March 2021, India’s external debt was placed at US$ 570.0 billion Public debt does not include the following items of borrowings: ·       small savings, ·       provident funds, ·       other accounts, reserve funds and deposits. Question 14 of 35 14. Question With reference to Primary Deficit, consider the following statements: The primary deficit is the sum of Fiscal deficit and interest payments. It includes the burden of the past debt. Which of the statements given above is/are correct? a) 1 only b) 2 only c) Both 1 and 2 d) Neither 1 nor 2 Correct Solution (d) Statement 1 Statement 2 Incorrect Incorrect Gross Primary deficit is defined as gross fiscal deficit minus net interest payments. Net primary deficit, is gross primary deficit minus net domestic lending. It excludes the burden of the past debt and shows the net increase in the government’s indebtedness due to the current year’s fiscal operations. A reduction in primary deficit is reflective of government’s efforts at bridging the fiscal gap during a financial year. Incorrect Solution (d) Statement 1 Statement 2 Incorrect Incorrect Gross Primary deficit is defined as gross fiscal deficit minus net interest payments. Net primary deficit, is gross primary deficit minus net domestic lending. It excludes the burden of the past debt and shows the net increase in the government’s indebtedness due to the current year’s fiscal operations. A reduction in primary deficit is reflective of government’s efforts at bridging the fiscal gap during a financial year. Question 15 of 35 15. Question With reference to Minimum Alternate Tax, consider the following statements: It is a direct tax imposed under Income Tax Act, 1961. It does not apply to any income accruing or arising to a company from life insurance business. It is not applicable to public corporate entities. How many of the statements given above is/are correct? a) Only one b) Only two c) All three d) None Correct Solution (b) Statement 1 Statement 2 Statement 3 Correct Correct Incorrect Minimum Alternate Tax (MAT) is a tax effectively introduced in India by the Finance Act of 1987, vide Section 115J of the Income Tax Act, 1961 (IT Act), to facilitate the taxation of ‘zero tax companies’ i.e., those companies which show zero or negligible income to avoid tax. It is a direct tax. Under MAT, such companies are made liable to pay to the government, by deeming a certain percentage of their book profit as taxable income. However, it does not apply to any income accruing or arising to a company from life insurance business. The applicability of MAT extends to all types of companies, including a private company, one-person company, public limited company and Nidhi company. Incorrect Solution (b) Statement 1 Statement 2 Statement 3 Correct Correct Incorrect Minimum Alternate Tax (MAT) is a tax effectively introduced in India by the Finance Act of 1987, vide Section 115J of the Income Tax Act, 1961 (IT Act), to facilitate the taxation of ‘zero tax companies’ i.e., those companies which show zero or negligible income to avoid tax. It is a direct tax. Under MAT, such companies are made liable to pay to the government, by deeming a certain percentage of their book profit as taxable income. However, it does not apply to any income accruing or arising to a company from life insurance business. The applicability of MAT extends to all types of companies, including a private company, one-person company, public limited company and Nidhi company. Question 16 of 35 16. Question In the context of the Indian Economy, what are Ways and Means advances? a) These are part of the infrastructure projects financing mechanism by the National Investment and Infrastructure Fund (NIIF). b) These are loans advanced by the World Bank to developing economies. c) These are temporary loans advanced by RBI to central and state governments. d) These are loans advanced by the IMF to overcome the medium-term balance of payments crisis. Correct Solution (c) The Reserve Bank of India gives temporary loan facilities to the centre and state governments as a banker to the This temporary loan facility is called Ways and Means Advances (WMA). This facility is provided to help them tide over temporary mismatches in the cash flow of their receipts and This is guided under Section 17(5) of the RBI Act, 1934.   Incorrect Solution (c) The Reserve Bank of India gives temporary loan facilities to the centre and state governments as a banker to the This temporary loan facility is called Ways and Means Advances (WMA). This facility is provided to help them tide over temporary mismatches in the cash flow of their receipts and This is guided under Section 17(5) of the RBI Act, 1934.   Question 17 of 35 17. Question Consider the following statements regarding the differences between Proportional and Progressive Taxation: In progressive taxation, tax rate increases with an increase in the income whereas, in proportional taxation same percentage of tax is levied on all taxpayers. As compared to progressive taxation, proportional taxation makes disposable income less sensitive to fluctuations in GDP. Which of the statements given above is/are correct? a) 1 only b) 2 only c) Both 1 and 2 d) Neither 1 nor 2 Correct Solution (c) Statement 1 Statement 2 Correct Correct A proportional tax is an income tax system where the same percentage of tax is levied on all taxpayers, regardless of their income. A proportional tax applies the same tax rate across low, middle, and high-income taxpayers. Whereas, progressive taxation is based on the taxable amount of an individual’s income. They follow an accelerating schedule, so high-income earners pay more than low-income earners. The proportional income tax acts as an automatic stabiliser – a shock absorber because it makes disposable income, and thus consumer spending, less sensitive to fluctuations in GDP as compared to  progressive taxation. When GDP rises, disposable income also rises but by less than the rise in GDP because a part of it is siphoned off as taxes. This helps limit the upward fluctuation in consumption spending. Incorrect Solution (c) Statement 1 Statement 2 Correct Correct A proportional tax is an income tax system where the same percentage of tax is levied on all taxpayers, regardless of their income. A proportional tax applies the same tax rate across low, middle, and high-income taxpayers. Whereas, progressive taxation is based on the taxable amount of an individual’s income. They follow an accelerating schedule, so high-income earners pay more than low-income earners. The proportional income tax acts as an automatic stabiliser – a shock absorber because it makes disposable income, and thus consumer spending, less sensitive to fluctuations in GDP as compared to  progressive taxation. When GDP rises, disposable income also rises but by less than the rise in GDP because a part of it is siphoned off as taxes. This helps limit the upward fluctuation in consumption spending. Question 18 of 35 18. Question Which of the following are the quantitative methods of credit control in India? Regulation of consumer credit Rationing Credit Discount Rate Policy Open Market Operations Select the correct answer using the code given below: a) 1 and 2 only b) 1, 2 and 3 only c) 3 and 4 only d) 1, 2, 3 and 4 Correct Solution (c) Statement 1 Statement 2 Statement 3 Statement 4 Incorrect Incorrect Correct Correct Regulation of consumer credit is a Qualitative method of money control. Rationing Credit credit is a Qualitative method of money control. Discount Rate Policy points to the rate of interest that is going to be paid by the banks and other financial institutions on the credits that they avail from the central bank through discount window loan procedure. This will infuse more liquidity in the market. Open Market Operations refers to buying and selling of bonds issued by the Government in the open market. When thes central bank want to infuse liquidity in the monetary system, it will buy government securities in the open market. Incorrect Solution (c) Statement 1 Statement 2 Statement 3 Statement 4 Incorrect Incorrect Correct Correct Regulation of consumer credit is a Qualitative method of money control. Rationing Credit credit is a Qualitative method of money control. Discount Rate Policy points to the rate of interest that is going to be paid by the banks and other financial institutions on the credits that they avail from the central bank through discount window loan procedure. This will infuse more liquidity in the market. Open Market Operations refers to buying and selling of bonds issued by the Government in the open market. When thes central bank want to infuse liquidity in the monetary system, it will buy government securities in the open market. Question 19 of 35 19. Question Which of the following was/were the financial sector reforms taken up by the Government of India post the economic reforms of 1991? Allowing Foreign Institutional Investors to invest in Indian financial markets. Necessary approval of RBI needed for all banks to set up new branches. Banks were allowed to generate resources from India and abroad. How many of the statements given above is/are correct? a) Only one b) Only two c) All three d) None Correct Solution (b) Statement 1 Statement 2 Statement 3 Correct Incorrect Correct Foreign Institutional Investors (FII), such as merchant bankers, mutual funds and pension funds, are allowed to invest in Indian financial markets. Those banks which fulfill certain conditions have been given freedom to set up new branches without the approval of the RBI and rationalize their existing branch networks.   Though banks have been given permission to generate resources from India and abroad, certain managerial aspects have been retained with the RBI to safeguard the interests of the account-holders and the nation. Incorrect Solution (b) Statement 1 Statement 2 Statement 3 Correct Incorrect Correct Foreign Institutional Investors (FII), such as merchant bankers, mutual funds and pension funds, are allowed to invest in Indian financial markets. Those banks which fulfill certain conditions have been given freedom to set up new branches without the approval of the RBI and rationalize their existing branch networks.   Though banks have been given permission to generate resources from India and abroad, certain managerial aspects have been retained with the RBI to safeguard the interests of the account-holders and the nation. Question 20 of 35 20. Question Which of the following category of people are exempted from income tax? a) Income of a member of a Scheduled Tribes who resides in any area in the State of Nagaland. b) Members of the Scheduled castes who resides in any area in the State of Rajasthan. c) Income of a member of a Scheduled Tribes who resides in any area in the State of Assam. d) Members of the Scheduled Tribes in any area in State of Himachal Pradesh. Correct Solution (a) Income of a member of a Scheduled Tribe [as per article 366(25) of the Constitution] is exempt from tax, if following conditions are satisfied: Such member resides in any area in the State of Nagaland, Manipur, Tripura, Arunachal Pradesh, Mizoram or district of North Cachar Hills, Mikir Hills, Khasi Hills, Jaintia Hills and Garo Hills or in the Ladakh region. Such exemption is available in respect of income which accrues/arises from any source in such areas or income by way of dividends/interest on securities arises from any area.   Incorrect Solution (a) Income of a member of a Scheduled Tribe [as per article 366(25) of the Constitution] is exempt from tax, if following conditions are satisfied: Such member resides in any area in the State of Nagaland, Manipur, Tripura, Arunachal Pradesh, Mizoram or district of North Cachar Hills, Mikir Hills, Khasi Hills, Jaintia Hills and Garo Hills or in the Ladakh region. Such exemption is available in respect of income which accrues/arises from any source in such areas or income by way of dividends/interest on securities arises from any area.   Question 21 of 35 21. Question Consider the following statements about Anthrax: It is a highly infectious disease that is caused by Bacillus anthracis. It affects animals like cows, sheep, goats, and wild herbivores. It can infect humans if they come in contact with infected animals. It can be treated with antibiotics such as ciprofloxacin, doxycycline, or levofloxacin. Choose the correct code: a) Only one b) Only two c) Only three d) All four Correct Solution (d) Anthrax is a highly infectious disease that is caused by Bacillus anthracis. Hence statement 1 is correct. Anthrax bacteria also occur naturally in soil. It affects animals like cows, sheep, goats, and wild herbivores. Hence statement 2 is correct. The disease manifests in three forms depending on the route of infection – cutaneous, gastrointestinal, and inhalational. It can infect humans if they come in contact with infected animals. Hence statement 3 is correct. It can be diagnosed by identifying Bacillus anthracis in blood, skin lesions, or respiratory secretions through laboratory culture, PCR, or ELISA tests. It can be treated with antibiotics such as ciprofloxacin, doxycycline, or levofloxacin. Hence statement 4 is correct. Vaccines are also available for both livestock and humans. Incorrect Solution (d) Anthrax is a highly infectious disease that is caused by Bacillus anthracis. Hence statement 1 is correct. Anthrax bacteria also occur naturally in soil. It affects animals like cows, sheep, goats, and wild herbivores. Hence statement 2 is correct. The disease manifests in three forms depending on the route of infection – cutaneous, gastrointestinal, and inhalational. It can infect humans if they come in contact with infected animals. Hence statement 3 is correct. It can be diagnosed by identifying Bacillus anthracis in blood, skin lesions, or respiratory secretions through laboratory culture, PCR, or ELISA tests. It can be treated with antibiotics such as ciprofloxacin, doxycycline, or levofloxacin. Hence statement 4 is correct. Vaccines are also available for both livestock and humans. Question 22 of 35 22. Question It is located in the state of Tamil Nadu. It lies adjacent to the Bay of Bengal along the east coast. It is one of the major wetlands on the Coromandel Coast after Pulicat Lake. The southern part of the wetland has been reserved land since 2001. It is listed as one of Tamil Nadu’s 141 prioritised wetlands, Kazhuveli is also a wetland of international significance and a potential Ramsar site. It has a feeding ground for long-distance migrants from the cold subarctic regions of Central Asia and Siberia including Black-tailed Godwits, Eurasian Curlew, White Stork, Ruff, and Dunlin. The above paragraph refers to which of the following? a) Kazhuveli Bird Sanctuary b) Vedanthangal Bird Sanctuary c) Vettangudi Bird Sanctuary d) Kanjirankulam Bird Sanctuary Correct Solution (a) Kazhuveli Bird Sanctuary is located in the state of Tamil Nadu. It lies adjacent to the Bay of Bengal along the east coast. It is one of the major wetlands on the Coromandel Coast after Pulicat Lake. The southern part of the wetland has been reserved land since 2001. It is listed as one of Tamil Nadu’s 141 prioritised wetlands, Kazhuveli is also a wetland of international significance and a potential Ramsar site. It has a feeding ground for long-distance migrants from the cold subarctic regions of Central Asia and Siberia including Black-tailed Godwits, Eurasian Curlew, White Stork, Ruff, and Dunlin. Hence option a is correct.   Incorrect Solution (a) Kazhuveli Bird Sanctuary is located in the state of Tamil Nadu. It lies adjacent to the Bay of Bengal along the east coast. It is one of the major wetlands on the Coromandel Coast after Pulicat Lake. The southern part of the wetland has been reserved land since 2001. It is listed as one of Tamil Nadu’s 141 prioritised wetlands, Kazhuveli is also a wetland of international significance and a potential Ramsar site. It has a feeding ground for long-distance migrants from the cold subarctic regions of Central Asia and Siberia including Black-tailed Godwits, Eurasian Curlew, White Stork, Ruff, and Dunlin. Hence option a is correct.   Question 23 of 35 23. Question Consider the following statements about the Directorate of Revenue Intelligence: It is the premier intelligence and enforcement agency on anti-smuggling matters. It works under the aegis of the Central Bureau of Investigation (CBI). It is headquartered in New Delhi. How many of the above statements are correct? a) Only one b) Only two c) All three d) None Correct Solution (b) The Directorate of Revenue Intelligence is the premier intelligence and enforcement agency on anti-smuggling matters. Hence statement 1 is correct. It has been carrying out its mandate of preventing and detecting cases of smuggling of narcotic drugs & psychotropic substances, wildlife items, notes, foreign currency, hazardous & environmentally sensitive materials, antiques, etc., and taking punitive action against the organised crime groups engaged therein. It works under the aegis of the Central Board of Indirect Taxes & Customs (CBIC), Government of India. Hence statement 2 is incorrect. It is also engaged in unearthing commercial frauds and instances of customs duty evasion. It is headquartered in New Delhi. Hence statement 3 is correct. Incorrect Solution (b) The Directorate of Revenue Intelligence is the premier intelligence and enforcement agency on anti-smuggling matters. Hence statement 1 is correct. It has been carrying out its mandate of preventing and detecting cases of smuggling of narcotic drugs & psychotropic substances, wildlife items, notes, foreign currency, hazardous & environmentally sensitive materials, antiques, etc., and taking punitive action against the organised crime groups engaged therein. It works under the aegis of the Central Board of Indirect Taxes & Customs (CBIC), Government of India. Hence statement 2 is incorrect. It is also engaged in unearthing commercial frauds and instances of customs duty evasion. It is headquartered in New Delhi. Hence statement 3 is correct. Question 24 of 35 24. Question Consider the following statements about Pompe Disease: It is caused due to mutations in the GAA gene. It affects males and females equally. It is treated through enzyme replacement therapy (ERT). How many of the above statements are correct? a) Only one b) Only two c) All three d) None Correct Solution (c) Pompe Disease is caused due to mutations in the GAA gene. Hence statement 1 is correct. The GAA gene provides instructions for producing an enzyme called acid alpha-glucosidase (also known as acid maltase). This enzyme is active in lysosomes, which are structures that serve as recycling centers within cells. The enzyme normally breaks down glycogen into a simpler sugar called glucose, which is the main energy source for most cells. Mutations in the GAA gene prevent acid alpha-glucosidase from breaking down glycogen effectively, which allows this sugar to build up to toxic levels in lysosomes. This buildup damages organs and tissues throughout the body, particularly the muscles, leading to the progressive signs and symptoms of Pompe disease. It affects males and females equally. Hence statement 2 is correct. Some common side effects and symptoms include muscle weakness, respiratory issues, heart problems, and difficulty swallowing. It is treated through enzyme replacement therapy (ERT). Hence statement 3 is correct. Incorrect Solution (c) Pompe Disease is caused due to mutations in the GAA gene. Hence statement 1 is correct. The GAA gene provides instructions for producing an enzyme called acid alpha-glucosidase (also known as acid maltase). This enzyme is active in lysosomes, which are structures that serve as recycling centers within cells. The enzyme normally breaks down glycogen into a simpler sugar called glucose, which is the main energy source for most cells. Mutations in the GAA gene prevent acid alpha-glucosidase from breaking down glycogen effectively, which allows this sugar to build up to toxic levels in lysosomes. This buildup damages organs and tissues throughout the body, particularly the muscles, leading to the progressive signs and symptoms of Pompe disease. It affects males and females equally. Hence statement 2 is correct. Some common side effects and symptoms include muscle weakness, respiratory issues, heart problems, and difficulty swallowing. It is treated through enzyme replacement therapy (ERT). Hence statement 3 is correct. Question 25 of 35 25. Question Consider the following statements: The Doctrine of Laches states that the court will only assist those individuals who are vigilant about their rights and not those who are negligent. The Doctrine of Eclipse asserts that a law that infringes fundamental rights is not null or void from the outset, but is merely non-enforceable. The Doctrine of Territorial Nexus dictates that laws made by a state legislature are not applicable outside that state unless there is a sufficient nexus between the state and the object. The Doctrine of Casus Omissus asserts that a matter which should have been provided for in a statute cannot be supplied by the courts. How many of the above statements are correct? a) Only one b) Only two c) Only three d) All four Correct Solution (d) The Doctrine of Laches states that the court will only assist those individuals who are vigilant about their rights and not those who are negligent. Hence statement 1 is correct. The Doctrine of Eclipse asserts that a law that infringes fundamental rights is not null or void from the outset, but is merely non-enforceable. Hence statement 2 is correct. The Doctrine of Territorial Nexus dictates that laws made by a state legislature are not applicable outside that state unless there is a sufficient nexus between the state and the object. Hence statement 3 is correct. The Doctrine of Casus Omissus asserts that a matter which should have been provided for in a statute cannot be supplied by the courts. Hence statement 4 is correct. The Doctrine of Promissory Estoppel is a legal principle that prevents a person from retracting a promise they made, even if the promise was not supported by a contract. The Doctrine of Colourable Legislation is a principle used to determine the legislative competence of laws enacted by various legislatures. The Doctrine of Harmonious Construction posits that a provision of the statute should not be interpreted in isolation but as a whole, to eliminate any inconsistency or repugnancy. The Doctrine of Incidental or Ancillary Powers suggests that the power to legislate on a particular issue also includes the power to legislate on ancillary matters that are reasonably connected to that issue. The Doctrine of Waiver implies that a person can intentionally relinquish their right or privilege or choose not to exercise their right or privilege conferred on them by the state Incorrect Solution (d) The Doctrine of Laches states that the court will only assist those individuals who are vigilant about their rights and not those who are negligent. Hence statement 1 is correct. The Doctrine of Eclipse asserts that a law that infringes fundamental rights is not null or void from the outset, but is merely non-enforceable. Hence statement 2 is correct. The Doctrine of Territorial Nexus dictates that laws made by a state legislature are not applicable outside that state unless there is a sufficient nexus between the state and the object. Hence statement 3 is correct. The Doctrine of Casus Omissus asserts that a matter which should have been provided for in a statute cannot be supplied by the courts. Hence statement 4 is correct. The Doctrine of Promissory Estoppel is a legal principle that prevents a person from retracting a promise they made, even if the promise was not supported by a contract. The Doctrine of Colourable Legislation is a principle used to determine the legislative competence of laws enacted by various legislatures. The Doctrine of Harmonious Construction posits that a provision of the statute should not be interpreted in isolation but as a whole, to eliminate any inconsistency or repugnancy. The Doctrine of Incidental or Ancillary Powers suggests that the power to legislate on a particular issue also includes the power to legislate on ancillary matters that are reasonably connected to that issue. The Doctrine of Waiver implies that a person can intentionally relinquish their right or privilege or choose not to exercise their right or privilege conferred on them by the state Question 26 of 35 26. Question Consider the following statements about the European Free Trade Association (EFTA): It is an intergovernmental organization established by the Washington Convention. It promotes free trade and economic integration between its members within Europe and globally. It currently has four member countries – Iceland, Liechtenstein, Norway, and Switzerland. How many of the above statements are correct? a) Only one b) Only two c) All three d) None Correct Solution (b) The European Free Trade Association (EFTA) is an intergovernmental organization established by the Stockholm Convention. Hence statement 1 is incorrect. EFTA is not a customs union because the individual EFTA States are free to set their own customs tariffs and arrange other foreign trade measures vis-à-vis the non-EFTA States. It promotes free trade and economic integration between its members within Europe and globally. Hence statement 2 is correct. The EFTA countries have developed one of the largest networks of Free Trade Agreements (FTAs). These FTAs span over 60 countries and territories, including the EU. The EFTA members are all open, competitive economies committed to the progressive liberalization of trade in the multinational arena as well as in free trade agreements. It currently has four member countries – Iceland, Liechtenstein, Norway, and Switzerland. Hence statement 3 is correct.   Note: Customs unions are groups of countries that apply one common system of procedures, rules, and tariffs for all or almost all their imports, exports, and transit goods. Usually, countries participating in customs unions share common trade and competition policies.   Incorrect Solution (b) The European Free Trade Association (EFTA) is an intergovernmental organization established by the Stockholm Convention. Hence statement 1 is incorrect. EFTA is not a customs union because the individual EFTA States are free to set their own customs tariffs and arrange other foreign trade measures vis-à-vis the non-EFTA States. It promotes free trade and economic integration between its members within Europe and globally. Hence statement 2 is correct. The EFTA countries have developed one of the largest networks of Free Trade Agreements (FTAs). These FTAs span over 60 countries and territories, including the EU. The EFTA members are all open, competitive economies committed to the progressive liberalization of trade in the multinational arena as well as in free trade agreements. It currently has four member countries – Iceland, Liechtenstein, Norway, and Switzerland. Hence statement 3 is correct.   Note: Customs unions are groups of countries that apply one common system of procedures, rules, and tariffs for all or almost all their imports, exports, and transit goods. Usually, countries participating in customs unions share common trade and competition policies.   Question 27 of 35 27. Question Consider the following statements about Exchange-Traded Funds (ETFs): It is a collection of marketable securities that tracks an index, a commodity, bonds, or a basket of assets. It can be purchased or sold on a stock exchange in the same way that regular stocks can. Choose the correct code: a) 1 only b) 2 only c) Both 1 and 2 d) Neither 1 nor 2 Correct Solution (c) Exchange-traded funds (ETFs) are a collection of marketable securities that track an index, a commodity, bonds, or a basket of assets. Hence statement 1 is correct. It can be structured to track anything from the price of an individual commodity to a large and diverse collection of securities. ETFs can even be structured to track specific investment strategies. It can be purchased or sold on a stock exchange in the same way that regular stocks can. Hence statement 2 is correct. The traded price of an ETF changes throughout the day like any other stock, as it is bought and sold on the stock exchange. The trading value of an ETF is based on the net asset value of the underlying stocks that it represents. Incorrect Solution (c) Exchange-traded funds (ETFs) are a collection of marketable securities that track an index, a commodity, bonds, or a basket of assets. Hence statement 1 is correct. It can be structured to track anything from the price of an individual commodity to a large and diverse collection of securities. ETFs can even be structured to track specific investment strategies. It can be purchased or sold on a stock exchange in the same way that regular stocks can. Hence statement 2 is correct. The traded price of an ETF changes throughout the day like any other stock, as it is bought and sold on the stock exchange. The trading value of an ETF is based on the net asset value of the underlying stocks that it represents. Question 28 of 35 28. Question Consider the following statements: The Global Cooling Watch Report was released by the UN Environment Programme (UNEP)-led Cool Coalition. The Global Cooling Pledge was signed at COP25 to address the growing demand for cooling while mitigating its climate impacts. Choose the correct code: a) 1 only b) 2 only c) Both 1 and 2 d) Neither 1 nor 2 Correct Solution (a) The Global Cooling Watch Report was released by the UN Environment Programme (UNEP)-led Cool Coalition. Hence statement 1 is correct. It is a comprehensive analysis of the global cooling sector and focuses not only on space cooling but also on cold chains for food and health. Its key highlights are: Currently, cooling equipment currently accounts for 20% of total electricity consumption and is expected to more than double by 2050. Under a business-as-usual scenario, emissions from cooling are predicted to account for more than 10 percent of global emissions in 2050. The Global Cooling Pledge was signed at COP28 to address the growing demand for cooling while mitigating its climate impacts. Hence statement 2 is incorrect. Around 63 countries have signed the pledge. However, India has not signed it yet. Incorrect Solution (a) The Global Cooling Watch Report was released by the UN Environment Programme (UNEP)-led Cool Coalition. Hence statement 1 is correct. It is a comprehensive analysis of the global cooling sector and focuses not only on space cooling but also on cold chains for food and health. Its key highlights are: Currently, cooling equipment currently accounts for 20% of total electricity consumption and is expected to more than double by 2050. Under a business-as-usual scenario, emissions from cooling are predicted to account for more than 10 percent of global emissions in 2050. The Global Cooling Pledge was signed at COP28 to address the growing demand for cooling while mitigating its climate impacts. Hence statement 2 is incorrect. Around 63 countries have signed the pledge. However, India has not signed it yet. Question 29 of 35 29. Question Consider the following statements: Convertible preference shares are those shares that can be easily converted into equity shares. Non-convertible preference shares are those shares that cannot be converted into equity shares. Redeemable preference shares are those shares that can be repurchased or redeemed by the issuing company at a fixed rate and date. Non-redeemable preference shares are those shares that cannot be redeemed or repurchased by the issuing company at a fixed date. Choose the correct code: a) Only one b) Only two c) Only three d) All four Correct Solution (d) Convertible preference shares are those shares that can be easily converted into equity shares. Hence statement 1 is correct. Non-convertible preference shares are those shares that cannot be converted into equity shares. Hence statement 2 is correct. Redeemable preference shares are those shares that can be repurchased or redeemed by the issuing company at a fixed rate and date. Hence statement 3 is correct. These types of shares help the company by providing a cushion during times of inflation. Non-redeemable preference shares are those shares that cannot be redeemed or repurchased by the issuing company at a fixed date. Hence statement 4 is correct. These types of shares help companies by acting as a lifesaver during times of inflation. Incorrect Solution (d) Convertible preference shares are those shares that can be easily converted into equity shares. Hence statement 1 is correct. Non-convertible preference shares are those shares that cannot be converted into equity shares. Hence statement 2 is correct. Redeemable preference shares are those shares that can be repurchased or redeemed by the issuing company at a fixed rate and date. Hence statement 3 is correct. These types of shares help the company by providing a cushion during times of inflation. Non-redeemable preference shares are those shares that cannot be redeemed or repurchased by the issuing company at a fixed date. Hence statement 4 is correct. These types of shares help companies by acting as a lifesaver during times of inflation. Question 30 of 35 30. Question Consider the following statements about Chousingha: It is endemic to the Indian subcontinent. It is the smallest antelope found in Asia. It is listed as endangered on the IUCN Red List. How many of the above statements are correct? a) Only one b) Only two c) All three d) None Correct Solution (b) Chousingha is endemic to the Indian subcontinent. Hence statement 1 is correct. The four-horned antelope, or chousingha, is a small antelope found in India and Nepal. It is the smallest antelope found in Asia. Hence statement 2 is correct. They are usually diurnal and solitary by nature; however, they can be spotted in loose groups of three to four. Animals are sedentary, inhabiting more or less the same region throughout their lives. It is listed as vulnerable on the IUCN Red List. Hence statement 3 is incorrect. Incorrect Solution (b) Chousingha is endemic to the Indian subcontinent. Hence statement 1 is correct. The four-horned antelope, or chousingha, is a small antelope found in India and Nepal. It is the smallest antelope found in Asia. Hence statement 2 is correct. They are usually diurnal and solitary by nature; however, they can be spotted in loose groups of three to four. Animals are sedentary, inhabiting more or less the same region throughout their lives. It is listed as vulnerable on the IUCN Red List. Hence statement 3 is incorrect. Question 31 of 35 31. Question The diagonal of a rectangle is √21 cm and its area is 50 sq. cm. What is the perimeter of the rectangle? (Note – Diagonal equals the square root of the width squared plus the height squared) a) 11 b) 22 c) 14 d) 28 Correct Solution (b) For a rectangle, d^2 = l^2 + b^2 where l= length, b = breadth and d = diagonal of the of the rectangle   d= √21 d^2 = l^2 + b^2 l^2 + b^2= 21 ….. (1) Area = l × b = 50 …..(2)   We know that (a+b)^2 = a^2 + b^2 + 2ab Using the same formula, we have (l+b)^2 = l^2 + b^2 + 2lb (l+b)^2 = 21 + 2*50 (l+b) = 11 Perimeter of the rectangle = 2(l + b) = 2(11) = 22 cm   Incorrect Solution (b) For a rectangle, d^2 = l^2 + b^2 where l= length, b = breadth and d = diagonal of the of the rectangle   d= √21 d^2 = l^2 + b^2 l^2 + b^2= 21 ….. (1) Area = l × b = 50 …..(2)   We know that (a+b)^2 = a^2 + b^2 + 2ab Using the same formula, we have (l+b)^2 = l^2 + b^2 + 2lb (l+b)^2 = 21 + 2*50 (l+b) = 11 Perimeter of the rectangle = 2(l + b) = 2(11) = 22 cm   Question 32 of 35 32. Question A and B playing a game with rolling a fare dice, one after the other replacing it every time till one of them gets a six. If A begins the game, then the probability that A wins the game is a) 1/6 b) 2/6 c) 5/11 d) 6/11 Correct Solution (c) Let  S  denote  the  success  (getting  a  ‘6’)  and  F  denote  the  failure  (not  getting  a  ‘6’)  . Thus, P(S)= 1/6 ​ =p, P(F)= 5/6​ =q P(A wins  in  first  throw)=P(S)=p P(A  wins  in  third  throw)=P(FFS)=qqp P(A  wins  in  fifth  throw)=P(FFFFS)=qqqqp So,  P(A  wins)=p+qqp+qqqqp+… =p(1+q^2 +q^4 +…) = 1−q^2 p ​ = (1/6)/(1− 25/36) ​ ​ =  6/11 ​ P(B  wins)=1–P(A  wins) P(B  wins)  =1− 6/11 ​ = 5/11 ​ So,  P(A  wins)= 6/11 ​ ,  P(B  wins)= 5/11 Incorrect Solution (c) Let  S  denote  the  success  (getting  a  ‘6’)  and  F  denote  the  failure  (not  getting  a  ‘6’)  . Thus, P(S)= 1/6 ​ =p, P(F)= 5/6​ =q P(A wins  in  first  throw)=P(S)=p P(A  wins  in  third  throw)=P(FFS)=qqp P(A  wins  in  fifth  throw)=P(FFFFS)=qqqqp So,  P(A  wins)=p+qqp+qqqqp+… =p(1+q^2 +q^4 +…) = 1−q^2 p ​ = (1/6)/(1− 25/36) ​ ​ =  6/11 ​ P(B  wins)=1–P(A  wins) P(B  wins)  =1− 6/11 ​ = 5/11 ​ So,  P(A  wins)= 6/11 ​ ,  P(B  wins)= 5/11 Question 33 of 35 33. Question When a particular positive number is divided by 3, the remainder is 1. If the same number is divided by 5, the remainder is 4. If the difference between the quotients of the division is 3, then find the number. a) 34 b) 19 c) 49 d) 64 Correct Solution (b) Let the quotients when this number is divided by 3 and 5 be x and y respectively. (Note that x will be greater than y as 3 is smaller than 5) Number = 3x + 1 = 5y + 4 Given that, x – y = 3 On solving both equation we get, x = 6, y=3 Thus the number is 5 × 3 + 4= 19. Incorrect Solution (b) Let the quotients when this number is divided by 3 and 5 be x and y respectively. (Note that x will be greater than y as 3 is smaller than 5) Number = 3x + 1 = 5y + 4 Given that, x – y = 3 On solving both equation we get, x = 6, y=3 Thus the number is 5 × 3 + 4= 19. Question 34 of 35 34. Question In 10 years, A will be thrice as old as B was 5 years ago. If A is now 9 years older than B, the present age of B is a) 19 b) 21 c) 17 d) 15 Correct Solution (c) Let B’s present age = x years. Then, A’s present age = (x + 9) years. (x + 9) + 10 = 3(x – 5) => x + 19 = 3x – 15 => x =17 Incorrect Solution (c) Let B’s present age = x years. Then, A’s present age = (x + 9) years. (x + 9) + 10 = 3(x – 5) => x + 19 = 3x – 15 => x =17 Question 35 of 35 35. Question Out of the four annual examinations, each with a total of 100 marks, a student secured average marks of 35%, 65% and 50% in the first, second and third annual examinations. To have an overall average of 60%, how many marks does the student need to secure in the fourth annual examination? a) 60 b) 70 c) 80 d) 90 Correct Solution (d) Let the average marks in the third Annual examination be x. Total marks = (Marks in first + second + third + forth) Annual examination = > 4(60) ( 100/100) = ( 35/100) (100) + ( 65/100) (100) + ( 50/100) (100) + ( x/100) (100) => 4(60) = 35 + 65 +50 + x =>x = 90 So, the student must score 90% in the fourth annual examination to secure 60% overall average. ∴ Average marks in the third annual examination (90/100) × 100 = 90 marks.   Incorrect Solution (d) Let the average marks in the third Annual examination be x. Total marks = (Marks in first + second + third + forth) Annual examination = > 4(60) ( 100/100) = ( 35/100) (100) + ( 65/100) (100) + ( 50/100) (100) + ( x/100) (100) => 4(60) = 35 + 65 +50 + x =>x = 90 So, the student must score 90% in the fourth annual examination to secure 60% overall average. ∴ Average marks in the third annual examination (90/100) × 100 = 90 marks.   window.wpProQuizInitList = window.wpProQuizInitList || []; window.wpProQuizInitList.push({ id: '#wpProQuiz_3623', init: { quizId: 3623, mode: 1, globalPoints: 70, timelimit: 1800, resultsGrade: [0], bo: 704, qpp: 0, catPoints: [70], formPos: 0, lbn: "Test-summary", json: {"31922":{"type":"single","id":31922,"catId":0,"points":2,"correct":[0,0,0,1]},"31924":{"type":"single","id":31924,"catId":0,"points":2,"correct":[0,0,0,1]},"31925":{"type":"single","id":31925,"catId":0,"points":2,"correct":[0,1,0,0]},"31927":{"type":"single","id":31927,"catId":0,"points":2,"correct":[0,1,0,0]},"31930":{"type":"single","id":31930,"catId":0,"points":2,"correct":[0,1,0,0]},"31932":{"type":"single","id":31932,"catId":0,"points":2,"correct":[0,0,1,0]},"31934":{"type":"single","id":31934,"catId":0,"points":2,"correct":[0,0,0,1]},"31936":{"type":"single","id":31936,"catId":0,"points":2,"correct":[0,1,0,0]},"31937":{"type":"single","id":31937,"catId":0,"points":2,"correct":[0,1,0,0]},"31939":{"type":"single","id":31939,"catId":0,"points":2,"correct":[0,1,0,0]},"31941":{"type":"single","id":31941,"catId":0,"points":2,"correct":[0,1,0,0]},"31943":{"type":"single","id":31943,"catId":0,"points":2,"correct":[0,1,0,0]},"31945":{"type":"single","id":31945,"catId":0,"points":2,"correct":[0,1,0,0]},"31948":{"type":"single","id":31948,"catId":0,"points":2,"correct":[0,0,0,1]},"31950":{"type":"single","id":31950,"catId":0,"points":2,"correct":[0,1,0,0]},"31952":{"type":"single","id":31952,"catId":0,"points":2,"correct":[0,0,1,0]},"31955":{"type":"single","id":31955,"catId":0,"points":2,"correct":[0,0,1,0]},"31957":{"type":"single","id":31957,"catId":0,"points":2,"correct":[0,0,1,0]},"31960":{"type":"single","id":31960,"catId":0,"points":2,"correct":[0,1,0,0]},"31961":{"type":"single","id":31961,"catId":0,"points":2,"correct":[1,0,0,0]},"31963":{"type":"single","id":31963,"catId":0,"points":2,"correct":[0,0,0,1]},"31966":{"type":"single","id":31966,"catId":0,"points":2,"correct":[1,0,0,0]},"31968":{"type":"single","id":31968,"catId":0,"points":2,"correct":[0,1,0,0]},"31969":{"type":"single","id":31969,"catId":0,"points":2,"correct":[0,0,1,0]},"31970":{"type":"single","id":31970,"catId":0,"points":2,"correct":[0,0,0,1]},"31972":{"type":"single","id":31972,"catId":0,"points":2,"correct":[0,1,0,0]},"31975":{"type":"single","id":31975,"catId":0,"points":2,"correct":[0,0,1,0]},"31976":{"type":"single","id":31976,"catId":0,"points":2,"correct":[1,0,0,0]},"31979":{"type":"single","id":31979,"catId":0,"points":2,"correct":[0,0,0,1]},"31981":{"type":"single","id":31981,"catId":0,"points":2,"correct":[0,1,0,0]},"31982":{"type":"single","id":31982,"catId":0,"points":2,"correct":[0,1,0,0]},"31984":{"type":"single","id":31984,"catId":0,"points":2,"correct":[0,0,1,0]},"31986":{"type":"single","id":31986,"catId":0,"points":2,"correct":[0,1,0,0]},"31988":{"type":"single","id":31988,"catId":0,"points":2,"correct":[0,0,1,0]},"31991":{"type":"single","id":31991,"catId":0,"points":2,"correct":[0,0,0,1]}} } }); All the Best IASbaba

Daily Prelims CA Quiz

UPSC Quiz – 2024 : IASbaba’s Daily Current Affairs Quiz 9th April 2024

For Previous Daily Quiz (ARCHIVES) – CLICK HERE The Current Affairs questions are based on sources like ‘The Hindu’, ‘Indian Express’ and ‘PIB’, which are very important sources for UPSC Prelims Exam. The questions are focused on both the concepts and facts. The topics covered here are generally different from what is being covered under ‘Daily Current Affairs/Daily News Analysis (DNA) and Daily Static Quiz’ to avoid duplication. The questions would be published from Monday to Saturday before 2 PM. One should not spend more than 10 minutes on this initiative. Gear up and Make the Best Use of this initiative. Do remember that, “the difference between Ordinary and EXTRA-Ordinary is PRACTICE!!” Important Note: Don’t forget to post your marks in the comment section. Also, let us know if you enjoyed today’s test 🙂After completing the 5 questions, click on ‘View Questions’ to check your score, time taken, and solutions.To take the Test Click Here

[DAY 31] 60 DAY RAPID REVISION (RaRe) SERIES for UPSC Prelims 2024 – GEOGRAPHY, CURRENT AFFAIRS & CSAT TEST SERIES!

Archives Hello Friends The 60 Days Rapid Revision (RaRe) Series is IASbaba’s Flagship Initiative recommended by Toppers and loved by the aspirants’ community every year. It is the most comprehensive program which will help you complete the syllabus, revise and practice tests on a daily basis. The Programme on a daily basis includes Daily Prelims MCQs from Static (Monday – Saturday) Daily Static Quiz will cover all the topics of static subjects – Polity, History, Geography, Economics, Environment and Science and technology. 20 questions will be posted daily and these questions are framed from the topics mentioned in the schedule. It will ensure timely and streamlined revision of your static subjects. Daily Current Affairs MCQs (Monday – Saturday) Daily 5 Current Affairs questions, based on sources like ‘The Hindu’, ‘Indian Express’ and ‘PIB’, would be published from Monday to Saturday according to the schedule. Daily CSAT Quiz (Monday – Friday) CSAT has been an Achilles heel for many aspirants. Daily 5 CSAT Questions will be published. Note – Daily Test of 20 static questions, 10 current affairs, and 5 CSAT questions. (35 Prelims Questions) in QUIZ FORMAT will be updated on a daily basis. To Know More about 60 Days Rapid Revision (RaRe) Series – CLICK HERE   60 Day Rapid Revision (RaRe) Series Schedule – CLICK HERE  Important Note Comment your Scores in the Comment Section. This will keep you accountable, responsible and sincere in days to come. It will help us come out with the Cut-Off on a Daily Basis. Let us know if you enjoyed today’s test 🙂  You can post your comments in the given format  (1) Your Score (2) Matrix Meter (3) New Learning from the Test Time limit: 0 Test-summary 0 of 35 questions completed Questions: 1 2 3 4 5 6 7 8 9 10 11 12 13 14 15 16 17 18 19 20 21 22 23 24 25 26 27 28 29 30 31 32 33 34 35 Information The following Test is based on the syllabus of 60 Days Plan-2023 for UPSC IAS Prelims 2022. To view Solutions, follow these instructions: Click on – ‘Start Test’ button Solve Questions Click on ‘Test Summary’ button Click on ‘Finish Test’ button Now click on ‘View Questions’ button – here you will see solutions and links. You have already completed the test before. Hence you can not start it again. Test is loading... You must sign in or sign up to start the test. You have to finish following test, to start this test: Results 0 of 35 questions answered correctly Your time: Time has elapsed You have scored 0 points out of 0 points, (0) Average score     Your score     Categories Not categorized 0% Your result has been entered into leaderboard Loading Name: E-Mail: Captcha: maximum of 70 points Pos. Name Entered on Points Result Table is loading No data available 1 2 3 4 5 6 7 8 9 10 11 12 13 14 15 16 17 18 19 20 21 22 23 24 25 26 27 28 29 30 31 32 33 34 35 Answered Review Question 1 of 35 1. Question Consider the following: Its colour varies from light grey to ash grey. It is rich in potash nut poor in phosphorus. It is present in Lower Ganga valley Which of the following soils satisfies the above features? a) Alluvial soil b) Laterite soil c) Marshy Soil d) Red Soil Correct Solution (a) Alluvial soil This soil is well-drained and poorly drained with an immature profile in undulating areas. This soil has a phosphorous deficiency and is rich in potash and lime. It is present in Lower Ganga valley. The colour of soil varies from light grey to ash grey. This soil is suited for Rice, maize, wheat, sugarcane, oilseeds, etc. Incorrect Solution (a) Alluvial soil This soil is well-drained and poorly drained with an immature profile in undulating areas. This soil has a phosphorous deficiency and is rich in potash and lime. It is present in Lower Ganga valley. The colour of soil varies from light grey to ash grey. This soil is suited for Rice, maize, wheat, sugarcane, oilseeds, etc. Question 2 of 35 2. Question Consider the following statements They are formed as the result of intense leaching due to tropical rains. They are poor in organic matter, nitrogen, phosphate and calcium. They are widely used for bricks making which is used for house construction. The humus content is removed by bacteria. Which of the following Soil best suits the description given in the above statements? a) Alluvial Soil b) Arid Soil c) Forest Soil d) Laterite Soil Correct Solution (d) Laterite Soil They are formed as the result of intense leaching due to tropical rains. It is well developed in areas with high temperature and high rainfall. They are poor in organic matter, nitrogen, phosphate and calcium. They are rich in potash and iron oxide. They are widely used for bricks making which is used for house construction. The humus content is removed by bacteria. With the application of manures and fertilizers it can be made use for cultivation. Incorrect Solution (d) Laterite Soil They are formed as the result of intense leaching due to tropical rains. It is well developed in areas with high temperature and high rainfall. They are poor in organic matter, nitrogen, phosphate and calcium. They are rich in potash and iron oxide. They are widely used for bricks making which is used for house construction. The humus content is removed by bacteria. With the application of manures and fertilizers it can be made use for cultivation. Question 3 of 35 3. Question Arrange the following soils in descending order of their prevalence in the country. Laterite soil Alluvial soil Black soil Select the correct answer using the code given below. a) 3-2-1 b) 2-3-1 c) 3-1-2 d) 2-1-3 Correct Solution (b) Alluvial soils are by far the largest and the most important soil group of India. Covering about 15 lakh sq km or about 45.6 per cent of the total land area of the country. Black soils are spread over 5.46 lakh sq km i.e. 16.6 per cent of the total geographical area of the country. Laterite and lateritic soils are widely spread in India and cover an area of 2.48 lakh sq km. Incorrect Solution (b) Alluvial soils are by far the largest and the most important soil group of India. Covering about 15 lakh sq km or about 45.6 per cent of the total land area of the country. Black soils are spread over 5.46 lakh sq km i.e. 16.6 per cent of the total geographical area of the country. Laterite and lateritic soils are widely spread in India and cover an area of 2.48 lakh sq km. Question 4 of 35 4. Question Consider the following statements The Gondwana rocks are young metamorphic rocks. The Cuddapah rocks are ancient igneous rocks. Choose the correct code: a) 1 only b) 2 only c) Both 1 and 2 d) Neither 1 nor 2 Correct Solution (d) Statement 1 Statement 2 Incorrect Incorrect The Gondwana rocks are young sedimentary rocks. After the process of weathering they give rise to comparatively less mature soils. The soil found here is more or less of uniform character but of low fertility. The Cuddapah rocks are ancient sedimentary rocks. After the process of weathering, they give calcareous and argillaceous soils. The soil found here is mostly devoid of metalliferous minerals. Incorrect Solution (d) Statement 1 Statement 2 Incorrect Incorrect The Gondwana rocks are young sedimentary rocks. After the process of weathering they give rise to comparatively less mature soils. The soil found here is more or less of uniform character but of low fertility. The Cuddapah rocks are ancient sedimentary rocks. After the process of weathering, they give calcareous and argillaceous soils. The soil found here is mostly devoid of metalliferous minerals. Question 5 of 35 5. Question Consider the following statements about a Soil type: It swells and become sticky when wet and shrinks when dried. It retains moisture for a very long time. It is covers most of the Deccan Plateau. Identify the soil type based on above characteristics. a) Laterite soil b) Alluvial Soil c) Red Soil d) Black Soil Correct Solution (d) Black soil covers most of the Deccan Plateau and the upper reaches of the Godavari and the Krishna. It is also known as Regur Soil or Black Cotton Soil. It is clayey, deep and impermeable. It swells and becomes sticky when wet and shrink when dried. It has characteristics of ‘self-ploughing’. The soil retains the moisture for a very long time. They are rich in lime, iron, magnesia and alumina. Incorrect Solution (d) Black soil covers most of the Deccan Plateau and the upper reaches of the Godavari and the Krishna. It is also known as Regur Soil or Black Cotton Soil. It is clayey, deep and impermeable. It swells and becomes sticky when wet and shrink when dried. It has characteristics of ‘self-ploughing’. The soil retains the moisture for a very long time. They are rich in lime, iron, magnesia and alumina. Question 6 of 35 6. Question Consider the following pairs: Soil Layer Region 1.  Horizon A Topmost Zone, where organic materials get incorporated with the mineral matter 2.  Horizon E Bedrock of the soil 3.  Horizon C Loose/ weathered parent material. Which of the pairs given above are correctly matched? a) 1 and 3 only b) 2 and 3 only c) 1 and 2 only d) 1, 2 and 3 Correct Solution (a) A (topsoil): Mostly minerals from parent material with organic matter incorporated. A good material for plants and other organisms to live. E (eluviated): Leached of clay, minerals, and organic matter, leaving a concentration of sand and silt particles of quartz or other resistant materials – missing in some soils but often found in older soils and forest soils. B (subsoil): Rich in minerals that leached (moved down) from the A or E horizons and accumulated here. C (parent material): The deposit at Earth’s surface from which the soil developed. R (bedrock): A mass of rock such as granite, basalt, quartzite, limestone or sandstone that forms the parent material for some soils – if the bedrock is close enough to the surface to weather. This is not soil and is located under the C horizon. Incorrect Solution (a) A (topsoil): Mostly minerals from parent material with organic matter incorporated. A good material for plants and other organisms to live. E (eluviated): Leached of clay, minerals, and organic matter, leaving a concentration of sand and silt particles of quartz or other resistant materials – missing in some soils but often found in older soils and forest soils. B (subsoil): Rich in minerals that leached (moved down) from the A or E horizons and accumulated here. C (parent material): The deposit at Earth’s surface from which the soil developed. R (bedrock): A mass of rock such as granite, basalt, quartzite, limestone or sandstone that forms the parent material for some soils – if the bedrock is close enough to the surface to weather. This is not soil and is located under the C horizon. Question 7 of 35 7. Question Which of the below given pairs is/are correctly matched? Soil                                                 Properties Red soil                                                 Calcareous Alkali Soil                                               pH – 9 Black soil                                           rich in Kaolinite Laterite soil                                       rich in iron and aluminium Choose the correct code: a) Only one b) Only two c) Only three d) All four Correct Solution (b) Match 1 Match 2 Match 3 Match 4 Incorrect Correct Incorrect Correct Red soils are red in color and are due to presence of iron oxide and are not Calcareous. They are acidic mainly due to the parent rock. Alkali, or Alkaline, soils are clay soils with high pH (greater than 8.5), a poor soil structure and a low infiltration capacity. Often they have a hard calcareous layer at 0.5 to 1 metre depth. Kaolinite is a clay mineral is the weathering product of feldspars Laterite is both a soil and a rock type rich in iron and aluminium and is commonly considered to have formed in hot and wet tropical areas. Incorrect Solution (b) Match 1 Match 2 Match 3 Match 4 Incorrect Correct Incorrect Correct Red soils are red in color and are due to presence of iron oxide and are not Calcareous. They are acidic mainly due to the parent rock. Alkali, or Alkaline, soils are clay soils with high pH (greater than 8.5), a poor soil structure and a low infiltration capacity. Often they have a hard calcareous layer at 0.5 to 1 metre depth. Kaolinite is a clay mineral is the weathering product of feldspars Laterite is both a soil and a rock type rich in iron and aluminium and is commonly considered to have formed in hot and wet tropical areas. Question 8 of 35 8. Question Consider the following statements: In India red soil and alluvial soil consist for more than 50% of the soil in the country. Red and lateritic soil is mainly found in North India. Alluvial Soil is found in abundance at the upper banks of Cauvery. How many of the above statements are correct? a) Only one b) Only two c) All three d) None Correct Solution (a) Statement 1 Statement 2 Statement 3 Correct Incorrect Incorrect In India red soil and alluvial soil consist for more than 50% of the soil in the country. Alluvial Soil – 40% Red Soil – 18% Red is mainly found in eastern and southern parts of Deccan plateau, Odisha and Chattisgarh. And the Laterite soil is found in Karnataka, Tamil Nadu, Andhra Pradesh, Kerala and Madhya Pradesh. Alluvial Soil is found in abundance in Ganga – Brahmaputra plains. Incorrect Solution (a) Statement 1 Statement 2 Statement 3 Correct Incorrect Incorrect In India red soil and alluvial soil consist for more than 50% of the soil in the country. Alluvial Soil – 40% Red Soil – 18% Red is mainly found in eastern and southern parts of Deccan plateau, Odisha and Chattisgarh. And the Laterite soil is found in Karnataka, Tamil Nadu, Andhra Pradesh, Kerala and Madhya Pradesh. Alluvial Soil is found in abundance in Ganga – Brahmaputra plains. Question 9 of 35 9. Question Which of the following statements is/are correct regarding red soils? Red soil occurs mostly in eastern and southern part of Deccan Plateau. It is red in colour because of the presence of copper oxides. It looks yellow when it occurs in a hydrated form. How many of the above statements are correct? a) Only one b) Only two c) All three d) None Correct Solution (b) Statement 1 Statement 2 Statement 3 Correct Incorrect Correct Red soil occurs mostly in eastern and southern part of Deccan Plateau. It is also found in parts of Odisha and Chhattisgarh. It is red in colour because of the presence of iron oxides. It looks yellow when it occurs in a hydrated form. Incorrect Solution (b) Statement 1 Statement 2 Statement 3 Correct Incorrect Correct Red soil occurs mostly in eastern and southern part of Deccan Plateau. It is also found in parts of Odisha and Chhattisgarh. It is red in colour because of the presence of iron oxides. It looks yellow when it occurs in a hydrated form. Question 10 of 35 10. Question Which of the following are the reasons for division of Indo-Brahma River system into 3 branches today (i.e.Indus, Ganga and Brahmaputra)? The upheaval in the western Himalayas Uplift of the Potwar Plateau Down thrusting of the Malda gap Select the correct statements a) 1 and 2 only b) 2 and 3 only c) 1 and 3 only d) All of the above Correct Solution (d) Statement 1 Statement 2 Statement 3 Correct Correct Correct Pleistocene upheaval in the western Himalayas Uplift of the Potwar Plateau (Delhi Ridge), which acted as the water divide between the Indus and Ganga drainage systems The down thrusting of the Malda gap area between the Rajmahal hills and the Meghalaya plateau during the mid-Pleistocene period, diverted the Ganga and the Brahmaputra systems to flow towards the Bay of Bengal Incorrect Solution (d) Statement 1 Statement 2 Statement 3 Correct Correct Correct Pleistocene upheaval in the western Himalayas Uplift of the Potwar Plateau (Delhi Ridge), which acted as the water divide between the Indus and Ganga drainage systems The down thrusting of the Malda gap area between the Rajmahal hills and the Meghalaya plateau during the mid-Pleistocene period, diverted the Ganga and the Brahmaputra systems to flow towards the Bay of Bengal Question 11 of 35 11. Question Consider the following statements River Luni originates from Aravalli range near Ajmer The water of river Luni is both fresh and saline The Luni River drains in gulf of Khambhat How many of the above statements are correct? a) Only one b) Only two c) All three d) None Correct Solution (b) Statement 1 Statement 2 Statement 3 Correct Correct Incorrect River Luni originates from Naga hills in Aravalli range near Ajmer   The river has fresh water initially but by the time it reaches Barmer, the water starts to turn saline due to rich salt content of the soil. The river Luni ends in marshy lands of Rann of Kutch. Incorrect Solution (b) Statement 1 Statement 2 Statement 3 Correct Correct Incorrect River Luni originates from Naga hills in Aravalli range near Ajmer   The river has fresh water initially but by the time it reaches Barmer, the water starts to turn saline due to rich salt content of the soil. The river Luni ends in marshy lands of Rann of Kutch. Question 12 of 35 12. Question Consider the following statements about Mahadayi/Mandovi river: Kalasa and Banduri are its tributaries. The famous Dudhsagar falls is located on this river. Mahadayi river water dispute is between Karnataka, Goa and Maharashtra. How many of the above statements are correct? a) Only one b) Only two c) All three d) None Correct Solution (c) The Mahadayi/Mandovi River also known as Mahadayi or Mhadei river, is described as the lifeline of the Indian state of Goa. The river has a length of 77 kilometres (48 miles), 29 kilometres (18 miles) in Karnataka and 52 kilometres (32 miles) in Goa. It originates from a cluster of 30 springs at Bhimgad in the Western Ghats in the Belagavi district of Karnataka Dudhsagar falls and Vajrpoha falls located on this river. The Kalasa-Banduri Nala is a project undertaken by the Government of Karnataka. Incorrect Solution (c) The Mahadayi/Mandovi River also known as Mahadayi or Mhadei river, is described as the lifeline of the Indian state of Goa. The river has a length of 77 kilometres (48 miles), 29 kilometres (18 miles) in Karnataka and 52 kilometres (32 miles) in Goa. It originates from a cluster of 30 springs at Bhimgad in the Western Ghats in the Belagavi district of Karnataka Dudhsagar falls and Vajrpoha falls located on this river. The Kalasa-Banduri Nala is a project undertaken by the Government of Karnataka. Question 13 of 35 13. Question West flowing rivers of peninsular India do not form deltas but form only estuaries. Which of the following reasons can be attributed to this phenomenon? Non-perennial flow of water Absence of meanders Passage through hard rocks. How many of the above statements are correct? a) Only one b) Only two c) All three d) None Correct Solution (a) Statement-1 and 2: incorrect Statement-3: correct Absence of meanders and non-perennial flow of water are characteristics of Peninsular rivers, including the east flowing rivers which form deltas. The west flowing rivers of India do not form delta because they passes through hard rocks. This means that these are devoid of sediments which are essential for formation of delta Incorrect Solution (a) Statement-1 and 2: incorrect Statement-3: correct Absence of meanders and non-perennial flow of water are characteristics of Peninsular rivers, including the east flowing rivers which form deltas. The west flowing rivers of India do not form delta because they passes through hard rocks. This means that these are devoid of sediments which are essential for formation of delta Question 14 of 35 14. Question Tel, Ib, Jonk, Ong are tributaries of which of the following river? a) Subarnarekha b) Baitrani c) Damodar d) Mahanadi Correct Solution (d) The Seonath, the Hasdeo, the Mand and the Ib joins Mahanadi from left. The Ong, the Tel and the Jonk joins it from right. Incorrect Solution (d) The Seonath, the Hasdeo, the Mand and the Ib joins Mahanadi from left. The Ong, the Tel and the Jonk joins it from right. Question 15 of 35 15. Question ‘River Yamuna’ passes through which of the following states? Uttarakhand Punjab Uttar Pradesh Haryana Select the correct code: a) 1, 2 and 3 b) 2, 3 and 4 c) 1, 3 and 4 d) 1, 2 and 4 Correct Solution (c) River Yamuna passes through Uttarakhand, Himachal Pradesh, Uttar Pradesh, Haryana, Delhi. Incorrect Solution (c) River Yamuna passes through Uttarakhand, Himachal Pradesh, Uttar Pradesh, Haryana, Delhi. Question 16 of 35 16. Question Which of the following ‘Panch Prayag’s’ are correctly matched? Karnaprayag: Pindar and Alaknanda Rudraprayag: Nandakini and Alaknanda Nandaprayag: Mandakini and Alaknanda Vishnuprayag: Dhauliganga and Alaknanda Choose correct option from below: a) 1 only b) 1 and 2 only c) 1 and 4 only d) 3 and 4 only Correct Solution (c) The five confluences, known as the Panch Prayag, are all along the Alaknanda. They are, in downstream order: Vishnuprayag, where the Dhauliganga joins the Alaknanda; Nandprayag, where the Nandakini joins; Karnaprayag, where the Pindar joins, Rudraprayag, where the Mandakini joins; Devprayag, where the Bhagirathi joins the Alaknanda to form the Ganges River proper. Incorrect Solution (c) The five confluences, known as the Panch Prayag, are all along the Alaknanda. They are, in downstream order: Vishnuprayag, where the Dhauliganga joins the Alaknanda; Nandprayag, where the Nandakini joins; Karnaprayag, where the Pindar joins, Rudraprayag, where the Mandakini joins; Devprayag, where the Bhagirathi joins the Alaknanda to form the Ganges River proper. Question 17 of 35 17. Question With reference to the Peninsular rivers, consider the following statements: They are characterized by trellis and radial drainage patterns. They have larger catchment area than the Himalayan rivers Which of the statements given above is/are correct? a) 1 only b) 2 only c) Both 1 and 2 d) Neither 1 nor 2 Correct Solution (a) Statement-1: Correct Statement-2: Incorrect Peninsular rivers are characterized by trellis, radial and rectangular drainage patterns while Himalayan rivers are characterized by dendritic pattern in plains Himalayan rivers have larger catchment area than the peninsular rivers Incorrect Solution (a) Statement-1: Correct Statement-2: Incorrect Peninsular rivers are characterized by trellis, radial and rectangular drainage patterns while Himalayan rivers are characterized by dendritic pattern in plains Himalayan rivers have larger catchment area than the peninsular rivers Question 18 of 35 18. Question Consider the following statements River Sone has its origin in the Pachmarhi Biosphere Reserve. It flows through Madhya Pradesh and Bihar only. Which of the above statements is/are correct? a) 1 only b) 2 only c) Both 1 and 2 d) Neither 1 nor 2 Correct Solution (d) Statement 1 Statement 2 Incorrect Incorrect The Sone originates near Amarkantak in Anuppur district of Madhya Pradesh, just east of the headwater of the Narmada River, and flows north-northwest through Shahdol district in Madhya Pradesh state before turning sharply eastward where it encounters the southwest-northeast-Kaimur Range. The Sone parallels the Kaimur hills, flowing east-northeast through Uttar Pradesh, Jharkhand and Bihar states to join the Ganges just west of Patna. Incorrect Solution (d) Statement 1 Statement 2 Incorrect Incorrect The Sone originates near Amarkantak in Anuppur district of Madhya Pradesh, just east of the headwater of the Narmada River, and flows north-northwest through Shahdol district in Madhya Pradesh state before turning sharply eastward where it encounters the southwest-northeast-Kaimur Range. The Sone parallels the Kaimur hills, flowing east-northeast through Uttar Pradesh, Jharkhand and Bihar states to join the Ganges just west of Patna. Question 19 of 35 19. Question Consider the following statements about types of Delta Arcuate delta results when light depositions give rise to shallow, shifting distributaries and a general fan-shaped profile. Bird’s Foot Delta emerges when limestone sediment deposits do not allow downward seepage of water. Lobate Delta develops when fluvial action and depositional process dominate the system. How many of the statements given above is/are correct? a) Only one b) Only two c) All three d) None Correct Solution (c) Statement 1 Statement 2 Statement 3 Correct Correct Correct Arcuate or Fan-shaped (Curved) This type of delta results when light depositions give rise to shallow, shifting distributaries and a general fan-shaped profile. Examples: Nile, Ganga, Indus Bird’s Foot Delta (Elongated) This type of delta emerges when limestone sediment deposits do not allow downward seepage of water. The distributaries seem to be flowing over projections of these deposits which appear as a bird’s foot. Example: Mississippi River High-constructive deltas – Elongate and Lobate Delta Develops when fluvial action and depositional process dominate the system. The Godavari River represents lobate delta. Incorrect Solution (c) Statement 1 Statement 2 Statement 3 Correct Correct Correct Arcuate or Fan-shaped (Curved) This type of delta results when light depositions give rise to shallow, shifting distributaries and a general fan-shaped profile. Examples: Nile, Ganga, Indus Bird’s Foot Delta (Elongated) This type of delta emerges when limestone sediment deposits do not allow downward seepage of water. The distributaries seem to be flowing over projections of these deposits which appear as a bird’s foot. Example: Mississippi River High-constructive deltas – Elongate and Lobate Delta Develops when fluvial action and depositional process dominate the system. The Godavari River represents lobate delta. Question 20 of 35 20. Question Which of the above pairs are correctly matched? Rivers                                                Dams The Uri Dam                                        River Indus The Salal Dam                                    River Chenab The Chamera dam                              River Sutlej The Tulbul Project                              River Jhelum Choose the correct code: a) 1 and 3 b) 1, 2, and 3 c) 2 and 4 d) 1, 2, 3 and 4 Correct Solution (c) Match 1 Match 2 Match 3 Match 4 Incorrect Correct Incorrect Correct The Uri Dam– The dam has been constructed on the river Jhelum in the state of Jammu and Kashmir.   The Salal Dam– The dam has been constructed on the river Chenab at Dhyangarh in the state of Jammu and Kashmir.   The Chamera Dam– The dam has been constructed on the river Ravi in Chamba district, Himachal Pradesh. The dam fulfils the hydroelectric needs of the region.   The Tulbul Project– The project has been commissioned on the river Jhelum in the state of Jammu and Kashmir. Wular Barrage on the Wular Lake(a Ramsar wetland site) is constructed.   Incorrect Solution (c) Match 1 Match 2 Match 3 Match 4 Incorrect Correct Incorrect Correct The Uri Dam– The dam has been constructed on the river Jhelum in the state of Jammu and Kashmir.   The Salal Dam– The dam has been constructed on the river Chenab at Dhyangarh in the state of Jammu and Kashmir.   The Chamera Dam– The dam has been constructed on the river Ravi in Chamba district, Himachal Pradesh. The dam fulfils the hydroelectric needs of the region.   The Tulbul Project– The project has been commissioned on the river Jhelum in the state of Jammu and Kashmir. Wular Barrage on the Wular Lake(a Ramsar wetland site) is constructed.   Question 21 of 35 21. Question Consider the following statements: The International Civil Aviation Organization (ICAO)has established the Carbon Offsetting and Reduction Scheme for International Aviation (CORSIA) to address aviation emissions. TheWorld Economic Forum has launched the Clean Skies for Tomorrow initiative, which aims to accelerate the production and use of Sustainable Aviation Fuel (SAF). Choose the correct code: a) 1 only b) 2 only c) Both 1 and 2 d) Neither 1 nor 2 Correct Solution (c) Sustainable Aviation Fuel (SAF), also referred to as bio-jet fuel, is created using domestically developed methods using cooking oil and oil-rich seeds from plants. Scaling up the production and use of SAF in India can bring several benefits, including reducing GHG emissions, improving air quality, enhancing energy security, creating jobs in the renewable energy sector, and promoting sustainable development. The International Civil Aviation Organization (ICAO) has established the Carbon Offsetting and Reduction Scheme for International Aviation (CORSIA) to address aviation emissions. Hence statement 1 is correct. The World Economic Forum has launched the Clean Skies for Tomorrow initiative, which aims to accelerate the production and use of Sustainable Aviation Fuel (SAF). Hence statement 2 is correct. Incorrect Solution (c) Sustainable Aviation Fuel (SAF), also referred to as bio-jet fuel, is created using domestically developed methods using cooking oil and oil-rich seeds from plants. Scaling up the production and use of SAF in India can bring several benefits, including reducing GHG emissions, improving air quality, enhancing energy security, creating jobs in the renewable energy sector, and promoting sustainable development. The International Civil Aviation Organization (ICAO) has established the Carbon Offsetting and Reduction Scheme for International Aviation (CORSIA) to address aviation emissions. Hence statement 1 is correct. The World Economic Forum has launched the Clean Skies for Tomorrow initiative, which aims to accelerate the production and use of Sustainable Aviation Fuel (SAF). Hence statement 2 is correct. Question 22 of 35 22. Question Consider the following pairs: Volcano Country Marapi Volcano Japan Mount Etna Italy Mauna Loa Hawaii Sakurajima Volcano Indonesia How many pairs are correctly matched? a) One pair b) Two pairs c) Three pairs d) Four pairs Correct Solution (b) Volcano Country Marapi Volcano Indonesia Mount Etna Italy Mauna Loa Hawaii Sakurajima Volcano Japan Hence option b is correct. Incorrect Solution (b) Volcano Country Marapi Volcano Indonesia Mount Etna Italy Mauna Loa Hawaii Sakurajima Volcano Japan Hence option b is correct. Question 23 of 35 23. Question Consider the following statements about the Biodiversity Heritage Sites (BHS): They arerecognised under the Environment Protection Act of 1986. They are notified by the central government in consultation with the state government. Which of the statements given above is/are correct? a) 1 only b) 2 only c) Both 1 and 2 d) Neither 1 nor 2 Correct Solution (d) The Biodiversity Heritage Sites (BHS) are areas that are unique, ecologically fragile ecosystems that have a rich biodiversity. BHS must comprise any one or more of the following components – species richness, high endemism, presence of rare, endemic, and threatened species, keystone species, species of evolutionary significance, wild ancestors of domestic/cultivated species, etc. They are recognised under Section 37 of the Biological Diversity Act, of 2002. Hence statement 1 is incorrect. The creation of BHS may not put any restriction on the prevailing practices and usages of the local communities, other than those voluntarily decided by them. The purpose is to enhance the quality of life of the local communities through conservation measures. They are notified by the State Government in consultation with the local bodies. Hence statement 2 is incorrect. Nallur Tamarind Grove in Bengaluru, Karnataka was the first Biodiversity Heritage Site of India, declared in 2007. Incorrect Solution (d) The Biodiversity Heritage Sites (BHS) are areas that are unique, ecologically fragile ecosystems that have a rich biodiversity. BHS must comprise any one or more of the following components – species richness, high endemism, presence of rare, endemic, and threatened species, keystone species, species of evolutionary significance, wild ancestors of domestic/cultivated species, etc. They are recognised under Section 37 of the Biological Diversity Act, of 2002. Hence statement 1 is incorrect. The creation of BHS may not put any restriction on the prevailing practices and usages of the local communities, other than those voluntarily decided by them. The purpose is to enhance the quality of life of the local communities through conservation measures. They are notified by the State Government in consultation with the local bodies. Hence statement 2 is incorrect. Nallur Tamarind Grove in Bengaluru, Karnataka was the first Biodiversity Heritage Site of India, declared in 2007. Question 24 of 35 24. Question Consider the following statements about the Arsenic and Metal Removal by Indian Technology (AMRIT): It is a water purifier developed for the removal of arsenic and metal ions from water. It can be used for both domestic and community levels. It is developed by the Ministry of Jal Shakti. How many of the above statements are correct? a) Only one b) Only two c) All three d) None Correct Solution (b) The Arsenic and Metal Removal by Indian Technology (AMRIT) is a water purifier developed for the removal of arsenic and metal ions from water. Hence statement 1 is correct. The technology uses nano-scale iron oxy-hydroxide which selectively removes arsenic when water is passed through it. It can be used for both domestic and community levels. Hence statement 2 is correct. It was developed by the Indian Institute of Technology (IIT), Madras. Hence statement 3 is incorrect.   Incorrect Solution (b) The Arsenic and Metal Removal by Indian Technology (AMRIT) is a water purifier developed for the removal of arsenic and metal ions from water. Hence statement 1 is correct. The technology uses nano-scale iron oxy-hydroxide which selectively removes arsenic when water is passed through it. It can be used for both domestic and community levels. Hence statement 2 is correct. It was developed by the Indian Institute of Technology (IIT), Madras. Hence statement 3 is incorrect.   Question 25 of 35 25. Question Consider the following statements about Kawal Tiger Reserve: It forms part of the Deccan peninsula-central highlands. It has the catchment for the rivers Narmada and Tapi. It has connectivity to the Tadoba-Andhari Tiger Reserve of Maharashtra. Its vegetation includes southern tropical dry deciduous forests. How many of the above statements are correct? a) Only one b) Only two c) Only three d) All four Correct Solution (c) Kawal Tiger Reserve forms part of the Deccan peninsula-central highlands. Hence statement 1 is correct. It is located in the north-eastern part of Telangana, with the Godavari River on one side and the Maharashtra border on the other. It has the catchment for the rivers Godavari and Kadam, which flow towards the south of the sanctuary. Hence statement 2 is incorrect. It has connectivity to the Tadoba-Andhari Tiger Reserve of Maharashtra in the North and to the Indravati Tiger Reserve of Chhattisgarh towards its north-eastern side. Hence statement 3 is correct. It has diverse habitats, comprising dense forests, grasslands, open areas, rivers, streams, and water bodies. Its vegetation includes southern tropical dry deciduous forests. Hence statement 4 is correct. Incorrect Solution (c) Kawal Tiger Reserve forms part of the Deccan peninsula-central highlands. Hence statement 1 is correct. It is located in the north-eastern part of Telangana, with the Godavari River on one side and the Maharashtra border on the other. It has the catchment for the rivers Godavari and Kadam, which flow towards the south of the sanctuary. Hence statement 2 is incorrect. It has connectivity to the Tadoba-Andhari Tiger Reserve of Maharashtra in the North and to the Indravati Tiger Reserve of Chhattisgarh towards its north-eastern side. Hence statement 3 is correct. It has diverse habitats, comprising dense forests, grasslands, open areas, rivers, streams, and water bodies. Its vegetation includes southern tropical dry deciduous forests. Hence statement 4 is correct. Question 26 of 35 26. Question Consider the following statements about the Project Q*: It refers to anew artificial intelligence model being developed the OpenAI. It demonstrates advanced reasoning capabilities similar to humans. Virtual assistants and spam filters are examples of general artificial intelligence. Self-driving cars and advanced chatbots are examples of narrow artificial intelligence. Choose the correct code: a) Only one b) Only two c) Only three d) All four Correct Solution (b) Project Q* refers to a new artificial intelligence model being developed the OpenAI. Hence statement 1 is correct. It demonstrates advanced reasoning capabilities similar to humans. Hence statement 2 is correct. Virtual assistants and spam filters are examples of narrow artificial intelligence. Hence statement 3 is incorrect. Self-driving cars and advanced chatbots are examples of general artificial intelligence. Hence statement 4 is incorrect. Specifications Narrow AI General AI Learning ability Narrow AI focuses on a single task under various constraints and limitations. AGI is envisioned to have the capacity for generalization and adaptation across diverse domains. Operational domain Fixed domain models provided by programmers Self-learn and reasons with its operating environment Data processing Narrow AI analyses data by using ML, Natural Language Processing, Deep Learning, and Artificial Neural Networks General AI uses an advanced version of these technologies for analysis. Limitation Limited to a single task, lacks broader intelligence May lack depth in certain areas and have ethical concerns Incorrect Solution (b) Project Q* refers to a new artificial intelligence model being developed the OpenAI. Hence statement 1 is correct. It demonstrates advanced reasoning capabilities similar to humans. Hence statement 2 is correct. Virtual assistants and spam filters are examples of narrow artificial intelligence. Hence statement 3 is incorrect. Self-driving cars and advanced chatbots are examples of general artificial intelligence. Hence statement 4 is incorrect. Specifications Narrow AI General AI Learning ability Narrow AI focuses on a single task under various constraints and limitations. AGI is envisioned to have the capacity for generalization and adaptation across diverse domains. Operational domain Fixed domain models provided by programmers Self-learn and reasons with its operating environment Data processing Narrow AI analyses data by using ML, Natural Language Processing, Deep Learning, and Artificial Neural Networks General AI uses an advanced version of these technologies for analysis. Limitation Limited to a single task, lacks broader intelligence May lack depth in certain areas and have ethical concerns Question 27 of 35 27. Question Consider the following statements about the Cyber Surakshit Bharat Initiative: It is an initiative of the Ministry of Electronics and Information Technology (MeitY). It is an initiative to fortify the cyber security system in India with regard to the Government’s vision of a Digital India. It will be operated on three principles – education, awareness, and enablement. How many of the above statements are correct? a) Only one b) Only two c) All three d) None Correct Solution (c) The Cyber Surakshit Bharat Initiative is an initiative of the Ministry of Electronics and Information Technology (MeitY). Hence statement 1 is correct. It was launched in cooperation with the National e-Governance Division (NeGD) and various industry partners in India. It is an initiative to fortify the cyber security system in India with regard to the Government’s vision of a Digital India. Hence statement 2 is correct. It can be entitled as the first public-private enterprise of its kind. The partners involved in the origination of this scheme include chief IT companies like Intel, Microsoft, etc. It will be operated on three principles – education, awareness, and enablement. Hence statement 3 is correct. It will comprise a program of awareness on the importance of cybersecurity. The scheme will also include a number of workshops on the best enablement and practices of the officials with cybersecurity health tool kits for the management and mitigation of cyber threats. Incorrect Solution (c) The Cyber Surakshit Bharat Initiative is an initiative of the Ministry of Electronics and Information Technology (MeitY). Hence statement 1 is correct. It was launched in cooperation with the National e-Governance Division (NeGD) and various industry partners in India. It is an initiative to fortify the cyber security system in India with regard to the Government’s vision of a Digital India. Hence statement 2 is correct. It can be entitled as the first public-private enterprise of its kind. The partners involved in the origination of this scheme include chief IT companies like Intel, Microsoft, etc. It will be operated on three principles – education, awareness, and enablement. Hence statement 3 is correct. It will comprise a program of awareness on the importance of cybersecurity. The scheme will also include a number of workshops on the best enablement and practices of the officials with cybersecurity health tool kits for the management and mitigation of cyber threats. Question 28 of 35 28. Question Consider the following statements about Stonehenge: It is a massive stone circle monument located on Salisbury Plain in southern England. It was declared a UNESCO World Heritage Site of outstanding universal value in 1986. Choose the correct code: a) 1 only b) 2 only c) Both 1 and 2 d) Neither 1 nor 2 Correct Solution (c) Stonehenge is a massive stone circle monument located on Salisbury Plain in southern England. Hence statement 1 is correct. Work started on this super stone circle around 5,000 years ago in the late Neolithic Age, but it took over 1,000 years to build in four long stages. Archaeologists believe the final changes were made around 1,500 BC, in the early Bronze Age. It was declared a UNESCO World Heritage Site of outstanding universal value in 1986. Hence statement 2 is correct. Though there is no definite evidence as to the intended purpose of Stonehenge, it was presumably a religious site and an expression of the power and wealth of the chieftains, aristocrats, and priests who had it built. Incorrect Solution (c) Stonehenge is a massive stone circle monument located on Salisbury Plain in southern England. Hence statement 1 is correct. Work started on this super stone circle around 5,000 years ago in the late Neolithic Age, but it took over 1,000 years to build in four long stages. Archaeologists believe the final changes were made around 1,500 BC, in the early Bronze Age. It was declared a UNESCO World Heritage Site of outstanding universal value in 1986. Hence statement 2 is correct. Though there is no definite evidence as to the intended purpose of Stonehenge, it was presumably a religious site and an expression of the power and wealth of the chieftains, aristocrats, and priests who had it built. Question 29 of 35 29. Question Which of the following products have obtained Geographical Indication from Meghalaya? Tawlhlohpuan Lakadong Turmeric Garo Dakmanda Larnai Pottery Choose the correct code: a) Only one b) Only two c) Only three d) All four Correct Solution (c) The following products have obtained Geographical Indication from Meghalaya Lakadong Turmeric Garo Dakmanda Larnai Pottery Garo Chubitchi Tawlhlohpuan – Mizoram Hence option c is correct.   Incorrect Solution (c) The following products have obtained Geographical Indication from Meghalaya Lakadong Turmeric Garo Dakmanda Larnai Pottery Garo Chubitchi Tawlhlohpuan – Mizoram Hence option c is correct.   Question 30 of 35 30. Question Consider the following statements about the Painganga River: It originates in the Ajantha ranges in Madhya Pradesh. It is a major tributary of the Wardha River. Its major tributaries include the Adan, Kas, and Arunavati. How many of the above statements are correct? a) Only one b) Only two c) All three d) None Correct Solution (b) The Painganga River originates in the Ajantha ranges in Maharashtra. Hence statement 1 is incorrect. It provides irrigation to the Washim and Yavatmal districts in Maharashtra. It is a major tributary of the Wardha River. Hence statement 2 is correct. The Wardha River flows into the Wain Ganga River to form the Pranhita River, which finally joins the Godavari River. Its major tributaries include the Adan, Kas, Kayadhu, Pus, and Arunavati. Hence statement 3 is correct. Two dams are being constructed on the river, namely Upper Painganga and Lower Painganga. This dam is also known as the Isapur Dam.   Incorrect Solution (b) The Painganga River originates in the Ajantha ranges in Maharashtra. Hence statement 1 is incorrect. It provides irrigation to the Washim and Yavatmal districts in Maharashtra. It is a major tributary of the Wardha River. Hence statement 2 is correct. The Wardha River flows into the Wain Ganga River to form the Pranhita River, which finally joins the Godavari River. Its major tributaries include the Adan, Kas, Kayadhu, Pus, and Arunavati. Hence statement 3 is correct. Two dams are being constructed on the river, namely Upper Painganga and Lower Painganga. This dam is also known as the Isapur Dam.   Question 31 of 35 31. Question The ratio of Adam’s age 4 years ago and Eve’s age after 4 years is 1: 1. Presently, the ratio of their ages is 4 : 3. Find the ratio between Adam’s age 4 years hence and Eve’s age 4 years ago. a) 4: 3 b) 3: 4 c) 9: 5 d) 7: 3 Correct Solution (c) Currently, the ratio of their ages is 4: 3. Suppose, their ages are: 4x and 3x. Adam’s age 4 years ago = 4x – 4 Eve’s age after 4 years = 3x + 4 The ratio of Adam’s age 4 years ago and Eve’s age after 4 years is 1: 1 Therefore, = 4x−4/(3x+4) = 1/1 Solving, we get x = 8 We are required to find the ratio between Adam’s age 4 years hence and Eve’s age 4 years ago. Adam’s age: (4x + 4) Eve’s age: (3x – 4) Putting the value of x, we get: = 4x+4/ (3x−4) = [(4×8)+4] [(3×8)−4] = 9/5 So, the required ratio is 9: 5   Incorrect Solution (c) Currently, the ratio of their ages is 4: 3. Suppose, their ages are: 4x and 3x. Adam’s age 4 years ago = 4x – 4 Eve’s age after 4 years = 3x + 4 The ratio of Adam’s age 4 years ago and Eve’s age after 4 years is 1: 1 Therefore, = 4x−4/(3x+4) = 1/1 Solving, we get x = 8 We are required to find the ratio between Adam’s age 4 years hence and Eve’s age 4 years ago. Adam’s age: (4x + 4) Eve’s age: (3x – 4) Putting the value of x, we get: = 4x+4/ (3x−4) = [(4×8)+4] [(3×8)−4] = 9/5 So, the required ratio is 9: 5   Question 32 of 35 32. Question There is a Hockey match of Dhyanchand tomorrow, at Motera stadium. In recent years, it has rained only 73 days each year. Unfortunately, the weatherman has predicted rain for tomorrow. When it actually rains, the weatherman correctly forecasts rain 80% of the time. When it doesn’t rain, he incorrectly forecasts rain 20% of the time. What is the probability that it will rain on the day of Dhyanchand’s Hockey match? a) 1/5 b) 1/3 c) 1/2 d) 1/4 Correct Solution (c) The sample space is defined by two mutually-exclusive events – it rains or it does not rain. Additionally, a third event occurs when the weatherman predicts rain. Notation for these events appears below. Event A1 It rains on Dhyanchand’s Hockey match. Event A2 It does not rain on Dhyanchand’s Hockey match Event B The weatherman predicts rain. In terms of probabilities, we know the following: P (A1) = 73/365 = 0.2 [It rains 73 days out of the year.] P (A2) = 292/365 = 0.8 [It does not rain 292 days out of the year.] P (B|A1) = 0.8 [When it rains, the weatherman predicts rain 80% of the time.] P (B|A2) = 0.2 [When it does not rain, the weatherman predicts rain 20% of the time.] We want to know P (A1 |B), the probability it will rain on the day of Dhyanchand’s Hockey match, given a forecast for rain by the weatherman. The answer can be determined from Bayes’ theorem, as shown below. P(A1 |B) = [P(A1 ) P(B|A1 )] [P(A1 )P(B|A1 )+ P(A2 )P(B|A2 )] P(A1 |B) = [0.2 × 0.8] [(0.2 × 0.8)+ (0.8 × 0.2)] = 0.2 × 0.8 2(0.2 × 0.8) = ½ Incorrect Solution (c) The sample space is defined by two mutually-exclusive events – it rains or it does not rain. Additionally, a third event occurs when the weatherman predicts rain. Notation for these events appears below. Event A1 It rains on Dhyanchand’s Hockey match. Event A2 It does not rain on Dhyanchand’s Hockey match Event B The weatherman predicts rain. In terms of probabilities, we know the following: P (A1) = 73/365 = 0.2 [It rains 73 days out of the year.] P (A2) = 292/365 = 0.8 [It does not rain 292 days out of the year.] P (B|A1) = 0.8 [When it rains, the weatherman predicts rain 80% of the time.] P (B|A2) = 0.2 [When it does not rain, the weatherman predicts rain 20% of the time.] We want to know P (A1 |B), the probability it will rain on the day of Dhyanchand’s Hockey match, given a forecast for rain by the weatherman. The answer can be determined from Bayes’ theorem, as shown below. P(A1 |B) = [P(A1 ) P(B|A1 )] [P(A1 )P(B|A1 )+ P(A2 )P(B|A2 )] P(A1 |B) = [0.2 × 0.8] [(0.2 × 0.8)+ (0.8 × 0.2)] = 0.2 × 0.8 2(0.2 × 0.8) = ½ Question 33 of 35 33. Question A four-digit number N is of the form XYZZ. Each letter stands for a digit. The successor of N is of the form XPQQ. The predecessor of N is of the form XYZU then the value of Z + Q + U? a) 12 b) 15 c) 17 d) 21 Correct Solution (b) The above given conditions are possible only when U = 8, Z = 9 and Q = 0. Sum of these digits is 17. Incorrect Solution (b) The above given conditions are possible only when U = 8, Z = 9 and Q = 0. Sum of these digits is 17. Question 34 of 35 34. Question Find the least value of “b” for which 67b326 is divisible by 3? a) 3 b) 0 c) 2 d) 1 Correct Solution (b) The required number is b. Then, = 6+7+b+3+2+6=24+b is divisible by 3. Therefore, least value of b is 0.   Incorrect Solution (b) The required number is b. Then, = 6+7+b+3+2+6=24+b is divisible by 3. Therefore, least value of b is 0.   Question 35 of 35 35. Question The sum of the digits of a two-digit number is 11. If 45 is added to the number, then the digits are reversed. Find the number. a) 65 b) 56 c) 38 d) 74 Correct Solution (c) Given x + y = 11………(i) 10x + y + 45 = 10y + x ……….. (ii) From equation (i) and (ii), we get X = 3 and y =8 So, the number is 38.   Incorrect Solution (c) Given x + y = 11………(i) 10x + y + 45 = 10y + x ……….. (ii) From equation (i) and (ii), we get X = 3 and y =8 So, the number is 38.   window.wpProQuizInitList = window.wpProQuizInitList || []; window.wpProQuizInitList.push({ id: '#wpProQuiz_3619', init: { quizId: 3619, mode: 1, globalPoints: 70, timelimit: 1800, resultsGrade: [0], bo: 704, qpp: 0, catPoints: [70], formPos: 0, lbn: "Test-summary", json: {"31840":{"type":"single","id":31840,"catId":0,"points":2,"correct":[1,0,0,0]},"31843":{"type":"single","id":31843,"catId":0,"points":2,"correct":[0,0,0,1]},"31846":{"type":"single","id":31846,"catId":0,"points":2,"correct":[0,1,0,0]},"31848":{"type":"single","id":31848,"catId":0,"points":2,"correct":[0,0,0,1]},"31849":{"type":"single","id":31849,"catId":0,"points":2,"correct":[0,0,0,1]},"31852":{"type":"single","id":31852,"catId":0,"points":2,"correct":[1,0,0,0]},"31853":{"type":"single","id":31853,"catId":0,"points":2,"correct":[0,1,0,0]},"31855":{"type":"single","id":31855,"catId":0,"points":2,"correct":[1,0,0,0]},"31858":{"type":"single","id":31858,"catId":0,"points":2,"correct":[0,1,0,0]},"31859":{"type":"single","id":31859,"catId":0,"points":2,"correct":[0,0,0,1]},"31861":{"type":"single","id":31861,"catId":0,"points":2,"correct":[0,1,0,0]},"31864":{"type":"single","id":31864,"catId":0,"points":2,"correct":[0,0,1,0]},"31867":{"type":"single","id":31867,"catId":0,"points":2,"correct":[1,0,0,0]},"31870":{"type":"single","id":31870,"catId":0,"points":2,"correct":[0,0,0,1]},"31871":{"type":"single","id":31871,"catId":0,"points":2,"correct":[0,0,1,0]},"31874":{"type":"single","id":31874,"catId":0,"points":2,"correct":[0,0,1,0]},"31877":{"type":"single","id":31877,"catId":0,"points":2,"correct":[1,0,0,0]},"31878":{"type":"single","id":31878,"catId":0,"points":2,"correct":[0,0,0,1]},"31879":{"type":"single","id":31879,"catId":0,"points":2,"correct":[0,0,1,0]},"31882":{"type":"single","id":31882,"catId":0,"points":2,"correct":[0,0,1,0]},"31883":{"type":"single","id":31883,"catId":0,"points":2,"correct":[0,0,1,0]},"31886":{"type":"single","id":31886,"catId":0,"points":2,"correct":[0,1,0,0]},"31887":{"type":"single","id":31887,"catId":0,"points":2,"correct":[0,0,0,1]},"31889":{"type":"single","id":31889,"catId":0,"points":2,"correct":[0,1,0,0]},"31891":{"type":"single","id":31891,"catId":0,"points":2,"correct":[0,0,1,0]},"31893":{"type":"single","id":31893,"catId":0,"points":2,"correct":[0,1,0,0]},"31894":{"type":"single","id":31894,"catId":0,"points":2,"correct":[0,0,1,0]},"31896":{"type":"single","id":31896,"catId":0,"points":2,"correct":[0,0,1,0]},"31897":{"type":"single","id":31897,"catId":0,"points":2,"correct":[0,0,1,0]},"31898":{"type":"single","id":31898,"catId":0,"points":2,"correct":[0,1,0,0]},"31901":{"type":"single","id":31901,"catId":0,"points":2,"correct":[0,0,1,0]},"31902":{"type":"single","id":31902,"catId":0,"points":2,"correct":[0,0,1,0]},"31903":{"type":"single","id":31903,"catId":0,"points":2,"correct":[0,1,0,0]},"31904":{"type":"single","id":31904,"catId":0,"points":2,"correct":[0,1,0,0]},"31907":{"type":"single","id":31907,"catId":0,"points":2,"correct":[0,0,1,0]}} } }); All the Best IASbaba

DAILY CURRENT AFFAIRS IAS | UPSC Prelims and Mains Exam – 8th April 2024

Archives (PRELIMS & MAINS Focus)   Netaji Subhash Chandra Bose Island Syllabus Prelims – Geography Context: The unchecked population growth of spotted deer (Chital) is creating problems in Netaji Subhash Chandra Boss Island. Background:- Introduced to the Andaman and Nicobar Islands for game hunting in the early 1900s by the British, the herbivore multiplied unchecked for years in the absence of large predators.It has become an “invasive” problem for the Union Territory’s authorities. About  Netaji Subhash Chandra Bose Island Earlier Ross Island, now officially known as Netaji Subhash Chandra Bose Island, is an island of the Andaman Islands. The Island is situated three kilometres East of Central Port Blair. It belongs to the South Andaman administrative district, Andaman and Nicobar Islands. The Andaman and Nicobar Islands are strategically located in the Indian Ocean between the Indian mainland, Myanmar, and Indonesia, and are close to important maritime routes. The islands were first controlled by the Dutch, then by the British, and were taken over by the advancing Japanese military during World War II. The Azad Hind government was able to obtain de jure control of the islands from the Japanese by the end of 1943, and Subhash Chandra Bose arrived in Port Blair on December 29. Keeping in mind the historical significance of the Andaman & Nicobar Islands and to honour the memory of Netaji Subhas Chandra Bose, Ross Islands was renamed as Netaji Subhash Chandra Bose Dweep by Prime Minister during his visit to the Island in 2018. Neil Island and Havelock Island were also renamed Shaheed Dweep and Swaraj Dweep. All the three islands are major tourist spots. Source: Indian Express Green Hydrogen Syllabus Prelims & Mains – Environment Context: The Ministry of New and Renewable Energy (MNRE) has announced a Rs-496-crore (until 2025-26) scheme to support pilot projects that either test the viability of green hydrogen as a vehicle fuel or develop secure supporting infrastructure such as refuelling stations. Background: Big Indian commercial vehicle manufacturers such as Tata Motors, Volvo Eicher, and Ashok Leyland are doubling down on efforts to develop hydrogen-powered trucks and buses by ramping up research and development, and building manufacturing capacities. About Green Hydrogen : Hydrogen is colourless, and green hydrogen is ‘green’ only by virtue of the way it is produced, and the source of the energy used to manufacture it. Green hydrogen refers to hydrogen that is produced from the electrolysis of water — splitting it into hydrogen and oxygen — using an electrolyser powered by renewable energy.This is considered to be a virtually emission-free pathway for hydrogen production — it is ‘end-to-end’ green because it is powered by green energy, uses water as feedstock, and emits no carbon on consumption. Currently, most hydrogen produced for industrial consumption and applications is ‘grey’ hydrogen, which is produced from natural gas through energy-intensive processes, and has high carbon emissions.Except for a difference in the production pathway and emissions, green hydrogen is essentially the same as grey — or hydrogen categorised by any other colour. Challenges to the large-scale use of green hydrogen in the transportation sector: The foremost among challenges is the prohibitive cost of production, followed by challenges of storage and transportation at scale. Green hydrogen-powered vehicles are not yet seen as a suitable alternative to four-wheel battery electric vehicles (BEVs) due to challenges arising from fuel costs and building supporting infrastructure. Hydrogen is extremely flammable, which means that special care would be needed in handling the fuel at retail stations compared to diesel, petrol, or even CNG. Robust and fool-proof handling and safety standards need to be developed before pushing large-scale adoption. Currently, most cylinders manufactured in India are designed to carry compressed natural gas (CNG). But hydrogen is stored at a much higher pressure, and CNG cylinders cannot carry hydrogen. For cylinders to carry a high mass of hydrogen, the carbon fibre needs to be stronger, which makes high-pressure hydrogen cylinders expensive. This is a key barrier to the adoption of hydrogen as a transport fuel. For the same reason, the existing natural gas pipeline infrastructure is also not seen as viable. Source: Indian Express RAKHIGARHI Syllabus Prelims – History Context: The NCERT recently introduced certain revisions to the history syllabus of Class 12 students, highlighting that Harappans were based out in Rakhigarhi. Background: These revisions reflect a deeper understanding of the Harappan civilization and its historical context, shedding light on their origins and societal structures. About RAKHIGARHI Rakhigarhi is an archaeological site located in Hisar, Haryana, and is recognized as one of the largest Harappan (Indus Valley Civilization) sites in the Indian subcontinent. Rakhigarhi is situated in the plains of the Saraswati River, about 27 km from the seasonal Ghaggar river. The site showcases continuity from the Harappan age to the present times. It is recognized for its extensive Harappan heritage, offering insights into ancient urban life and culture. Excavations were carried out at Rakhigarhi to study its evolution from 6000 BC (Pre-Harappan phase) to 2500 BC. In order to study the genetic history of the Harappans, DNA was extracted from the skeletal remains excavated at Rakhigarhi. Previous Year Question Q1. Which of the following characterizes/characterize the people of Indus Civilization? They possessed great palaces and temples. They worshipped both male and female deities. They employed horse-drawn chariots in warfare. Select the correct statement/statements using the codes given below: 1 and 2 only 2 only 1, 2 and 3 None of the statements given above is correct Source: Hindu DEMOGRAPHIC DIVIDEND Syllabus Prelims – Economy Context: The World Bank warned that South Asian nations run the risk of “squandering its demographic dividend” because job creation is not keeping up with the growth in the working-age population. Background: According to data in the report, between 2000 and 2023, When the working-age population grew by an average of 19 million per year, the region added 10 million employments annually on average. About DEMOGRAPHIC DIVIDEND: The United Nations Population Fund defines demographic dividend as “the economic growth potential that can result from shifts in a population’s age structure, mainly when the share of the working-age population is larger than the non-working-age share of the population”. The demographic dividend in India presents various opportunities related to economic growth due to increased economic activities from a higher working-age population and lower dependent population. Features of India’s Demographic Dividend: Large and Growing Working-Age Population: The working-age population (15-64 years old) in India has surpassed the dependent population since 2018. Uniqueness: The window of opportunity for India’s demographic dividend spans five decades from 2005-06 to 2055-56, which is longer than any other country in the world. Increasing Education Levels: Education levels in India are rising, providing a more skilled and productive workforce. Increasing Gender Equality: There has been significant progress in recent years toward gender equality, including increasing levels of education and workforce participation among women. Source: Business Standard Previous Year Question India is regarded as a country of ‘Demographic Dividend’. This is due to: Its high population in the age group below 15 years Its high population in the age group of 15-64 years Its high population in the age group above 65 years Its high total population PROJECT AKASHTEER Syllabus Prelims – Current Event Context: The Indian Army has initiated the induction of control and reporting systems under ‘Project Akashteer’ to bolster its air defence capabilities. Background: Akashteer revolutionizes air defence operations, empowering India’s armed forces with cutting-edge capabilities. About PROJECT AKASHTEER: The ‘Project Akashteer’ is a significant initiative aimed at enhancing the air defence capabilities of the Indian Army. It is developed by Bharat Electronics Limited (BEL) as part of the ‘Atmanirbhar Bharat’ initiative. This project is designed to automate air defence control and reporting processes by digitizing them. The system integrates radar and communication systems at all levels into a unified network, providing an unprecedented level of situational awareness and control. This enables swift engagement of hostile targets, significantly reduces the risk of fratricide, and ensures the safety of friendly aircraft in contested airspace. A noteworthy aspect of ‘Akashteer’ is its emphasis on mobility and resilience. The system’s control centers, designed to be vehicle-based and mobile, can maintain operational capabilities even in challenging communication environments. The project is expected to significantly enhance the operational efficiency and integration of the Army’s air defence mechanisms. The total cost of the project is nearly Rs 1,982 crore. Source: Economic Times India – Italy Relations Syllabus Mains – GS 2 Context: The Union Cabinet had  approved the Migration and Mobility Agreement between India and Italy last December. Background: India and Italy are ancient civilisations with links going back 2,000 years. Italian port cities were important trading posts on the spice route. The Venetian merchant Marco Polo traveled to India in the 13th century and wrote about his experiences.After Independence, political relations between India and Italy were established in 1947. Since then, there has been a regular exchange of visits at political and official levels between both countries, including several visits by Heads of States. India and Italy Relations: There has been a regular exchange of visits at political and official levels between both countries. For instance, the Indian PM paid his first official visit to Italy in October 2021 to attend the G20 Summit. In March 2023, the Italian PM paid her first-ever state visit to India as the guest of honour of Raisina Dialogue. Italy is India’s 4th largest trading partner in the EU. The bilateral trade between the two countries was valued at US$ 14.25 billion in 2022-23, with the balance of trade being in India’s favour. India – Italy Military Cooperation Group (MCG) is a forum established to boost defence cooperation. India and Italy signed a defence cooperation agreement (in 2023) to promote cooperation in varied defence domains such as security and defence policy, and defence industrial cooperation among others. The relationship between India and Italy was elevated to a Strategic Partnership in March 2023 during the visit of the Italian Prime Minister to India. A connection was identified between the Indo-Pacific, in which India plays a leading role, and the enlarged Mediterranean where Italy sits at the centre of the Mediterranean Sea and acts as a natural bridge towards the Indo-Pacific. Italy’s decision to withdraw from the Belt and Road Initiative aligns with common goals in global connectivity between Rome and New Delhi. Italy supported India’s entry into the Wassenaar Arrangement and Australia Group. The Blue-Raman project (with an Italian company at its core) will bring the Mediterranean and the Indian Ocean closer in exchange for digital data through a submarine cable system. Italy supported India’s major initiatives including the International Solar Alliance (ISA), the India-Middle East-Europe Economic Corridor (IMEC), and the Global Biofuels Alliance (GBA). Concerns/Challenges/Issues in India-Italy Relations: India and Italy, despite having historical trade ties dating back to the Roman era have relatively low the current volume of trade and investment. Italian Mariners’ case sparked a conflict over legal jurisdiction and functional immunity, which ended only in 2020 with the verdict of the Permanent Court of Arbitration. In the Agusta Westland helicopter scandal, the defence firm owned by Italian company Leonardo was accused of bribery, resulting in India cancelling the procurement deal and banning Leonardo from the Indian defence market in 2015. In 2021, the India-Italy-Japan trilateral partnership was launched yet it has not been operationalised. Way Forward: Strengthen trade ties to increase trade volume, explore new avenues for investment, and promote economic cooperation. If the India and EU sign a trade pact in the coming time, Italy and India can enjoy major two-way trade benefits. It would also add fodder to the reinvigorated EU-India partnership. Italy and India can converge on multilateral issues, sharing a common imperative to shape and endorse a new global agenda together. Celebrate and promote the culinary heritage of both countries through food festivals and gastronomic events, with Italian pasta and pizza finding a place on Indian tables, and Indian spices adding a punch to Italian dishes. Source: MEA Practice MCQs Daily Practice MCQs Q1.) With reference to the ‘Project Akashteer’, consider the following statements: The ‘Project Akashteer’ is a significant initiative aimed at enhancing the air defence capabilities of the Indian Army. It is developed by Bharat Electronics Limited (BEL) as part of the ‘Atmanirbhar Bharat’ initiative. This project is designed to automate air defence control and reporting processes by digitizing them. How many of the statements given above are correct? Only one Only two All three None Q2.)  Consider the following: Growing working-age population Increasing education levels Increasing gender equality How many of the above are the features of India’s demographic dividend? Only one Only two All three None Q3.)With reference to Rakhigarhi, consider the following statements: Rakhigarhi is recognized as one of the largest Harappan sites in the Indian subcontinent. It is located in Rajasthan. The site showcases continuity from the Harappan age to the present times. How many of the statements given above are correct? Only one Only two All three None Comment the answers to the above questions in the comment section below!! ANSWERS FOR ’  8th April  2024 – Daily Practice MCQs’ will be updated along with tomorrow’s Daily Current Affairs.st ANSWERS FOR  6th April – Daily Practice MCQs Answers- Daily Practice MCQs Q.1) – d Q.2) – c Q.3) – b